SlideShare una empresa de Scribd logo
MARAVILLOSOS
PROBLEMAS DE
MATEMÁTICAS
Libro 14
http://matemelga.wordpress.com/
Ana, Blanca y Carmen tienen entre las tres 490 monedas de 1 euro.
Ana gastó la quinta parte de sus monedas, Blanca gastó la tercera parte de sus monedas
y Carmen gastó la cuarta parte de sus monedas.
Ahora las tres chicas tienen todas igual cantidad de monedas.
¿Cuántas monedas tenía inicialmente cada una?
SOLUCIÓN
Sean , , la cantidad de monedas que tienen, respectivamente, Ana, Blanca y Carmen.
Según el enunciado,
+ + = 490
= = ⇒
+ + = 490
= =
!
=
!
⇒ +
!
+
!
=
" #" !
=
$
= 490 ⇒
⇒ =
$ ×
$
⇒ = 150 ⇒ (
=
!
=
!
× 150 ⇒ = 180
=
!
=
!
× 150 ⇒ = 160
Por lo tanto,
Ana tenía 150 monedas
Blanca tenía 180 monedas
Carmen tenía 160 monedas
Calcula la superficie del rectángulo de la figura.
SOLUCIÓN
Nombramos los elementos auxiliares que se ven en la figura adjunta y observamos
que hay tres triángulos rectángulos a los que aplicamos el teorema de Pitágoras:
+ =
+ 6 = 2
+ 2 − = 6
ª ª + + 36 = 4
+ 4 − 4 + = 36
ª ª
ª ª
+ 4 − 4 + − + + 36 = 36 − 4 ⇒ 4 − 4 − 36 = 36 − 4 ⇒
⇒ 8 − 4 − 72 = 0
÷"
2 − − 18 = 0 ⇒ × 2 − = 18
Y el área del rectángulo es × 2 − =
18 cm2
Divide el conjunto de los números naturales desde 1 hasta 100 inclusive en dos
conjuntos A y B tales que A contenga 70 números, B contenga 30 números y la suma de
todos los números de A sea igual a la suma de todos los números de B.
SOLUCIÓN
Usamos la fórmula de la suma de los términos de una progresión aritmética para obtener de forma rápida una
solución.
La suma de los 100 primeros números naturales es
×
= 5050
Por lo tanto, los dos conjuntos deben sumar cada uno = 2525
La suma de los 70 primeros números naturales es
×
= 2485 y, como 2525 − 2485 = 40, podemos
construir el conjunto A tomando todos los primeros 70 números naturales excepto el 31, al que podemos
sustituir por el 71 = 31 + 40 y el conjunto B estará formado por los restantes.
A = {1,2,3,…,28,29,30,32,33,34,…,69,70,71}
B = {31,72,73,74,...,98,99,100}
Nota: evidentemente, ni este reparto es único y ni la forma de distribuir los números es única.
Se quieren colocar cerillas sobre algunos de los segmentos punteados en la
cuadrícula de la figura.
Empezando con la cerilla marcada, se pone cada una a continuación de la
anterior y se termina al conectar la última cerilla con la primera.
En cada celda se escribe el número de cerillas colocadas alrededor de ella.
Algunos de esos números ya están reflejados.
¿Cuál es el mínimo número de cerillas que hay que poner?
SOLUCIÓN
Se trata de hacer el camino más corto con las condiciones impuestas en algunas casillas.
16 cerillas
Sean ABC y ABD dos triángulos unidos por su lado AB.
El triángulo ABC tiene ^BAC=90o
y AB=2AC. El triángulo ABD tiene ^ADB=90o
y
AD=BD.
El segmento CD corta al segmento AB en O.
Calcula BO si se sabe que AC=4.
SOLUCIÓN
Llamamos = , valor buscado.
= 2 = 2 × 4 = 8 y el triángulo es rectángulo e isósceles, por lo que su
altura corta a la hipotenusa en su mitad y forma dos triángulos rectángulos y
también isósceles: y ⇒ = = = = 4
Observemos que, entonces, los triángulos rectángulos y son iguales por el
teorema de Thales ⇒ = = = 2
En conclusión, = = + = 2 + 4 =
6
¿Cuántos números naturales son tales que el producto de sus cifras es 121?
SOLUCIÓN
Las cifras de un número son los números naturales entre 0 y 9 y no es posible que un producto de algunos ellos
(iguales o no) de cómo resultado 11 ni, por supuesto, 11 = 121.
Por lo tanto, la respuesta es
ninguno
¿Para que valores reales de x se verifica que el resultado de la expresión
es un número entero?
SOLUCIÓN
Si < 0 ⇒
| |
=
| |
=
| |
= = −2, valor entero.
Si = 0 ⇒
| |
=
| |
=
| |
= , que no es un valor real.
Si > 0 ⇒
| |
=
| |
=
| |
= = 0, valor entero.
Por tanto, la expresión dada es un número entero
∀ ∈ ∋ ≠
(para todo número real excepto el cero)
¿Cuál es el menor número de elementos que hay que quitar del conjunto
para que el producto de los restantes elementos del conjunto sea un cuadrado perfecto?
SOLUCIÓN
Multiplicando todos, obtenemos = 10 × 20 × 30 × 40 × 50 × 60 × 70 × 80 × 90 ⇒
⇒ = 2 × 5 × 2 × 5 × 2 × 3 × 5 × 2 × 5 × 2 × 5 × 2 × 3 × 5 × 2 × 5 × 7 × 2 × 5 × 2 × 3 × 5 ⇒
⇒ = 2 × 3 × 5 × 7
Para que sea un cuadrado perfecto, todos los factores deben tener un exponente par, por lo que el factor 7
debe desaparecer del producto y también un 2 y un 5 con él para que se quite el número 70 = 2 × 5 × 7
Quedaría entonces = 10 × 20 × 30 × 40 × 50 × 60 × 80 × 90 = 2 × 3 × 5
Ahora sobran un 2 y un 5, por lo que basta eliminar el 10 = 2 × 5 (*)
El producto queda = 20 × 30 × 40 × 50 × 60 × 80 × 90 = 2 × 3 × 5 = 2 × 3 × 5 = 720000
Hay que eliminar, del conjunto,
2 números (10 y 70)
PostData: Razonando de manera similar (*) podrían sobrar tres 2 y un 5 o un 2, dos 3 y un 5, por lo que,
además del 70 se debería eliminar el 40 = 2 × 5 o el 90 = 2 × 3 × 5. Definitivamente, siempre dos números.
Calcula la proporción entre las superficies de la parte amarilla y de la parte naranja de la
figura adjunta.
SOLUCIÓN
Tomamos = 1 como medida de referencia, radio del círculo pequeño y mitad del lado
del cuadrado grande.
El radio del círculo grande , que a su vez es el lado del cuadrado pequeño (amarillo), lo
obtenemos a partir del triángulo rectángulo isósceles aplicando el teorema de
Pitágoras: = + = 1 + 1 = 2 ⇒ = √2
Entonces, las superficies de los dos cuadrados y de los dos círculos son:
Área del cuadrado pequeño (amarillo): Á = √2 = 2
Área del cuadrado grande: Á _ = 2 = 4
Área del círculo pequeño: Á _ = × 1 =
Área del círculo grande: Á _ = × √2 = 2
Calculamos ahora la superficie naranja que será la parte que está dentro del cuadrado grande más la parte que
está fuera de él:
Á = Á _ − Á _ ! +
Á " #$%#_&'Á " #()*_&
= +4 − , +
-'.
= 4 − + − 2 = 2
La proporción pedida es
Á " )/)%$00)
Á " 1)%)12)
= =
1
En una estación de montaña hay hoteles de 1, 2, 3 y 4 estrellas.
Hay más de un hotel de 4 estrellas y sumando el número de estrellas de
todos los hoteles de la estación se obtiene 69.
Más de la mitad de las estrellas corresponden a hoteles de sólo 1
estrella y el número de hoteles de 3 estrellas es 4 veces el de los que
tienen 4 estrellas.
¿Cuántos hoteles de 2 estrellas hay en la estación?
SOLUCIÓN
Llamamos , , , al número respectivo de hoteles de 1, 2, 3, 4 estrellas.
Según el enunciado, observamos que
1. ≥ 2
2. + 2 + 3 + 4 = 69
3. > = 34,5 ⇒ ≥ 35
4. = 4
.
⇒ ≥ 8
Por lo tanto, + 2 + 3 + 4 = 69
.
⇒ + 2 + 3 + = 69 ⇒ 2 + 4 = 69 −
.
⇒ 2 + 4 ≤ 69 − 35 ⇒
⇒ 2 + 4 ≤ 34
÷
+ 2 ≤ 17 ⇒ ≤ 17 − 2
.
⇒ ≤ 17 − 2 × 8 ⇒ ≤ 1
Como hay hoteles de 2 estrellas, la cantidad de ellos es =
1
En un barco pirata hay un cofre con monedas de oro.
Cinco de los piratas reciben su parte con el siguiente procedimiento: primero
Al recibe 1/8 del total y luego Ben recibe 1/6 de lo que queda en el cofre.
Más tarde Carl recibe 1/7 de lo que quedaba después de que les dieran a los
dos primeros, a continuación Dan recibe 1/5 de lo que queda y, finalmente, a
Ed le dan 1/4 de lo que resta.
Si hay tres piratas que recibieron igual cantidad de monedas, determina
quienes son.
SOLUCIÓN
Sea la cantidad inicial de monedas de oro que tiene el cofre.
• Al recibe y, en el cofre, quedan − = monedas.
• Ben recibe × = y, en el cofre, quedan − = − = monedas.
• Carl recibe × = y, en el cofre, quedan − = = monedas.
• Dan recibe × = y, en el cofre, quedan − = = monedas.
• Ed recibe × = y, en el cofre, quedan − = − = monedas.
Se observa inmediatamente que monedas reciben
Al, Dan y Ed
Calcula la proporción entre la superficie azul y la del cuadrado en la figura adjunta.
SOLUCIÓN
Tomamos = 1 como medida de referencia, lado del cuadrado. Entonces, =
Como los triángulos y son semejantes se puede establecer la proporción
= ⇒ = = ⇒ = , luego + = ⇒ + = 1 ⇒ = 1 ⇒ =
La proporción pedida es
Á
Á ! "
=
×
2 =
1 ×
2
3
2
1
=
2
3
2
=
1/3
Se tienen dos bidones, cada uno de ellos con 100 litros de capacidad, e,
inicialmente, contienen entre los dos 100 litros de agua.
Se agrega agua al primer bidón hasta completar su capacidad y luego se vierte
agua del primer bidón al segundo hasta completar la capacidad del segundo.
Finalmente se vierten 12 litros del segundo bidón en el primero.
Así resulta que en el segundo bidón hay 10 litros más de agua que en el
primero.
Determina cuánta agua tenía inicialmente cada bidón.
SOLUCIÓN
Sean , las cantidades, en litros de agua, que tiene cada bidón inicialmente: + = 100
Al segundo bidón le faltan 100 − litros para tener completa su capacidad.
Se llena el primer bidón hasta los 100 litros y se vierten los litros necesarios al segundo bidón para completarlo:
ahora tendrá el segundo bidón 100 litros y el primero litros, ya que ha vertido 100 − litros para completar
el segundo.
Al verter 12 litros del segundo al primero quedarán en el primer bidón + 12 litros y en el segundo bidón
100 − 12 = 88 litros.
La última afirmación del enunciado nos dice que 88 = + 12 + 10 ⇒ + 22 = 88 ⇒ = 66
Además, + = 100 ⇒ = 100 − = 100 − 66 = 34
Inicialmente había
34 litros en el primer bidón
66 litros en el segundo bidón
La bandera de Lancre es un rectángulo con dimensiones en la
proporción 3 : 5.
La bandera se divide en cuatro rectángulos de igual área, como se
muestra.
¿Cuál es la proporción de las longitudes de los lados del rectángulo
blanco?
SOLUCIÓN
Llamamos , a los lados del rectángulo blanco.
3 será la longitud del lado vertical de la bandera y, por la proporción
especificada, 5 es la longitud de su lado horizontal.
Al tener todos los rectángulos la misma superficie se verifica que
3 × 5 − = × ⇒ 15 − 3 = ⇒ 15 = 4 ⇒
⇒ = ⇒ =
4/15
Si
calcula
SOLUCIÓN
Es evidente que + + 1 = 0 ⇒ =
±√
≠ 0 y que + + 1 = 0 ⇒ = −1 −
Entonces = × = × −1 − = − − =⏞ − + 1 + ⇒ = 1
Como 1234 = 411 × 3 + 1 ⇒ = ×
= × = y además + + 1 = 0 ⇒ + 1 = − ,
se concluye que + = + = = =
- 1
Otra forma de resolverlo es usando las dos soluciones complejas =
±√
=
±√
=
− +
√
− −
√
! ⇒
⇒ = "
cos120° + × sen 120°
cos240° + × sen 240°
!
Usando la fórmula de Moivre, = *
cos 120° + × sen 120° = cos 3 × 120° + × sen 3 × 120°
cos 240° + × sen 240° = cos 3 × 240° + × sen 3 × 240°
! ⇒
⇒ = *
cos 3 × 120° + × sen 3 × 120° = cos 360° + × sen 360°
cos 3 × 240° + × sen 3 × 240° = cos 720° + × sen 720°
!
- .° × /.°
0111111112 = 1 + 0 × = 1 y, a
partir de aquí, seguir como en la resolución anterior.
Albert y Bernard son compañeros de trabajo de Cheryl y quieren saber cuándo es su
cumpleaños.
Cheryl les da una lista de diez posibles fechas: 15 de mayo, 16 de mayo, 19 de mayo,
17 de junio, 18 de junio, 14 de julio, 16 de julio, 14 de agosto, 15 de agosto, 17 de
agosto.
Después, Cheryl les dice por separado a Albert y Bernard, respectivamente, el mes y
el día de su cumpleaños.
Entonces Albert señala: "No se cuándo es el cumpleaños de Cheryl, pero se que
Bernard tampoco lo sabe", a lo que Bernard responde: "Al principio no sabía cuándo era el cumpleaños de
Cheryl, pero ahora ya lo se".
Albert reflexiona y concluye: "Entonces yo también se cuándo es su cumpleaños".
¿Cuándo es el cumpleaños de Cheryl?
SOLUCIÓN
Como Albert conoce el mes de cumpleaños y determina que Bernard no puede saber la fecha exacta de
nacimiento es imposible que el cumpleaños sea un 18 o un 19, días únicos que propone Cheryl, por lo que a
Albert no le ha dicho que su cumpleaños sea en mayo ni en junio.
El cumpleaños solo puede ser, entonces, en julio o en agosto: 14 de julio, 16 de julio, 14 de agosto, 15 de agosto
o 17 de agosto.
Bernard contesta que antes no lo sabía pero ahora sabe cuándo es el cumpleaños, conociendo solo el día. Esto
descarta el día 14 porque hay dos posibles: quedan los días 16 de julio, 15 de agosto y 17 de agosto.
Finalmente Albert sabe definitivamente, después de todas las declaraciones, cuándo es el cumpleaños sabiendo
el mes por lo que debe descartar duplicidades sabiendo el mes de cumpleaños (: ¡hay dos días posibles de
agosto!) y determina que el día de cumpleaños de Cheryl es el
16 de julio
Sea ABCD un cuadrado de papel de lados AB = BC = CD = DA =10
El cuadrado se dobla a lo largo de una línea recta, haciendo coincidir el vértice A
con el punto medio del lado BC
Esta línea recta corta al lado AB en E y al lado CD en F
Calcula la medida de EF
SOLUCIÓN
Según el esquema de la figura el ángulo es tal que tan = = =
Por construcción, = 90° − ⇒ cot = tan ⇒ cot =
Como sen = !" #
⇒ sen = !" $%
= &
'
=
(
⇒ sen =
√
En el triángulo rectángulo * , sen * = sen ⇒
+%
$%
=
√
⇒ $%
=
√
⇒
⇒ =
×√
=
5×√5
Un cuadrado tiene dos de sus vértices en una semicircunferencia y los otros
dos en el diámetro de la misma, como se muestra en la figura.
El radio de la circunferencia es de 1 cm.
¿Cuál es el área del cuadrado?
SOLUCIÓN
Llamamos a la mitad del lado del cuadrado.
Según el triángulo rectángulo que se construye y aplicando en él el teorema de
Pitágoras, + 2 = 1 ⇒ + 4 = 1 ⇒ =
El área del cuadrado es 2 = 4 = 4 × =
4/5 cm2
Sean ABC un triángulo isósceles con AB = AC y D el punto medio del lado BC.
La perpendicular a AC, trazada por D, corta al lado AC en E.
Sea F en AB tal que EF es paralela a BC.
Si BC =12 y CE = 4, calcula la medida del segmento EF.
SOLUCIÓN
Fijándonos en el esquema de la derecha, tenemos, por construcción, el triángulo
rectángulo donde la hipotenusa es = = = 6 y el cateto = 4
Por el teorema del cateto, × = ⇒ = = = =
De lo anterior = − = 6 − = ⇒ = 2 × = 2 × =
20/3
Un gráfico consta de 16 vértices y algunos segmentos que los conectan, como en la
imagen.
Hay un gusano en el vértice A y, en cada movimiento, puede moverse desde un
vértice a cualquier vértice vecino a lo largo de un segmento de conexión.
¿En cuál o cuales de los vértices P, Q, R, S, T puede estar el gusano después de 2019
movimientos?
SOLUCIÓN
El gusano alcanza, desde , los vértices , , en una cantidad par de movimientos y los vértices , en una
cantidad impar. Siempre.
Por tanto, después de 2019 movimientos el gusano puede estar en uno de estos vértices:
Q – S
Una librería ofrece cuadernos a 5 euros y realiza los siguientes
descuentos:
• en una compra de hasta 35 cuadernos inclusive hace un
descuento del 5%
• si se compran entre 36 y 55 cuadernos inclusive el descuento
es del 12%
• si se compran 56 o más cuadernos, el descuento es del 20%
Pablo compró con un descuento del 5% y al día siguiente compró
nuevamente, esta vez con un descuento del 12%.
Si Pablo hubiese comprado los cuadernos todos juntos, en una sola compra, le hubiese correspondido un
descuento del 20% y habría gastado 39 euros menos de lo que gastó.
Determina cuántos cuadernos compró cada día.
SOLUCIÓN
Sean , la cantidad de cuadernos que compró, respectivamente, el primer y el segundo día.
Es evidente que 1 ≤ ≤ 35; 36 ≤ ≤ 55; 56 ≤ + ⇒ 56 ≤ + ≤ 90
Según el enunciado, 5 × + 5 × = 5 × + × + 39
×
95 + 88 = 80 + 80 + 780 ⇒
⇒ 15 + 8 = 780 ⇒ = ⇒ = 52 − , con 36 ≤ ≤ 55
Como = 52 − es un valor natural ⇒ es múltiplo de 15 ⇒ = 45 ⇒ = 52 −
×#
= 52 − 24 ⇒ = 28
El primer día compró 28 cuadernos
El segundo día compró 45 cuadernos
Los números naturales a, b y c tienen cada uno tres cifras, y en cada número la primera
cifra es la misma que la última.
Si b=2a+1 y c=2b+1, ¿cuántos valores distintos hay para a?
SOLUCIÓN
Sea = ⇒ = 100 + 10 + = 101 + 10 . Se deduce entonces que
• = 2 + 1 = 2 × 101 + 10 + 1 = 202 + 20 + 1
• = 2 + 1 = 2 × 202 + 20 + 1 + 1 = 404 + 40 + 3
Como es de tres cifras, las posibilidades son:
• = 1 ⇒ = 404 + 40 + 3 = 40 + 403 ⇒
⇒
= 8 ⇒ = 40 × 8 + 403 ⇒ = 727 ⇒ = 363 ⇒ = 181
= 9 ⇒ = 40 × 9 + 403 ⇒ = 767 ⇒ = 383 ⇒ = 191
• = 2 ⇒ = 808 + 40 + 3 = 40 + 811, y es imposible que sea capicúa de tres cifras.
Hay, para ,
dos valores distintos: 181, 191
Sea N=8+98+998+9998+…..+999…(100 veces 9)…98 el resultado de la suma de 101 números que
tienen el último dígito 8 y los demás dígitos 9: desde el 8, que tiene cero nueves, hasta el que
tiene 100 dígitos nueve.
Calcula la suma de los dígitos de N.
SOLUCIÓN
= 8 + 98 + 998 + +9998 + ⋯ + 99999. . .9999 8 = 10 − 2 + 10 − 2 + + 10 − 2 + ⋯ + 10 − 2
Entonces, = 10 + 10 + 10 + ⋯ + 10 − 2 × 101 =
× !
!
− 202, usando la fórmula "# = $× !
!
de la suma de los % primeros términos de una progresión geométrica de razón &.
=
× !
!
− 202 =
×' ! (
− 202 =
× ...
)*+,-. /
− 202 = 11111. . .111 0 − 202 ⇒
= 11111. . .111
1
0908, por lo que la suma de sus cifras es 1 + 1 + 1+. . . +1 + 1 + 1
1
+ 0 + 9 + 0 + 8 =
98 × 1 + 9 + 8 =
115
Si se elimina cualquier cifra de un número de 4 cifras dado, el número de 3 cifras resultante
es un divisor del número original.
¿Cuántos números de 4 cifras tienen esta propiedad?
SOLUCIÓN
Sea el número de cuatro cifras = tal que, según se deduce del enunciado, ≠ 0, ≠ 0, y sea el número
de tres cifras
Entonces, ∃ ∈ ∋ × = ⇒ × = 10 × + ⇒ − 10 × =
!"
#$$$$$$$$$$$$% = 0
De lo anterior = 0, y sea el número de tres cifras = 0 = 10 ×
También, ∃ℎ ∈ ∋ ℎ × 0 = ⇒ ℎ × 10 × = 100 × + 10 ⇒ ℎ × = 10 × + ⇒
⇒ ℎ − 10 × =
'
!"
#$$$$$$$$$$% = 0
Por lo tanto = 00 y, por fin, tomando los números 00 = 100 y 00 = 100 se sigue por hipótesis que
• ∃( ∈ ∋ ( × 00 = ⇒ ( × 100 = 1000 + 100 ⇒ ( = 10 + ⇒ ( − 10 × =
• ∃) ∈ ∋ ) × 00 = ⇒ ) × 100 = 1000 + 100 ⇒ ) = 10 + ⇒ ) − 1 × = 10
De la combinación de las igualdades se deduce que ) − 1 × ( − 10 × = 10 ⇒ ) − 1 × ( − 10 = 10
Posibilidades:
• ) − 1 = 1 ⇒ = 10 . No tiene sentido por ser cifras y
• ) − 1 = 2 ⇒ 2 = 10 ⇒ = 1; = 5 … … … … … … … … … … . … . . = 1500
• ) − 1 = 5 ⇒ 5 = 10 ⇒ /
= 1; = 2 … … … … … … … … … … … . = 1200
= 2; = 4 … … … … … … … … … … … . = 2400
= 3; = 6 … … … … … … … … … … … . = 3600
= 4; = 8 … … … … … … … … … … … . = 4800
4
• ) − 1 = 10 ⇒ 10 = 10 ⇒ = ⇒
⎩
⎪
⎪
⎪
⎨
⎪
⎪
⎪
⎧
= 1; = 1 … … … … … … = 1100
= 2; = 2 … … … … … … = 2200
= 3; = 3 … … … … … … = 3300
= 4; = 4 … … … … … … = 4400
= 5; = 5 … … … … … … = 5500
= 6; = 6 … … … … … … = 6600
= 7; = 7 … … … … … … = 7700
= 8; = 8 … … … … … … = 8800
= 9; = 9 … … … … … … = 9900
4
En total,
14 números
Sea ABC un triángulo tal que ^A=45o
y ^C=30o
y sea D el punto medio del lado BC.
Calcula la medida del ángulo ^CAD.
SOLUCIÓN
Buscamos el ángulo = sabiendo que = 45° y = 30° ⇒ = 180° − − ⇒
⇒ = 105° y, además, = =
Aplicamos el teorema de los senos a los triángulos y :
=
° !
=
"
#
$ª
&ª
⇒
° !
=
"
⇒
°×() () °×
=
*+ °
,+°
⇒
⇒
√.
.
× cot − 1! =
°23+°!
,+°
=
°×() 3+°2() °× 3+°
,+°
=
√.
.
×
&
$
2
√4
$
&
$
⇒ cot − 1 = 1 + √3 ⇒
⇒ cot = 2 + √3 ⇒ tg =
*
.2√,
=
. √,
8.2√,9×8. √,9
⇒ tg = 2 − √3 ⇒ = arctg 82 − √39 =
15o
Un contenedor que tiene forma de caja con caras rectangulares está
parcialmente lleno con 120 m3
de agua.
La profundidad del agua es de 2 m, 3 m ó 5 m, según la cara de la caja
que se encuentre en el suelo, como se muestra en la figura.
¿Cuál es el volumen del contenedor?
SOLUCIÓN
Sean , , las longitudes de los lados de la caja correspondientes a las medidas parciales de 2, 3, 5 m
El volumen de la caja es y el enunciado indica que
2 = 120
3 = 120
5 = 120
ª× ª× ª
2 × 3 × 5 = 120 × 120 × 120 ⇒ 30 = 120 ⇒ = ⇒
⇒ = = 120 × = 120 × √4 = 120 × 2 =
240 m3
Sobre la mesa hay 21 cartas, una con cada uno de los números naturales de 1 hasta
21 ambos inclusive.
Miguel selecciona 4 cartas y se las muestra a Marga. Luego Marga le quita a Miguel
una carta, la que ella quiera.
Si la suma de los números de las 3 cartas con las que se quedó Miguel es múltiplo de
3, gana Marga y si no, gana Miguel.
Determina de cuántas maneras puede Miguel elegir las 4 cartas para estar seguro de
ganar, no importando lo bien que juegue Marga y teniendo en cuenta que dos
elecciones de las mismas 4 cartas, pero en distinto orden, se consideran la misma
elección.
SOLUCIÓN
Los números de las cuatro cartas deben ser tales que, sumados tres a tres, el resultado no sea múltiplo de 3
Si ordenamos los números según el resto que se obtiene al dividirlos por 3 obtenemos esta clasificación:
• 3 , que son los números 3, 6, 9, 12, 15, 18, 21. Total, 7 números
• 3 + 1, que son los números 1, 4, 7, 10, 13, 16, 19. Total, 7 números
• 3 + 2, que son los números 2, 5, 8, 11, 14, 17, 20. Total, 7 números
En el conjunto de las 4 cartas no puede haber 3 o más cartas pertenecientes a al mismo grupo de los
anteriormente citados pues desechando una de ellas (en caso de cartas de dos grupos, la de distinto grupo a las
otras 3) las otras suman un múltiplo de 3
En el conjunto de las 4 cartas no pueden estar representados los tres grupos, pues le bastaría a Marga dejar una
de cada grupo para que su suma fuese múltiplo de 3: 3 + 3 + 1 + 3 + 2 = 3 × + + + 1
En resumen, las posibilidades de una victoria segura de Miguel es que seleccione 2 de un grupo y 2 de otro:
cualquier carta que quite Marga dejará una suma de dos cartas de un grupo y una tercera de otro grupo cuya
suma es imposible que sea múltiplo de 3.
Es cuestión, entonces, de formar conjuntos de dos cartas de cada grupo de 7 y, dichos conjuntos, combinarlos
de dos en dos, con cada uno de cada uno de los 3 grupos.
Las maneras de elección de las cuatro cartas para una victoria segura de Miguel son, entonces,
× × =
7!
2! × 5!
×
7!
2! × 5!
×
3!
2! × 1!
=
7 × 6
2
×
7 × 6
2
× 3 =
1323
Tres amigos, Alex, Guille y Marc, salen a caminar todos los días.
Si Alex no lleva sombrero, entonces Guille sí.
Si Guille no lleva sombrero, entonces Marc sí.
Hoy Marc no lleva sombrero.
¿Quién lleva sombrero hoy con toda seguridad?
SOLUCIÓN
La afirmación " ⇒ í " [ ] equivale a la afirmación
" ⇒ í " [ ′]
La afirmación " ⇒ í " [ ] equivale a la afirmación
" ⇒ í " [ ′]
Entonces, como hoy ⇒ í , y no puede deducirse ninguna
conclusión más.
Hoy lleva sombrero
Guille
De un cuadrado de papel de lado 1 hay que recortar dos triángulos equiláteros iguales.
Halla el máximo valor posible del lado de los triángulos.
SOLUCIÓN
Sea la medida del lado de los triángulos equiláteros y revisamos las dos maneras de
conseguir los mayores triángulos equiláteros.
Según se aprecia en la figura de la derecha, esta es una forma de elegir los dos
triángulos equiláteros iguales de mayor lado dentro del cuadrado de lado 1.
Observamos, en el triángulo rectángulo que = 1, = − 1 − = 2 − 1
y = 60° ⇒ tg = ⇒ tg 60° = ⇒ = √3 ⇒ 2 − 1 =
√
=
√
⇒
= × 1 +
√
! =
"√
#
= 0,788675
Otra forma de elegir los dos triángulos equiláteros iguales de mayor lado dentro del
cuadrado de lado 1 es la que se observa en la figura de la derecha.
En este caso, aplicando el teorema de Pitágoras en el triángulo rectángulo que es la
mitad del equilátero, la altura del triángulo equilátero es ℎ = ( − ! = (
)
*
⇒
⇒ ℎ =
×√
Ahora bien, ℎ es la mitad de la diagonal del cuadrado: ℎ =
√ )" )
=
√
por lo que
×√
=
√
⇒ =
√
√
⇒
⇒ =
√#
= 0,816497
En resumen, mayor valor posible del lado corresponde a la segunda opción y es
√-
.
= 0,816497
Un número natural n se llama 6-bueno si su divisor más grande, excluyendo n,
es igual a n–6.
¿Cuántos números 6-buenos hay?
SOLUCIÓN
− 6 el divisor más grande del número natural . Se cumple entonces que = − 6 × , siendo el número
natural ≤ − 6.
De la expresión anterior, = ⇒ = 1 +
Posibilidades:
• − 6 = 1 ⇒ = 7 y = 7 … 7 = 1 × 7. Como se excluye contándolo como divisor para este
problema, = 7 cumple la condición.
• − 6 = 2 ⇒ = 8 y = 4 … 8 = 2 × 4. Existe un divisor de = 8 mayor que − 6: > − 6.
• − 6 = 3 ⇒ = 9 y = 3 … 9 = 3 × 3, por lo que = 9 cumple la condición.
• − 6 = 6 ⇒ = 12 y = 2 … 12 = 6 × 2, por lo que = 12 cumple la condición.
Los números 6-buenos son 7, 9, 12, luego son
3
La figura muestra un cubo desplegado.
Escribe en cada cara del cubo un número entero del 1 al 6 sin repetir
sabiendo que, al armar nuevamente el cubo, si para cada vértice se
calcula la multiplicación de los números de las tres caras que
concurren en ese vértice, se obtienen, en algún orden, los números
10, 12, 20, 24, 30, 36, 60 y 72 y teniendo en cuenta que el 1 ya está
colocado.
SOLUCIÓN
Según los números obtenidos en los vértices, las únicas
descomposiciones posibles en productos de 3 dígitos de 1 a 6 de los
valores extremos son 10 = 1 × 2 × 5 y 72 = 3 × 4 × 6, por lo que
los números de ambos productos estarán en caras opuestas dos a
dos.
En principio, podemos considerar que 1, 2, 5 confluyen en un vértice,
como se ve en la figura de la derecha.
Está claro que la única cara que combinará también con las 1 y 2
deberá ser la 6 para construir el 12 = 1 × 2 × 6, por lo que ésta
última cara se opondrá a la 5.
Además, la otra cara que combinará con las 1 y 5 deberá ser la 4 para construir el 20 = 1 × 4 × 5, por lo que
ésta última cara se opondrá a la 2. La cara restante es la 3, que se opondrá a la cara 1, quedando así el
desarrollo del cubo:
Una caja contiene 4 bombones y 1 caramelo.
Diego y Ángel sacan por turnos una golosina de la caja y se la comen. Quien saque el
caramelo gana.
Si Diego extrae primero, ¿cuál es la probabilidad de que gane Ángel?
SOLUCIÓN
Ángel únicamente puede ganar al efectuar la segunda o la cuarta extracción.
La probabilidad de que Ángel saque el caramelo en la segunda extracción se calcula teniendo en cuenta que en
la primera extracción Diego no saca el caramelo: = × =
La probabilidad de que Ángel saque el caramelo en la cuarta extracción se calcula teniendo en cuenta que en la
primera y tercera extracciones Diego no saca el caramelo ni Ángel en la segunda: = × × × =
Por tanto, la probabilidad pedida es + = + =
2/5
Jon hizo la lista de los números enteros positivos de cuatro dígitos ABBC que son múltiplos de 9
y tales que A, B, C son dígitos distintos con B=A+C.
Calcula cuántos números tiene la lista de Jon.
SOLUCIÓN
= 9 ⇒ + + + = + 2 + = 9 3 = 9 ⇒ = 3 , siendo un número natural
porque = + ≠ 0 ya que , , son cifras distintas entre sí.
Como es una cifra y 0 ≤ , < , las posibilidades son:
• = 1 ⇒ = 3 ⇒
= 1, = 2 … … … 1332
= 2, = 1 … … … 2331
• = 2 ⇒ = 6 ⇒
= 1, = 5 … … … 1665
= 2, = 4 … … … 2664
= 4, = 2 … … … 4662
= 5, = 1 … … … 5661
• = 3 ⇒ = 9 ⇒
⎩
⎪
⎪
⎨
⎪
⎪
⎧
= 1, = 8 … … … 1998
= 2, = 7 … … … 2997
= 3, = 6 … … … 3996
= 4, = 5 … … … 4995
= 5, = 4 … … … 5994
= 6, = 3 … … … 6993
= 7, = 2 … … … 7992
= 8, = 1 … … … 8991
Total, 2 + 4 + 8 =
14 números
Se muestran dos cuadrados adyacentes con longitudes de lados a y b (a<b).
¿Cuál es el área del triángulo sombreado?
SOLUCIÓN
Viendo el esquema representado en la figura la superficie pedida es igual a
= Á − Á − Á − Á =
×
2
−
×
2
−
×
2
− × =
+ ×
2
−
×
2
−
× −
2
− × − =
+ − − + − 2 + 2
2
=
a2
/2 unidades cuadradas
En una reunión cada invitado saludó a cada uno de los restantes con un apretón de
manos.
Hubo 36 apretones de manos entre dos mujeres y 28 apretones de manos entre dos
varones.
Calcula cuántos apretones de manos hubo entre un varón y una mujer.
SOLUCIÓN
Llamamos ℎ al número de hombres de la reunión. Los apretones de mano entre ellos fueron , =
!
!× !
⇒
⇒
×
= 28 ⇒ ℎ − ℎ − 56 = 0 ⇒ ℎ = 8: hay 8 hombres.
Llamamos al número de mujeres de la reunión. Los apretones de mano entre ellas fueron , =
!
!× !
⇒
⇒
×
= 36 ⇒ − − 72 = 0 ⇒ = 9: hay 9 mujeres.
Los apretones de manos entre hombres y mujeres fueron 8 × 9 =
72
Para calcular (a+b)/c, Laura teclea en la calculadora a+b/c y el resultado es 11,
siendo a, b y c enteros positivos.
Luego teclea b+a/c y se sorprende al ver que el resultado es 14.
Se da cuenta de que la calculadora está diseñada para calcular las divisiones antes
que las sumas.
¿Cuál es el resultado correcto de (a+b)/c ?
SOLUCIÓN
+ = 11 ⇒ = 11 −
+ = 14 ⇒ = 14 −
ª ª
= 25 − + ⇒ + = 25 − × + ⇒ + =
Como , , son enteros positivos, de la expresión anterior se deduce que
• + 1 = 5 ⇒ = 4 ⇒
+ = 11 ⇒ + = 11
+ = = 20
ª ª + = 11
!
= 9
⇒ #
= 8
= 12
• + 1 = 25 ⇒ = 24 ⇒ % + = 11 ⇒ + = 11
+ = 24
ª ª + = 11
!
= 13
, imposible porque no sería
un valor entero.
En resumen, = 8, = 12, = 4 ⇒ =
'
=
5
El número n tiene 100 cifras, todas iguales a 9.
Calcula la suma de las cifras de n2
SOLUCIÓN
= 999 … … .9 = 10 − 1 ⇒ = 10 − 1 = 100 − 2 × 10 + 1 ⇒
⇒ = 1 000 … 0 − 2 000 … 0 + 1 = 999 … 9 8 000 … 0 + 1 ⇒ = 999 … 9 8 000 … 0 1
luego la suma de sus cifras es 9 + 9 + 9 + ⋯ + 9 + 8 + 0 + 0 + 0 + ⋯ + 0 + 1 = 99 × 9 + 8 + 1 =
900
¿Para qué valores enteros n el número
es primo?
SOLUCIÓN
Si − 2 − 3 = 0 ⇒ =
± ×
=
±
=
−1
3
⇒ − 2 − 3 = + 1 × − 3
La expresión será un número primo si uno de los dos factores anteriores es igual a ±1 y el otro factores es, en
valor absoluto, primo.
Posibilidades:
• + 1 = −1 ⇒
= −2
− 3 = −5
⇒ | − 2 − 3| = | −1 × −5 | = |5| = 5, número primo
• + 1 = 1 ⇒
= 0
− 3 = −3
⇒ | − 2 − 3| = |1 × −3 | = |−3| = 3, número primo
• − 3 = −1 ⇒
= 2
+ 1 = 3
⇒ | − 2 − 3| = |3 × −1 | = |−3| = 3, número primo
• − 3 = 1 ⇒
= 4
+ 1 = 5
⇒ | − 2 − 3| = |5 × 1| = |5| = 5, número primo
En todos los casos se obtienen números primos, por lo que los valores pedidos son cuatro:
-2, 0, 2, 4
Sea ABCD un rectángulo con BC < CD y M, N los puntos medios de los
lados BC y CD respectivamente.
Este rectángulo es tal que el triángulo AMN es rectángulo con ^MNA = 90o
.
Si BC = 5, calcula la medida de CD.
SOLUCIÓN
Llamamos = ⇒ = = . Como = = 5 ⇒ =
Observamos que = 90° − = 90° − 180° − 90° − ⇒
= y, por tanto los triángulos rectángulos y son
semejantes.
Establecemos, entre ellos, la relación de semejanza = ⇒ = ⇒
⇒ ! = 5 × ⇒ #
= ⇒ = 50 ⇒ = √50 =
5×√2 ≈ 7,07
Una poligonal DEFB, con DE ⊥ ⊥EF y EF FB, se encuentra dentro del cuadrado ABCD
como se muestra en la figura.
Como DE = 5, EF = 1 y FB = 2, ¿cuál es la longitud del lado del cuadrado?
SOLUCIÓN
Trazamos la diagonal al cuadrado , que corta al segmento en
Llamamos = ⇒ 1 − =
Como los segmentos y son paralelos ⇒ = , por lo que los triángulos
rectángulos y son semejantes.
Establecemos, entre ellos, la razón de semejanza = ⇒ = ⇒ 5 = 2 − 2 ⇒
⇒ 7 = 2 ⇒ = y 1 − =
Por lo tanto, la diagonal del cuadrado es = + = √ + + √ + ⇒
⇒ = 2 + + 5 + = 2 × 1 + ! + 5 × 1 + ! = 2 ×
"
! + 5 ×
"
! ⇒
⇒ = 7 ×
"
! = × √50 ⇒ = √50, habiendo aplicado el teorema de Pitágoras en los triángulos
rectángulos citados.
Volviendo a aplicar el mismo teorema en el triángulo rectángulo $ , $ + $ =
%&%
'((() 2 × $ = 50,
de donde ⇒ $ = 25 ⇒ $ =
5
Un biólogo que estudia una colonia de aves migratorias hizo las siguientes observaciones
a lo largo de un día:
• A mediodía se fueron 30 machos que ya no regresaron, y quedaron en la colonia
2 hembras por cada macho.
• A la tarde se fueron 90 hembras, que ya no regresaron, y quedaron en la colonia
3 machos por cada hembra.
Determina cuántas aves tenía la colonia antes del mediodía.
SOLUCIÓN
Sean , el número respectivo de aves macho y aves hembra que tiene la colonia antes del mediodía.
Según las dos afirmaciones del enunciado, se cumple que
2 × − 30 =
− 30 = 3 × − 90
⇒
2 − 60 =
− 30 = 3 − 270
⇒
⇒
2 − = 60
3 − = 240
ª × ª 2 − = 60
5 = 540
⇒
=
=
⇒
= 84
= 108
⇒ + = 84 + 108 =
192 aves
Se seleccionan al azar tres números diferentes del conjunto {1, 2, 3, ..., 10}.
¿Cuál es la probabilidad de que uno de ellos sea la media aritmética de los otros dos?
SOLUCIÓN
Para que uno de los elegidos sea la media aritmética de los otros dos, estos deben ser de la misma paridad.
La probabilidad de que se elijan dos números, de los diez, de la misma paridad es 1 × = y que el tercero sea
la medía aritmética de anteriores (que siempre se encontrará entre los diez y será distinto de los dos primeros)
es × =
Como la media artimética puede ser, también, el primero o el segundo de los números elegidos, la probabilidad
pedida es 3 × =
1/6
Si se agregan a la derecha de 2019 tres dígitos a, b, c, el número de siete dígitos
2019abc es divisible por 541.
Halla todos los posibles valores de los dígitos a, b, c.
SOLUCIÓN
2019 = 2019000 + = 541
Como 2019000 = 3731 × 541 + 529 ⇒ 3731 × 541 + 529 + = 541 ⇒ 529 + = 541
Entonces, las posibilidades son:
• 529 + = 541 × 1 = 541 ⇒ = 541 − 529 = 12 ⇒ = 0, = 1, = 2
• 529 + = 541 × 2 = 1082 ⇒ = 1082 − 529 = 553 ⇒ = 5, = 5, = 3
• 529 + = 541 × 3 = 1623 ⇒ = 1623 − 529 = 1094, imposible.
Los posibles valores son
a = 0, b = 1, c = 2
a = 5, b = 5, c = 3
La sucesión a1, a2, a3, … comienza con a1=49
Para an+1, el término se obtiene añadiendo 1 a la suma de las cifras de an y elevando al
cuadrado el resultado.
Por ejemplo, a2=(4+9+1)2
=196
Calcula el valor de a2019
SOLUCIÓN
= 49 = 4 + 9 + 1 = 14 = 196
= 1 + 9 + 6 + 1 = 17 = 289 = 2 + 8 + 9 + 1 = 20 = 400
= 4 + 0 + 0 + 1 = 5 = 25 = 2 + 5 + 1 = 8 = 64 = 6 + 4 + 1 = 11 = 121
= 1 + 2 + 1 + 1 = 5 = 25 = 2 + 5 + 1 = 8 = 64 = 6 + 4 + 1 = 11 = 121
………… ………… ………… ………… ………… ………… …………
Como puede observarse, a partir del quinto término de la sucesión los valores se repiten cada tres términos, de
manera que = 25; = 64; = 121, ∀ ∈ !
Como 2019 = 3 × 673 = 3 × 672 + 3 ⇒ =
64
Paco y Joaquín se reparten una bolsa de caramelos con el siguiente
procedimiento: Paco saca uno, Joaquín saca dos, Paco saca tres, Joaquín
saca cuatro, y así cada uno, en su turno, saca uno más que los que acaba
de sacar el otro.
Cuando uno de los dos se encuentra con que no quedan suficientes
caramelos para sacar uno más que los que sacó el otro se lleva todo lo que
queda y concluye el reparto.
Si Paco sacó en total 2000 caramelos, ¿cuántos caramelos había inicialmente en la bolsa?
SOLUCIÓN
Es evidente que en la tanda número , Paco se lleva 2 − 1 caramelos y Joaquín 2 caramelos.
Si la última llevada (los restos) es de Joaquín, Paco se llevaría tandas completas de caramelos, por lo que el
total sería 1 + 3 + 5 + 7 + ⋯ + 2 − 1 =
×
= = 2000, lo cual es imposible pues es una
cantidad natural.
Por lo tanto, Paco es el que se lleva la última cantidad < 2 + 2 una vez que ha recibido tandas completas.
Es decir, + = 2000 y es el número cuyo cuadrado es más proximo a 2000 y menor que ese valor.
Como 44 < √2000 < 45 ⇒ = 44
Paco y Joaquín reciben 44 tandas completas de caramelos y el resto son = 2000 − = 2000 − 44 = 64
caramelos recibidos por Joaquín al inicio de la tanda 45.
El total de caramelos a repartir es 1 + 3 + 5 + ⋯ + 2 − 1 + 2 + 4 + 6 + ⋯ + 2 + 64 =
= 1 + 2 + 3 + ⋯ + 2 + 64 =⏞ 1 + 2 + 3 + ⋯ + 88 + 64 =
×
+ 64 = 89 × 44 + 64 =
3980 caramelos
(Paco se lleva 2000 y Joaquín 1980)
El cuadrado de la figura se rellena con los números 1, 2, 3, 4 y 5 de tal manera que cada fila y
cada columna contienen cada uno de ellos exactamente una vez.
Además, la suma de los números en cada una de las tres regiones con bordes en negrita es
igual.
¿Qué número está en la esquina superior derecha?
SOLUCIÓN
Es un cuadro 5 × 5 que tiene, en cada fila o columna, los números de 1 a 5, por lo que la suma de todos ellos es
1 + 2 + 3 + 4 + 5 × 5 = 15 × 5 = 75.
La zona inferior derecha tiene ya colocado un 2: las cinco casillas restantes sumarán 23. Como
no puede haber más de tres números iguales entre ellos (debido a la cantidad de columnas y/o
filas vacías), la única distribución posible será la que se ve en el margen derecho.
En el espacio central la suma de las casillas también es 25 y las casillas vacías
deben contener los valores que se ven en la imagen de la izquierda, al estar
colocados los restantes en las respectivas filas y/o columnas.
Las tres casillas que quedan suman 25 − 2 × 1 + 3 − 2 × 1 + 2 + 3 = 5,
por lo que dichas casillas deben tomar los valores 1, 2, 2, quedndo como se ve
en la imagen derecha.
En resumen, y mediante una deducción elemental se obtiene la distribución de valores en las casillas del espacio
central intermedio:
A partir de aquí, y con las condiciones establecidas, es muy sencillo completar el cuadro hasta llegar a
que permite decir que el número pedido es
3
Sea ABCD un trapecio de bases AB y CD , y lados no paralelos BC y DA , con
^BAD=^ADC=90o
.
La perpendicular a la diagonal AC trazada desde B corta a AC en E.
Si AB=125, AE=35 y CE=50, calcular el área del trapecio ABCD.
SOLUCIÓN
Señalamos en la imagen los valores y alguna línea y punto auxiliar.
Sea = = ℎ y = = ⇒ = 125 −
Además, = + = 35 + 50 = 85
En el triángulo rectángulo aplicamos el teorema de Pitágoras:
= − = 125 − 35 = 14400
Ahora aplicamos el mismo teorema en el triángulo rectángulo :
= + = 14400 + 50 = 16900
Calculamos y ℎ usando el teorema de Pitágoras en los triángulos rectángulos y :
+ =
+ =
⇒
ℎ + = 85
ℎ + 125 − = 16900
⇒ ℎ + = 7225
ℎ + 15625 − 250 + = 16900
⇒
⇒ ℎ + = 7225
15625 − 250 + 7225 = 16900
⇒ ℎ + = 7225
250 = 5950
⇒
ℎ = 7225 − = 6658,56 ⇒ ℎ = 81,6
=
!
!
= 23,8
La superficie del trapecio es
"#$%&
× =
( $)
× ℎ =
( $ *,+
× 81,6 =
6071,04 unidades cuadradas
Dos triángulos isósceles cuyos lados miden x, x, 6 y x, x, 8, respectivamente,
tienen igual área.
Halla x.
SOLUCIÓN
Llamando , a las respectivas alturas de ambos triángulos isósceles, se
verifica, por el teorema de Pitágoras, que
⎩
⎨
⎧ = −
= −
⇒ = √ − 9
= √ − 16
á
!!!!"
×√$%&'
=
×√$%&(
⇒
⇒ 3 × √ − 9 = 4 × √ − 16 ⇒ 9 × + − 9, = 16 × + − 16, ⇒ 9 − 81 = 16 − 256 ⇒ 7 = 175 ⇒
⇒ =
(12
1
= 25 ⇒ =
5
Ángela tiene un tablero de 115×7, o sea, de 115 filas con 7 casillas cada una y debe
colocar fichas en las casillas del tablero siguiendo las siguientes reglas:
• En cada casilla puede colocar una sola ficha.
• No pueden quedar dos filas idénticas, es decir, no puede haber dos filas que
tengan las mismas casillas ocupadas y las mismas casillas vacías.
Calcula la máxima cantidad de fichas que puede colocar Ángela en su tablero.
SOLUCIÓN
Representamos como a la casilla rellenada con una ficha y como a la casilla vacía.
La cantidad de posibilidades distintas de rellenado de filas es el número de variaciones con repetición de dos
elementos ( y ) tomados de 7 en 7: , , … serían algunas de estas variaciones.
La cantidad de filas posibles es , = 2 = 128 con, por simetría,
×
= 448 casillas y 448 casillas .
Por tanto, hay 128 − 115 = 13 filas que no aparecerán, y deberán ser las que contengan menos casillas .
Las describimos:
• la fila , sin casillas .
• las filas con una y seis : ,
=
!
!× !
= 7 filas, que contienen 7 × 1 = 7 casillas en total.
• 13 − 1 − 7 = 5 de las filas con dos y cinco que serían, en total, ,
=
!
!× !
= 21. Las cinco filas
de este tipo contienen 5 × 2 = 10 casillas en total.
La mínima cantidad de casillas que no estarían en el tablero, de todas las variaciones posibles, sería
0 + 7 + 10 = 17 por lo que la cantidad máxima de casillas que puede haber es 448 − 17 =
431
Se considera un número n de cuatro cifras, cuadrado perfecto, tal que todas sus cifras son
menores que 6.
Si a cada cifra se le suma 1, el número resultante es otro cuadrado perfecto.
Halla n.
SOLUCIÓN
Sea = = ⇒ 1000 + 100 + 10 + =
Además, 1000 × + 1 + 100 × + 1 + 10 × + 1 + + 1 = ⇒ − = 1000 + 100 + 10 + 1
Es decir, − = 1111 = 101 × 11 ⇒ + × − = 101 × 11 ⇒
+ = 101
− = 11
, única distribución
posible para que los cuadrados sean de cuatro cxifras.
+ = 101
− = 11
ª ª
ª ª 2 = 90 ⇒ = 45
2 = 112 ⇒ = 56
⇒ = = 45 =
2025
Sean a y b dos números naturales tales que
Determina el mayor valor posible de a/b.
SOLUCIÓN
= ⇒ + 5 = + 15 − 2 − 30 ⇒ 10 − 2 − 30 = 0 ⇒ 5 = + 15 ⇒ =
Entonces, = = = + ⇒ = +
Evidentemente, el mayor valor de la fracción corresponderá al menor valor posible de , teniendo en cuenta
que tanto él como = son números naturales por lo que debe ser = 5 ⇒ = = 4
En resumen, =
4/5
Sea un cuadrado ABCD de lado 1 y un cuadrado interior de lado 1/2.
Halla el radio de la circunferencia que es tangente a dos de los lados del cuadrado ABCD y
que pasa por un vértice del cuadrado interior.
SOLUCIÓN
Llamamos a la longitud del radio de la circunferencia.
Observamos que la diagonal del cuadrado grande vale, aplicando el teorema de Pitágoras,
en el triángulo : = √1 + 1 = √2
Aplicando el teorema de Pitágoras en , = + = + = =
√
=
√
=
Aplicando el teorema de Pitágoras en , = √ + = √2 = √2 ×
Con todo lo anterior, = + + =
√
+ + √2 × = √2 ⇒ + √2 × =
√
⇒ =
√
× √
⇒
⇒ =
√ × √
× √ × √
=
√
=
(2 – √2)/2 = 0,2929
Sea ABC un triángulo tal que el ángulo A es el triple del ángulo B
Si BC = 5 y CA = 3, calcula la medida del lado AB
SOLUCIÓN
Sea = el valor buscado y = ⇒ = 3
Aplicando el teorema de los senos, = ⇒ 3 × sen 3 = 5 × sen ⇒
⇒ 3 × sen 2 × cos + cos2 × sen = 5 × sen ⇒
⇒ 3 × 2 × sen × cos × cos + cos − sen × sen = 5 × sen
÷
!!!"
⇒ 3 × 2 × cos + cos − sen = 5 ⇒ 3 × 3 × cos − 1 + cos = 5 ⇒ 4 × cos − 1 = ⇒
⇒ 4 × cos = + 1 =
%
⇒ cos =
%
&
= ⇒ cos = '
Aplicaremos ahora el teorema del coseno. Para ello, calculamos ( = 180° − − = 180° − − 3 ⇒
⇒ ( = 180° − 4 , y, entonces, = 3 + 5 − 2 × 3 × 5 × cos ( = 9 + 25 − 30 × cos 180° − 4 ⇒
⇒ = 34 + 30 × cos 4
Por otro lado, cos 2 = cos − sen = 2 × cos − 1 = 2 × − 1 =
-
− 1 =
&
Y cos 4 = cos 2 − sen 2 = 2 × cos 2 − 1 = 2 × .
&
/ − 1 = 0
− 1 = −
1
0
Resultando así que = 34 + 30 × cos4 = 34 + 30 × .−
1
0
/ = 34 −
12
⇒ = ⇒ = ' =
3,266
Sea f una función tal que
• f(n) = 0 si las cifras de las unidades de n es 4
• f(a×b) = f(a) + f(b)
Calcula f(2019)
SOLUCIÓN
Como 6 × 4 = 24, 6 + 4 = 6 × 4 = 24 ⇒ 6 + 0 = 0 ⇒ 6 = 0
Entonces, 2019 × 6 = 12114 ⇒ 2019 + 6 = 2019 × 6 = 12114 ⇒ 2019 + 0 = 0 ⇒
⇒ 2019 =
0
Sea P el número que se obtiene al multiplicar los factoriales de los primeros 2008 enteros positivos:
Halla uno de estos factoriales tal que, eliminándolo, la multiplicación de los 2007 factoriales restantes sea un
cuadrado perfecto.
SOLUCIÓN
Si tenemos en cuenta que ! × + 1 ! = ! × ! × + 1 = + 1 × ! ., el producto propuesto queda
= 1! × 2! × 3! × … … … × 2007! × 2008! = 1! × 2! × 3! × 4! × … … … × 2007! × 2008! ⇒
⇒ = 2 × 1! × 4 × 3! × 6 × 5! × … … … × 2006 × 2005! × 2008 × 2007! ⇒
⇒ = 2 × 4 × 6 × … × 2006 × 2008 × 1! × 3! × 5! × … … … × 2005! × 2007! ⇒
⇒ = 2 × 4 × 6 × … × 2006 × 2008 × , con = 1! × 3! × 5! × … … … × 2005! × 2007!
De lo anterior se deduce que
= 2 × 4 × 6 × … × 2006 × 2008 × = 2 × 1 × 2 × 3 × … × 1003 × 1004 × ⇒
= 2 × 1004! × ⇒ = 1004! × 2 ×
siendo el factorial a eliminar el
1004!
Halla un número natural n tal que si a su expresión se le coloca un 2 por la izquierda y un 1 por
la derecha, el número resultante sea igual a 33n
SOLUCIÓN
Si es el número de dígitos de incrementado en una unidad, 2 1 = 2 × 10 + 10 + 1 = 33 ⇒
⇒ 2 × 10 + 1 = 23 ⇒ =
×
=
…
Es cuestión de ir dividiendo 2000 … entre 23 hasta que de un resto de 22:
2000 … 23
160
22
86
Esto significa que 2000 = 23 × 86 + 22 ⇒ 2000 + 1 = 23 × 86 + 23 ⇒ 2001 = 23 × 87 ⇒ = =
87
siendo 2871 = 33 × 87
Halla los dígitos X, Y, Z, con X > Y > Z tales que la siguiente resta entre números de tres cifras sea correcta.
SOLUCIÓN
Con las condiciones expuestas se deduce, a partir de cada resta individual de derecha a izquierda que
10 + − =
10 + − 1 − =
− 1 − =
⇒
10 + − =
= 9
− 1 − =
⇒
= 10 − 4 − 9 ⇒ = 5
= 9
2 = 8 ⇒ = 4
X=9; Y=5; Z=4
Halla un número de tres cifras sabiendo que la suma de sus cifras es 9, el producto de
las mismas es 24 y, además, el número leído de derecha a izquierda es 27/38 del
número primitivo.
SOLUCIÓN
Sea el número
El enunciado nos dice que
+ + = 9
× × = 24
= ×
, siendo , , dígitos
= × ⇒ 38 × 100 + 10 + = 27 × 100 + 10 + ⇒ 2662 − 110 − 3773 = 0
÷
÷
242 − 10 − 343 = 0
+ + = 9 ⇒ = 9 − − , luego 242 − 10 − 343 = 0 ⇒ 242 − 10 × 9 − − − 343 = 0 ⇒
⇒ 252 = 333 + 90 ⇒ =
"#$%
&
⇒ = +
"#$%
&
⇒ = +
$"# %
En resumen, '
= +
$"# %
× × = 24
, siendo , , dígitos
Se obtiene, de lo anterior, que = 2 ⇒ = 2 +
$× # %
= 3 ⇒ =
(
)×"
=
(
×
= 4
En número es
342
Si la escalera mecánica está detenida Antonio la sube en 30 segundos y si la escalera mecánica
está funcionando una persona que no se mueve la sube en 60 segundos.
Determina cuánto tarda Antonio en subir si la escalera funciona pero él camina sobre ella.
SOLUCIÓN
Llamamos a la longitud de la escalera y al tiempo pedido.
La velocidad de Antonio subiendo la escalera que está en marcha será la suma de la velocidad de Antonio con la
escalera parada y la velocidad de subida de la escalera:
=
30
+
60
=
2 +
60
=
3
60
=
20
⇒ =
20
⇒ =
20 segundos
Sea ABCD un rectángulo cuyos lados miden AB=a y BC=b. Dentro del rectángulo se
trazan dos circunferencias tangentes exteriormente de manera que una es
tangente a los lados AB y AD y la otra es tangente a los lados CB y CD.
Calcula la distancia entre los centros de las circunferencias en función de a y b.
SOLUCIÓN
Dibujamos y señalamos puntos y líneas:
es el centro de la circunferencia de radio y es el centro de la circunferencia de
radio
La longitud pedida es = = + que es la hipotenusa del triángulo rectángulo
, siendo sus catetos = e =
Observando las longitudes horizontales y las verticales se observa que
+ + = =
+ + = =
⇒
+ =
+ =
⇒
= −
= −
Y por el teorema de Pitágoras en el triángulo rectángulo : = + ⇒ = − + − ⇒
⇒ = − 2 + + − 2 + ⇒ − 2 × + × + + = 0 ⇒
⇒ = + ± + − + = + ± √ + 2 + − − ⇒ = + ± √2
Es evidente que ≤ < + ⇒ < + , luego el valor pedido es
+ − √!
Sea ABCD un cuadrado de lados AB=BC=CD=DA=16 y P un punto en el lado BC
La recta perpendicular a AP trazada por A corta a la prolongación del lado CD en Q
Si AP=20, calcula DQ
SOLUCIÓN
Está claro que los ángulos y son iguales porque sus lados son perpendiculares
dos a dos: , y ,
Además = = 16 por lo que los triángulos citados son iguales: = = 20
Aplicando el teorema de Pitágoras en el triángulo ,
= − = 20 − 16 = 400 − 256 = 144 ⇒ = √144 =
12
Se escriben las cifras de 2019 como sigue:
Calcula cuántos dígitos se deben escribir para que la suma de los dígitos escritos sea 3900.
SOLUCIÓN
Como 2 + 0 + 1 + 9 = 12, en la primera presencia del año el valor de la suma es 12. En la segunda presencia es
12 × 2 = 24, en la tercera 12 × 3 = 36 y, sucesivamente, en la n-sima es 12 × = 12
La suma de los dígitos hasta la n-sima presencia es 12 + 24 + 36 + ⋯ + 12 = 12 × 1 + 2 + 3 + ⋯ + =
= 12 ×
×
⇒ 6 × × + 1 = 3900 ⇒ 6 + 6 − 3900 = 0
÷
+ − 650 ⇒ =
±√ ×
⇒
⇒ =
±√
=
±
, habiendo usado la fórmula de la suma de los n primeros términos de una progresión
aritmética.
Por lo tanto, en el contexto del problema, = = 25
La primera presencia del año exige 4 dígitos, la segunda 8, la tercera 12 y, sucesivamente, la n-sima 4
En ese momento, la cantidad de dígitos es 4 + 8 + 12 + ⋯ + 4 = 4 × 1 + 2 + 3 + ⋯ + = 4 ×
×
Entonces, el número de dígitos es 4 ×
×
= 2 × 25 × 26 =
1300
Calcula la suma de los dígitos del número
SOLUCIÓN
= 10 − 2019 = 1 000. . .000 − 2019 = 999. . .999 7981
Por lo tanto, la suma de sus dígitos es 2015 × 9 + 7 + 9 + 8 + 1 = 18135 + 25 =
18160
Un cuadrado ABCD se divide en dos cuadrados y tres rectángulos, como se muestra en
la figura.
El área de cada uno de los cuadrados es a y el área de cada uno de los dos rectángulos
más pequeños es b.
Si a + b = 24 y la raiz cuadrada de a es un número natural, halla todos los valores
posibles del área del rectángulo sperior.
SOLUCIÓN
Como es un cuadrado perfecto, llamamos a la longitud del lado de los cuadrados
pequeños, valor entero positivo.
De ahí, la longitud del lado del cuadrado grande es 2 y está claro que < . Como
+ = 24 = 16 ⇒ = √ = √16 = 4
La superficie buscada es, entonces Á ! − 2 × $ + % = $2 %&
− 2 × $ + % =
= 8&
− 2 × 24 = 64 − 48 =
16 unidades cuadradas
Se tiene un cubo de arista n, pintado de rojo y se divide el cubo en n3
cubitos de arista 1.
La cantidad de cubitos que no tienen ninguna cara pintada es igual a 27 veces la cantidad
de cubitos que tienen exactamente 2 caras pintadas.
Halla n.
SOLUCIÓN
Evidentemente > 2 porque, en caso contrario, no habría cubitos interiores, sin pintar.
La cantidad de cubitos interiores, sin pintar, es − 2 pues de cada cara hay que quitar los que están en la
superficie.
Los cubitos con dos caras pintadas son los que se encuentran en las 12 aristas, excepto los de las esquinas:
12 × − 2
Así, − 2 = 27 × 12 × − 2
÷ ;
− 2 = 27 × 12 = 3 × 3 × 2 = 3 × 2 ⇒ − 2 = 3 × 2
Es decir, − 2 = 18 ⇒ = 18 + 2 =
20
Se pretende cubrir totalmente un cuadrado de lado k (k entero mayor que uno) con
los siguientes rectángulos: 1 rectángulo de 1x1, 2 rectángulos de 2x1, 4 rectángulos
de 3x1, ... , 2n
rectángulos de (n+1)x1, de tal manera que los rectángulos no se
superpongan ni excedan los límites del cuadrado.
Halla los valores de k para los que esto es posible y, para cada valor de k encontrado,
dibuja una solución.
SOLUCIÓN
Se trata de encontrar > 1 entero tal que = 1 × 1 × 1 + 2 × 2 × 1 + 4 × 3 × 1 + ⋯ + 2 × + 1 × 1 ⇒
⇒ = 2 × 1 + 2 × 2 + 2 × 3 + ⋯ + 2 × + 1
Tenemos en cuenta la suma de los primeros términos de una progresión geométrica de razón :
×
Como 2 , 2 , 2 , … , 2 es una progresión geométrica de ! − # + 1 términos de razón 2, su suma es
entonces
$×% &'$( )
= 2 − 2
De ahí, = 2 × 1 + 2 × 2 + 2 × 3 + ⋯ + 2 × + 1 =
= 2 + 2 + 2 + ⋯ + 2 + 2 + 2 + ⋯ + 2 + 2 + ⋯ + 2 + ⋯ + 2 + 2 + 2 ⇒
⇒ = 2 − 2 + 2 − 2 + 2 − 2 + ⋯ + 2 − 2 + 2 − 2 ⇒
⇒ = 2 × + 1 − 2 + 2 + 2 + ⋯ + 2 = 2 × + 1 − 2 − 2 ⇒
⇒ = 2 × + 1 − 1 + 2 ⇒ = 2 × + 1
Ahora bien, = 2 × + 1 ⇒ − 1 = 2 × ⇒ + 1 × − 1 = 2 × y impar y, como
2 > , deberá cumplirse que *
+ 1 = 2+
− 1 = 2,
×
-, donde . > / y . + / = + 1 y obsérvese que la diferencia
entre los dos factores es de dos unidades.
Esta igualdad se cumple únicamente para = 3 y = 7: 1 + 1 = 22
− 1 = 2 × 3
-, por lo que
k = 7
y una solución es
Toni sumó cinco números naturales consecutivos y el resultado que obtuvo es el número de
cinco cifras 1x84x, con el dígito de las unidades x igual al de las unidades de mil.
Determina los cinco números que sumó Toni y da todas las posibilidades.
SOLUCIÓN
Sean los números , + 1, + 2, + 3, + 4
Según el enunciado, + + 1 + + 2 + + 3 + + 4 = 1 84 ⇒ 5 + 10 = 10000 + 1000 + 840 + ,
por lo que 5 = 10830 + 1001 ⇒ = ⇒ = 2166 + 200 +
Como es un dígito y un número natural, los únicos valores admisibles son:
= 0 ⇒ = 2166 + 200 × 0 + ⇒ = 2166
= 5 ⇒ = 2166 + 200 × 5 + ⇒ = 3167
y las quíntuplas posibles son
2166, 2167, 2168, 2169, 2170
3167, 3168, 3169, 5170, 3171
A cada número entero positivo n de dos cifras se le resta la suma de los cuadrados de
sus dígitos.
¿Para qué valores de n esta diferencia es la mayor posible?
SOLUCIÓN
Sea el número de dos cifras genérico = 10 + con la cifra de las decenas y la de las unidades.
Es evidente que ≥ y la igualdad solo ocurre cuando = 0 o = 1. Será entonces, en estos casos, cuando
el valor propuesto = 10 + − − sea, en cada decena, mayor.
Por lo tanto, hay que analizar el valor = 10 + − − = 10 + − − = 10 − con unidades
= 0 o = 1.
Planteando la función cuadrática = 10 − , como = 10 − 2 = 0 si = 5 y "
5 = "
= −2 < 0 ⇒
⇒ tiene un máximo en = 5, luego este valor da diferencia máxima cuando = 0 o = 1.
En conclusión, la diferencia máxima posible es = 5 = 10 × 5 − 5 = 25 con los números
n = 50 y n = 51
Un tren viaja de Zaragoza a Barcelona, con dos paradas
intermedias, primero Lérida y después Tarragona.
Cuando se detiene en Lérida, la cantidad de pasajeros que sube es
igual a 3/4 de los pasajeros que viajaron de Zaragoza hasta Lérida,
y bajan 39 pasajeros.
En la estación de Tarragona, la cantidad de pasajeros que sube es
igual a 3/4 de los pasajeros que viajaron de Lérida hasta
Tarragona, y bajan 39 pasajeros.
La cantidad de pasajeros que llegaron a Barcelona es igual a la
cantidad de pasajeros que salieron de Zaragoza.
Averigua cuántos pasajeros salieron de Zaragoza.
SOLUCIÓN
Sea la cantidad de pasajeros que salieron de Zaragoza y llegaron a Barcelona.
De Lérida salen + − 39 = − 39 pasajeros, y de Tarragona salen − 39 + × − 39 − 39 =
= × − 39 − 39 pasajeros, por lo que × − 39 − 39 = ⇒ − − 39 = ⇒
⇒ 49 − 1092 − 624 = 16 ⇒ ⇒ 49 − 16 = 1092 + 624 ⇒ 33 = 1716 ⇒ = =
52 pasajeros
Curiosamente, siempre va en el tren la misma cantidad de pasajeros: 52
La víctima de un accidente morirá a menos que reciba en los próximos 10 minutos una
transfusión de sangre tipo A-Rh positivo.
Se dispone de gran número de donantes de los cuales sólo se sabe que el 40% tienen sangre de
ese tipo.
Se necesitan dos minutos para determinar el tipo de sangre del posible donante y dos minutos
para realizar la transfusión.
¿Cuál es la probabilidad de que se salve si el hospital dispone de un sólo equipo de tipificación
de sangre?
SOLUCIÓN
Hay un 40% de donantes adecuados y un 60% de donantes no adecuados.
La probabilidad de que en los dos primeros minutos se identifique un donante bueno es =
La probabilidad de que se encuentre al donante entre los minutos dos y cuatro es × = × =
Que sea entre los minutos cuatro y seis × × = × × =
Que sea entre los minutos seis y ocho × × × = × × × =
Inmediatamente después de la localización se realiza la transfusión hasta, como mucho, el minuto diez.
La probabilidad de que se salve es + + + = = =
0,8704 = 87,04 %
Sea ABCD un cuadrilátero de lados AB, BC, CD y DA tal que ^ABC=90o
, ^ACD=90o
y BC=CD.
Las diagonales AC y BD se cortan en O.
Si ^AOD=110o
, calcula ^BAC.
SOLUCIÓN
Sea = el ángulo a determinar, en el triángulo rectángulo
El triángulo es isósceles pues = ⇒ =
°
2 × + = 180° ⇒
⇒ 2 × + + 90° = 180° ⇒ 2 × + = 90° [ ]
Por otro lado, en el triángulo , + + = 180° ⇔ + + = 180° ⇒
⇒ + 110° + = 180° ⇒ + = 70° [ ]
Restando, [ ] − [ ]: = 20°
% & °
20° + = 70° ⇒ = 50°
Y, en el triángulo rectángulo , = 90° − = 90° − 50° =
40o
Para la construcción de una pista circular de patinaje sobre hielo, se
tienen las propuestas de dos empresas.
La empresa Ice&Co cobra 40 euros por m2
de pista, 30 euros el metro
de cerco y un adicional fijo de 400 euros para gastos generales.
La empresa WinterSports cobra 36 euros por m2
de pista, 40 euros el
metro de cerco y un adicional fíjo de 1200 euros.
¿Para qué valores del diámetro de la pista es más ventajoso contratar la empresa Ice&Co?
SOLUCIÓN
Sea el radio, en metros, de la pista de hielo que se desea construir.
La empresa Ice&Co cobrará = 40 × × + 30 × 2 × × + 400 = 40 + 60 + 400 euros y la
empresa WinterSports cobrará = 36 × × + 40 × 2 × × + 1200 = 36 + 80 + 1200 euros
por el trabajo.
Habrá un valor concreto del radio en el que el coste sea el mismo. Lo calculamos:
40 + 60 + 400 = 36 + 80 + 1200 ⇒ 4 − 20 − 800 = 0 ⇒ =
± ×
⇒
⇒ =
±√
!"# =
√
=
×$
%
&
⇒ =
'
× (1 + $1 + ) = 10,86 m
Para valores de < 10,86: ,
10 = 40 × 10 + 60 × 10 + 400 = 4600 + 400 = 14851,33 €
10 = 36 × 10 + 80 × 10 + 1200 = 4400 + 1200 = 15023,01 €
0 ⇒
⇒ < : más ventajoso el contrato con Ice&Co
Para valores de > 10,86: ,
11 = 40 × 11 + 60 × 11 + 400 = 5500 + 400 = 17678,76 €
11 = 36 × 11 + 80 × 11 + 1200 = 5236 + 1200 = 17649,38 €
0 ⇒
⇒ > : más ventajoso el contrato con WinterSports
Es decir, que el contrato con Ice&Co es más ventajoso para valores del diámetro menores de 2 = 2 × 10,86 =
21,72 metros
En una reunión de 152 científicos, algunos son matemáticos y los demás son físicos.
El promedio de las edades de todos los científicos es de 41 años, el promedio de las
edades de los matemáticos es 35 años, y el promedio de las edades de los físicos es
51 años.
Determina cuántos científicos de esta reunión son matemáticos.
SOLUCIÓN
Sean , la cantidad respectiva de matemáticos y de físicos existentes en la reunión: + = 152
La media de edades de los matemáticos es
á
= 35 ⇒ á = 35
La media de edades de los físicos es
í
"
= 51 ⇒ #í = 51
O sea, la suma de todas las edades de los científicos es 35 + 51 , por lo que la media de edades de los
científicos es
$% &%'"
'%(
= 41 ⇒ 35 + 51 = 41 × 152 = 6232
Entonces, ,
+ = 152
35 + 51 = 6232
- 'ª→%'×'ª
0111112 ,
51 + 51 = 7752
35 + 51 = 6232
-
'ª4(ª
0112 16 = 1520 ⇒ =
'%(6
'7
=
95 matemáticos
Sean x y d números naturales tales que el resto de dividir x por d es igual a 4 y el resto de dividir
14x por d es 17.
Halla el resto de dividir 210x por d.
SOLUCIÓN
= × + 4
14 = × + 17
⇒ 14 × + 4 = + 17 ⇒ 14 + 56 = + 17 ⇒ − 14 = 56 − 17 ⇒
⇒ × − 14 = 39 La única factorización de 39, aparte de la trivial, es 39 = 13 × 3, pero > 17 ⇒
⇒
= 39
− 14 = 1
Por lo tanto, 14 = + 17 ⇒ 14 = 39 + 17
×
!!!!!!" 210 = 15 × 39 + 17 = 39 × 15 + 255
Como 255 = 39 × 6 + 21 ⇒ 210 = 39 × 15 + 39 × 6 + 21 ⇒ 210 = 39 × 15 + 6 + 21
y el resto pedido es
21
En la expresión
Agustín reemplazó cada * por un signo + o un signo – de modo que quedaron 5 signos de cada clase, y realizó
la expresión indicada.
El resultado es un número positivo de dos dígitos que es múltiplo de 7.
Determina qué número obtuvo Agustín e indica una posible asignación de los signos + y – con la que se
obtiene ese número.
SOLUCIÓN
La suma de los diez primeros números naturales es 55: 1 + 2 + 3 + 4 + 5 + 6 + 7 + 8 + 9 + 10 = 55
Por lo tanto, el número múltiplo de 7 resultante de la operación debe ser 49 = 7 × 7 ≥ ≥ 7 × 2 = 14
Analizamos cada caso:
• = 7 × 7 = 49. Como 55 − 49 = 6, debería haber 5 números negativos cuya suma fuera = 3 para,
así, la suma de los demás positivos sería 55 − 3 = 52 y el total 52 − 3 = 49. Imposible porque no hay
cinco números que sumen −3
• = 7 × 6 = 42. Como 55 − 42 = 13, debería haber 5 números negativos cuya suma fuera , lo cual
es imposible por no ser un número entero.
• = 7 × 5 = 35. Como 55 − 35 = 20, debería haber 5 números negativos cuya suma fuera = 10
para, así, la suma de los demás positivos sería 55 − 10 = 45 y el total 45 − 10 = 35. Imposible porque
no hay cinco números que sumen −10
• = 7 × 4 = 28. Como 55 − 28 = 27, debería haber 5 números negativos cuya suma fuera , lo cual
es imposible por no ser un número entero.
• = 7 × 3 = 21. Como 55 − 21 = 34, debería haber 5 números negativos cuya suma fuera = 17
para, así, la suma de los demás positivos sería 55 − 17 = 38 y el total 38 − 17 = 21. Los cinco números
que sumasen −17 podrían ser −1 − 2 − 3 − 4 − 7 = −17
• = 7 × 2 = 14. Como 55 − 14 = 41, debería haber 5 números negativos cuya suma fuera , lo cual
es imposible por no ser un número entero.
En resumen, el número que obtuvo Agustín es
21
con esta posible operación:
– 1 – 2 – 3 – 4 + 5 + 6 – 7 + 8 + 9 + 10
y otra posible operación es:
– 1 – 2 – 3 + 4 – 5 – 6 + 7 + 8 + 9 + 10
Se dispone de 98 tarjetas y en cada una de ellas está escrito uno de los números 1, 2, 3, ..., 98
(no hay números repetidos).
Ordena las 98 tarjetas de modo tal que, al considerar dos tarjetas consecutivas, la diferencia
entre el número mayor y el número menor escritos en ellas sea siempre mayor que 48.
SOLUCIÓN
Es evidente que la tarjeta número 49 debe iniciar o terminar la secuencia, pues solo hay otro número entre los
indicados que se diferencie, con él, un valor superior a 48: es 98, que debe ir a su lado.
A partir de ahí, por lógica, una secuencia adecuada es
49-98-48-97-47-96-46-95-… … …-5-54-4-53-3-52-2-51-1-50
Sea ABCD un cuadrado de lado 6. Sean P en el lado BC y Q en el lado CD tales que las rectas
AP y AQ dividen al cuadrado en tres figuras de áreas iguales.
Calcula el área del triángulo APQ
SOLUCIÓN
Como las tres partes son iguales los triángulos rectángulos y son iguales: =
Sea = =
El área del triángulo es la tercera parte del área del cuadrado:
×
= × ⇒
⇒ = × 6 ⇒ 3 = 12 ⇒ = 4
Entonces, Á = Á = 3 ! Á = 12 ⇒ Á + Á = 12 ⇒ Á +
#
= 12,
pues $ = $ = 6 − = 6 − 4 = 2
Á + 2 = 12 ⇒ Á =
10 unidades cuadradas
Determina todas las funciones f tales que
cualesquiera que sean los números reales x, y.
SOLUCIÓN
Consideramos − + 2 + 1 = + × − , ∀ , ∈
Si = = 0, − + 2 + 1 = + × − 0 − 0 + 1 = 0 × 0 ⇒
⇒ 0 = 1 ⇒ 0 = ±1
Si = , − + 2 + 1 = + × − − + 2 + 1 = 2 × 0 ⇒
±
2 + 1 = 2 × ±1 ⇒ 2 = ± 2 + 1 , ∀ ∈ ⇒ = ± + 1 , ∀ ∈
Verificando ahora las dos expresiones:
•
− + 2 + 1
+ × −
+ 1 − + 1 + 2 + 1 = − + 2 + 1
+ + 1 × − + 1 = − + + + − + 1
⇒
⇒
− 1 − + 1 + 2 + 1 = − + 2 + 1
+ + 1 × − + 1 = − + 2 + 1
⇒ Iguales. Ok
•
− + 2 + 1
+ × −
! ! − − 1 − − − 1 + 2 + 1 = − + + 2 + 1
− − − 1 × − + − 1 = − + + + − + 1
⇒
⇒
− 1 − + 1 + 2 + 1 = − + + 2 + 1
− − − 1 × − + − 1 = − + 2 + 1
⇒ Distintos
Las funciones son, entonces,
f(x) = x + 1, con x real
Angelines escribe una lista de números naturales.
El primer número es el 1; luego escribe los múltiplos de 2, desde 2 hasta 22
; a continuación escribe los
múltiplos de 3, desde 3 hasta 32
; luego los múltiplos de 4, desde 4 hasta 42
, y así siguiendo hasta escribir, por
primera vez, el 2019. La lista empieza de la siguiente manera:
Determina cuántos números tiene la lista de Angelines.
SOLUCIÓN
Para cada , la lista hasta consta, exactamente, de números.
Como 2019 = 3 × 673, el número 2019 aparecerá en primer lugar como tercero de la lista de los factores de
673.
Por lo tanto, la lista constará de todos los correspondientes de 1 a 672 más los tres factores citados:
1 + 2 + 3 + 4 … + 672 + 3 =⏞
é
ó
é !1 + 672" × 672
2
+ 3 = 673 × 336 + 3 =
226131 números
Halla el menor entero positivo n tal que las 73 fracciones
sean todas irreducibles.
SOLUCIÓN
Evidentemente, debe ser impar. Si es par hay 36 fracciones, al menos, simplficables.
Como las fracciones son del tipo , si = − 2 la fracción es = = = siempre reducible.
En conclusión, debe ser > 91 − 2 = 89 e impar.
Además, como el primer número lo obtendremos cuando + 2 sea primo: =
95
Si = 91, = = es reducible
Si = 93, = = es reducible
La aldea de Astérix comparte un banquete de camaradería a escote,
con precio fijo, en honor a Obélix, que es invitado y no paga.
El coste total asciende a 1680 denarios y hay que pagarlo.
Dividen entre el número de participantes que pagan pero el dinero no
alcanza porque 4 de ellos se han marchado, así que cada uno de los
presentes debe agregar 1 denario.
Calcula cuántos comensales hubo en el almuerzo.
SOLUCIÓN
Sea el número inicial de comensales. El gasto de cada uno debe ser euros.
Como faltan 4 para pagar, el gasto de los − 4 que quedan debe ser de + 1 euros:
− 4 × + 1 = 1680 ⇒ 1680 + − − 4 = 1680 ⇒ − 4 − 6720 = 0 ⇒
⇒ = 2 ± √2 + 6720 = 2 ± √6724 = 2 ± 82
En el contexto del problema, el número de participantes que debían pagar en el almuerzo es = 2 + 82 = 84
y, con Obélix,
85
Las mesas de la biblioteca están numeradas correlativamente.
¿Cuál es el número de la mesa en donde está leyendo Blanca?
SOLUCIÓN
Leyendo los números de las mesas desde la posición de Blanca,
su mesa es la número
67
Se eligen cinco números naturales distintos, a, b, c, d, e, ordenados de menor a mayor: 1≤a<b<c<d<e.
Se calcula el mínimo común múltiplo de cada número con el siguiente: mcm(a, b); mcm(b, c); mcm(c, d);
mcm(d, e); y, finalmente, se efectúa la suma de sus inversos
¿Cuál es el máximo valor que puede tener el resultado final S ?
SOLUCIÓN
Cuanto más pequeños sean los valores de los “mínimo común múltiplo” más grandes serán las fracciones y, por
tanto, .
Es evidente que debe ser = 1; = 2. A partir de ahí si pretendemos que = 3, para que haya un mínimo
común múltiplo mcm , lo más pequeño posible debe ser = 6. Por el mismo razonamiento, debe ser
= 12 para que los denominadores sean los más pequeños posibles y, así, sea quizás el mayor valor posible:
= ,
+ ,
+ ,
+ ,
= ,
+ ,
+ ,
+ ,
⇒
⇒ = + + + = =
Haciendo los números sucesivas potencias de 2 conseguimos un valor mayor, que es el mayor posible:
= ,
+ ,
+ ,
+ ,
= ,
+ ,
+ ,!
+ !,
⇒
⇒ = + + !
+ =
!
=
15/16
Se tienen dos figuras superpuestas: el cuadrado ABCD de lado 6 y el triángulo isósceles ABE de
base AB, con AE=BE y E fuera del cuadrado.
Si el área de la superposición es igual a 3/4 del área del cuadrado, calcula el área de la porción
del triángulo que no se superpone con el cuadrado.
SOLUCIÓN
Se trata de calcular la superficie del triángulo isósceles :
×
, siendo la longitud de su
base y ℎ la de la altura.
La superficie del cuadrado es 6 = 36 unidades cuadradas.
Como el área de superposición (el trapecio sombreado ) es del área del cuadrado,
×
= × 36 ⇒ 3 + 18 = 27 ⇒ 3 = 9 ⇒ = 3 unidades lineales.
Los triángulos y , por lo que se puede establecer la proporción = ⇒ = ⇒ = ℎ ⇒
⇒ ℎ + 6 = 2ℎ ⇒ ℎ = 6 unidades lineales.
En resumen, la superficie buscada es
×
=
×
=
9 unidades cuadradas
La figura adjunta está formada por tres cuadrados iguales, en color naranja, y tres
rombos iguales, en color azul.
Halla los ángulos de los rombos.
SOLUCIÓN
Llamamos al ángulo de los rombos correspondiente al vértice común de los cuadriláteros.
Como los cuadrados tienen ángulos de 90° ⇒ 3 + 3 × 90° = 360° ⇒ 3 = 360° − 270° = 90° ⇒ = 30°
El otro ángulo de los rombos es 180° − = 180° − 30° = 150°
En resumen, los rombos tiene los ángulos de medidas
30o
y 150o
En el cuadrado ABCD, sea P en el lado AB tal que AP2
=BP×BC y sea M el punto medio
de BP.
Si N es el punto interior del cuadrado tal que AP=PN y MN es paralelo a BC, calcula la
medida del ángulo ^BAN.
SOLUCIÓN
Sea la longitud del lado del cuadrado y = = ⇒ = − .
Según el enunciado, = × ⇒ = − × ⇒ − + = 0
÷
⇒ − + = 0
÷
− 1 + = 0.
Haciendo = ⇒ + − 1 = 0 ⇒ =
√ !
⇒ =
√"
, y solo tomando el
valor positivo por su construcción como cociente de longitudes.
Como = − × ⇒ − = =
Entonces, # =
$%
=
&% &$
= ⇒ # = + # = + = +
'
⇒ # = × 1 +
'
En el triángulo rectangulo # , # = − # = − = −
'
= −
'
!
⇒
⇒ # = × 1 −
'
!
⇒ # = × (1 −
'
!
En resumen, tan , =
-.
&-
=
×( /
0
×
/ =
( /
0
/ =
(
/
×
/
( /
⇒ tan , = 1
/
/ = (
'
'
= 1
√234
√234
⇒
⇒ tan , = (
" √"
5 √"
= (
6" √"7×65 √"7
65 √"7×65 √"7
= ( 8 9×√"
!
⇒ tan , = :5 − 2 × √5 ⇒ , =
36o
Una empresa maderera obtuvo un contrato para cortar árboles de un
bosque y los ecologistas iniciaron una protesta en su contra.
Para evitar las protestas, el gerente de la empresa agregó la siguiente
cláusula al contrato:
“En el bosque, el 99% del total de árboles son pinos, y la empresa sólo
cortará pinos. Cuando se termine el contrato, el 97% del total de árboles
del bosque serán pinos.”
Determina qué porcentaje del bosque será cortado por la empresa al cumplirse esta cláusula del contrato.
SOLUCIÓN
Llamamos al número total de árboles existentes y al número de pinos que se cortarán.
La cantidad de pinos existentes es de × 99% =
Si cortamos pinos quedarán − que deben ser el 97% de los árboles que quedan: − . Por lo tanto,
− = − × 97% =
×
⇒ 99 − 100 = 97 − 97 ⇒ 2 = 3 ⇒ = ×
O sea, se cortarán los del bosque: el
66,67 % del bosque
Del entero positivo n se sabe que:
• n no es múltiplo de 5
• n – 96 es múltiplo de 128
• n tiene 2001 dígitos
• todos los dígitos de n son pares
• la suma de los dígitos de n es 2×2001 – 4 = 3998
• la suma de los cuadrados de los dígitos de n es 4×2001 = 8004
Halla n.
SOLUCIÓN
Sean , , … , los dígitos de , todos pares.
Según el enunciado,
+ + ⋯ + = 3398 = 2 × 2001 − 4
+ + ⋯ + = 8004 = 4 × 2001
Tomando los valores mitades de los dígitos,
+ + ⋯ + = 2001 − 2 = 1999
+ + ⋯ + = 2001
, de donde se deduce
inmediatamente que debe haber, como mitades de dígitos, 1997 unos, 1 dos y 3 ceros.
En consecuencia, los dígitos del número son 1997 doses, 1 cuatro y 3 ceros.
Por otro lado, el enunciado dice que − 96 = 128 ⇒ = 128" + 96 ⇒ = 32 × #4" + 3$
Es múltiplo de 32: sus cinco últimas cifras deben formar un número múltiplo de 32
Las dos últimas cifras (teniendo en cuenta las que tiene y que no es múltiplo de 5) que le permiten ser múltiplo
de 4 son 04 y 24, por lo que acaba en 4
De esas terminaciones, únicamente 024 y 224 le permiten ser múltiplo de 8: acaba en 24
De las anteriores terminaciones, 0224 y 2224 le permiten ser múltiplo de 16: acaba en 224
De las anteriores terminaciones, 00224 y 20224 le permiten ser múltiplo de 32: acaba en 0224
Entonces,
Si acabase en 20224, ninguna combinacón de dos dígitos más por delante #02, 20, 22$ y restándole 96 da un
múltiplo de 128
Si acabase en 00224, = 32 × #4" + 3$ = 10&
' + 00224 = 10&
' + 7 × 32 = 10&
' + #4 + 3$ × 32, por lo
que debería ser 10&
' = 32 × 4( y ' debería ser múltiplo de 4. La única posibilidad, con las cifras que quedan, es
que acabe en 20
Por lo tanto. El número debe ser
) = *** … … ***+,,,-,,,.
//& 01232
444**5
Sea ABC un triángulo acutángulo.
Se considera el punto D del lado AB tal que CD es perpendicular a AB, y el
punto E del lado AB tal que CE es la bisectriz del ángulo ^ACD.
Sea F el punto del lado BC tal que ^BAF = ^ACE, y G el punto de intersección
de AF y CE.
Si se sabe que el triángulo CFG es equilátero, calcula los ángulos del triángulo ABC.
SOLUCIÓN
Dibujamos el esquema del problema y llamamos = = =
Como el triángulo es equilátero, sus tres ángulos miden 60°
En el triángulo rectángulo , = 90° − = 90° −
Por lo tanto, = 180° − = 180° − 90° − ⇒ = 90° +
En el triángulo , + + = 180° + 90° + + = 180° ⇒ = 90° − 2
Al ser opuestos por el vértice, = ⇒ 60° = 90° − 2 ⇒ 2 = 30° ⇒ = 15°
En el triángulo , + + = 180° ⇒ + 90° + + = 180°
$ %&°
= 90° − 2 = 90° − 30° ⇒
⇒ = 60° (*)
Observemos que = + = + 60°
$ %&°
= 15° + 60° ⇒ = 75° (**)
En el triángulo , + + = 180° ⇒ 60° + + 75° = 180° ⇒ = 180° − 60° − 75° ⇒ = 45° (***)
De (*), (**), (***) se concluye que
A = 60o
, B = 45o
, C = 75o
Juan midió el largo del terreno de su tío con pasos de 54 cm. Después midió el
tío con pasos de 72 cm.
Quedaron marcadas en total 61 pisadas pero, a veces, la misma marca
correspondía a dos pisadas, una de Juan y otra del tío.
¿Cuál es el largo del terreno?
SOLUCIÓN
mcm 54 , 72 = mcm 2 × 3 , 2 × 3 = 2 × 3 = 216, por lo que el largo del terreno es un múltiplo de
216: 216 metros.
Por lo tanto, Juan da = 4 pasos y su tío da = 3 pasos, coincidiendo en las pisadas veces además
de la pisada inicial.
Por lo tanto, el total de pisadas visibles será 4 + 3 − + 1 = 61 ⇒ 6 = 60 ⇒ = 10, y el largo del terreno
es 216 = 216 × 10 =
2160 metros
Miguel hizo la lista de todas las progresiones aritméticas de números enteros positivos tales que la diferencia
es igual a 3 y la suma de sus términos es igual a
Calcula cuántas progresiones tiene la lista de Miguel.
SOLUCIÓN
La progresión aritmética se caracterizará por el primer término, , la diferencia, 3, y el número de términos,
> 1.
Último término es + 3 × − 1 = + 3 − 3, por lo que la suma de todos los términos es
×
= 2010 ⇒ 2 + 3 − 3 = 4020 ⇒ 2 = − 3 + 3, valor entero y par
Como 4020 = 2 × 3 × 5 × 67, probamos por los factores divisores de 4020
Observamos que, salvo el signo, la expresión debe ser entera y par, por lo que si es impar la fracción debe
ser par, lo que siempre sucede y si es par la fracción debe ser impar, para que 2 sea par.
• impar
o = 3 ⇒ 2 = − 3 × 3 + 3 ⇒ 2 = 1334 ⇒ = 667 … … … … … … … … = 3; = 667
o = 5 ⇒ 2 = − 3 × 5 + 3 ⇒ 2 = 792 ⇒ = 396 … … … … … … … … = 5; = 396
o = 3 × 5 = 15 ⇒ 2 = − 3 × 15 + 3 ⇒ 2 = 226 ⇒ = 113 … … … = 15; = 113
o = 67 ⇒ 2 =
!
− 3 × 67 + 3 ⇒ 2 < 0, contradictorio con el enunciado
o > 67 ⇒ < 0, contradictorio con el enunciado
• par
o = 2 = 4 ⇒ 2 = − 3 × 4 + 3 ⇒ 2 = 996 ⇒ = 498 … … … … … … = 4; = 498
o = 2 × 3 = 12 ⇒ 2 = − 3 × 12 + 3 ⇒ 2 = 302 ⇒ = 151 … … … = 12; = 151
o = 2 × 5 = 20 ⇒ 2 = − 3 × 20 + 3 ⇒ 2 = 144 ⇒ = 72 … … … = 20; = 72
o = 2 × 3 × 5 = 60 ⇒ 2 = − 3 × 60 + 3 ⇒ 2 < 0, contradictorio con el enunciado
o > 60 ⇒ < 0, contradictorio con el enunciado
En total hay
6 progresiones aritméticas en la lista
Halla el área de la zona sombreada de la figura, que está comprendida entre el triángulo
equilátero y la semicircunferencia de radio 1.
SOLUCIÓN
Dibujamos los segmentos auxiliares que se ven, nombramos vértices y
señalamos los datos del problema.
Por los valores, se deduce inmediatamente que los cuatro triángulos, en los
que queda dividido el triángulo equilátero original , , , ,
son equiláteros de lado unidad.
Es evidente también que los tres segmentos circulares , , tienen la
misma superficie y esta se obtiene restando el área del respectivo sector
circular menos el área del triángulo equilátero asociado.
Entonces, el área pedida es
= Á − Á + Á + Á
! #$%#&'(%
) $ &(' &* %+$#$
,----------------. = Á + Á ⇒
⇒ = Á + Á 01 − Á 2
%$á &'! 4&$'á %!
) $ &(' &* %+$#$
,----------------. = Á 01
Es decir, la superficie buscada es igual a la de uno de los sectores circulares: = Á 01 =
5×78×90°
;90°
=
π/6 = 0,5236 unidades cuadradas
En un rectángulo de 1 x 101, dividido en casillas cuadradas de 1 x 1, Guille escribió un número entero en cada
casilla de manera tal que la suma de los tres números escritos en tres casillas consecutivas era siempre igual a
9. Luego borró todos los números escritos excepto el de la tercera casilla y el de la décima casilla, contadas de
izquierda a derecha: un 3 y un 10 respectivamente.
Halla el número que había escrito Guille en la última casilla.
SOLUCIÓN
Señalamos las primeras casillas con valores desconocidos:
Se cumple que
+ + 3 = 9
+ 3 + = 9
3 + + = 9
⇒
=
=
, por lo que se repite la secuencia numérica continuamente:
Se deduce inmediatamente que = 10, luego + + 3 = 9 ⇒ 10 + + 3 = 9 ⇒ = −4 y se rellena la tabla
con el número 10 en las casillas 3 − 2, con el número −4 en las casillas 3 − 1 y con el número 3 en las
casillas 3 , ∀ ∈ , hasta la casilla 101 = 3 × 34 − 1:
El número que escribió Guille en la última casilla es
– 4
Sean p=2×3×5×7×11×13×... el producto de todos los números primos hasta 2020
y q=3×5×7×9×11×13×... el producto de todos los números impares hasta 2020.
Halla la penúltima cifra de la derecha del producto p×q.
SOLUCIÓN
El producto constará de un solo número par, el 2, y, el resto, una cantidad notable de números impares de los
cuales más de uno acaba en 5.
Por uno cualquiera de estos últimos y el 2, el producto × acaba en 0 y, debido al resto de múltiplos de 5 y de
solo impares, acaba en 50 por lo que el penúltimo número de la derecha deberá ser un
5
Félix hizo la lista de todos los números de 7 dígitos distintos que se
forman con los dígitos 1, 2, 3, 4, 5, 6 y 7.
Si los números de la lista de Félix están ordenados de menor a mayor,
calcula qué posición de la lista ocupa el número 3654721.
SOLUCIÓN
Delante del citado número estarán
• todos los que empiezan por 1 y por 2: 2 ×
• todos los que empiezan por 31, 32, 34 y por 35: 4 ×
• todos los que empiezan por 361, 362 y por 364: 3 ×
• todos los que empiezan por 3651 y por 3652: 2 ×
• todos los que empiezan por 36541 y por 36542: 2 ×
• todos los que empiezan por 365471:
En total, 2 × + 4 × + 3 × + 2 × + 2 × + = 2 × 6! + 4 × 5! + 3 × 4! + 2 × 3! + 2 × 2! + 1! =
= 2 × 720 + 4 × 120 + 3 × 24 + 2 × 6 + 2 × 2 + 1 = 1440 + 480 + 72 + 12 + 4 + 1 = 2009 números
delante del indicado.
Por lo tanto estará en la posición 2009 + 1 =
2010
Halla las tres últimas cifras del número
SOLUCIÓN
Como 19 = 20 − 1, según el binomio de Newton,
19 = 20 − 1 = × 20 × 1 − × 20 × 1 + ⋯ + × 20 × 1 − × 20 × 1
Si observamos, todos los términos, salvo los cuatro últimos, tendrán un factor que contendrá una potencia de
20 con exponente mayor o igual a 4 por lo que esos términos finalizarán en, al menos, cuatro ceros, lo que no
afectará al problema que nos ocupa teniendo en cuenta que, al haber restas, el primer término de esos cuatro sí
que interviene en la solución final.
Por lo tanto calculamos los cuatro últimos términos del desarrollo anterior, cuyo resultado nos permitirá
obtener las tres últimas cifras de la potencia propuesta:
+
97
94
× 20 × 1 −
97
95
× 20 × 1 +
97
96
× 20 × 1 −
97
97
× 20 × 1 =
= +
97!
94! × 3!
× 8000 −
97!
95! × 2!
× 400 +
97!
96! × 1!
× 20 −
97!
97! × 0!
× 1 =
= +
97 × 96 × 95
6
× 8000 −
97 × 96
2
× 400 + 97 × 20 − 1 =
= +97 × 16 × 95 × 8000 − 97 × 48 × 400 + 97 × 20 − 1 = 1177659539
Las tres últimas cifras son
539
Sean A, B y C tres puntos en una recta r con B entre A y C, y sea D un
punto exterior a r.
Se traza la recta paralela a r por el punto D que denominamos s.
Se traza la bisectriz del ángulo ^ABD que corta a la recta s en P y se
traza la bisectriz del ángulo ^CBD que corta a la recta s en Q.
Si BP = 12 y BQ = 5, calcula BD.
SOLUCIÓN
Dibujamos todo el esquema propuesto y nombramos = a la longitud pedida.
Es evidente que, por el teorema de Thales, = = y = = , por lo que los triángulos
y son isósceles y = = = ⇒ = 2 .
Como 2 + 2 = 180° ⇒ + = 90° ⇒ el triángulo es rectángulo en y, por el teorema de Pitágoras,
= + ⇒ 2 = 12 + 5 = 169 ⇒ 2 = √169 = 13 ⇒ = =
13/2 = 6,5 unidades lineales
Sean A y B dos números naturales de tres cifras cada uno.
Si se escriben las cifras de A a continuación de las cifras de B se obtiene un número de seis
cifras que es igual a 6 veces el número de seis cifras que se obtiene al escribir las cifras de B
a continuación de las de A.
Halla A y B.
SOLUCIÓN
El enunciado dice que = 6 × , escribiendo los nuevos números de seis cifras concatenando los dos de
tres.
Entonces, = 6 × ⇒ 1000 × + = 6 × 1000 × + = 6000 × + 6 × ⇒
⇒ 994 × = 5999 × ⇒ 2 × 7 × 71 × = 7 × 857 ×
÷
⇒ 2 × 71 × = 857 ×
Como 857 es primo, y primo con 2 × 71 = 142, se deduce que
A = 142 y B = 857
Un kiosco vende pastillas de chocolate a 40 céntimos de euro cada una, caramelos a 2
por 10 céntimos y bombones a 10 céntimos cada uno.
Marta gastó 8,8 euros en 44 golosinas que repartió en partes iguales (de cada variedad)
entre sus 4 hijos sin que le sobrase nada.
¿Cuántas golosinas y de qué clases le correspondieron a cada uno?
SOLUCIÓN
Como Marta tiene 4 hijos y reparte equitativamente, en las golosinas de cada hijo gastó
,
= 2,2 euros y le dio
= 11 golosinas.
Llamamos , , a la cantidad respectiva de pastillas de chocolate, caramelos y bombones que le da a cada
uno.
Según el enunciado,
+ + = 11
40 + 5 + 10 = 220
÷ + + = 11
8 + + 2 = 44
ª ª
7 + = 33 ⇒ = 33 − 7
Las posibilidades son:
• = 1 ⇒ = 33 − 7 = 33 − 7 = 26 ⇒ = 11 − − = 11 − 1 − 26 < 0: imposible
• = 2 ⇒ = 33 − 7 = 33 − 14 = 19 ⇒ = 11 − − = 11 − 2 − 19 < 0: imposible
• = 3 ⇒ = 33 − 7 = 33 − 21 = 12 ⇒ = 11 − − = 11 − 3 − 12 < 0: imposible
• = 4 ⇒ = 33 − 7 = 33 − 28 = 5 ⇒ = 11 − − = 11 − 4 − 5 = 2
• = 5 ⇒ = 33 − 7 = 33 − 35 < 0: imposible
• Para valores superiores de obtendríamos siempre valores negativos de
Por tanto, a cada hijo le repartió
4 pastillas de chocolate
2 caramelos
5 bombones
Se dan dos rectángulos iguales: ABCD y APQR, tales que P está en el
interior del rectángulo ABCD y el lado PQ del rectángulo APQR intersecta
al lado DC del rectángulo ABCD en el punto E.
Si se sabe que:
• AB=CD=AP=QR=8
• AD=BC=AR=PQ=12
• DE=1,
halla el área de la figura ABCEP.
SOLUCIÓN
La superficie pedida (rellenada en naranja) será igual ala superficie del
rectángulo menos las superficies de los triángulos rectángulos
y :
Á = Á − Á − Á
En el triángulo rectángulo aplicamos el teorema de Pitágoras:
= + = 12 + 1 = 144 + 1 = 145
En el triángulo rectángulo aplicamos el teorema de Pitágoras:
= + ⇒ 145 = 8 + ⇒ ⇒ = 145 − 64 = 81 ⇒
⇒ = √81 = 9
Entonces, Á = Á − Á − Á ⇒
⇒ Á = × −
×
−
×
= 8 × 12 −
×
−
×!
= 96 − 6 − 36 =
54 unidades cuadradas

Más contenido relacionado

La actualidad más candente

100 problemas maravillosos de matemáticas - Libro 12
100 problemas maravillosos de matemáticas - Libro 12100 problemas maravillosos de matemáticas - Libro 12
100 problemas maravillosos de matemáticas - Libro 12
José Mari Melgarejo Lanero
 
100 problemas maravillosos de matemáticas - Libro 1
100 problemas maravillosos de matemáticas - Libro 1100 problemas maravillosos de matemáticas - Libro 1
100 problemas maravillosos de matemáticas - Libro 1
José Mari Melgarejo Lanero
 
100 problemas maravillosos de matemáticas - Libro 9
100 problemas maravillosos de matemáticas - Libro 9100 problemas maravillosos de matemáticas - Libro 9
100 problemas maravillosos de matemáticas - Libro 9
José Mari Melgarejo Lanero
 
100 problemas maravillosos de matemáticas - Libro 18
100 problemas maravillosos de matemáticas - Libro 18100 problemas maravillosos de matemáticas - Libro 18
100 problemas maravillosos de matemáticas - Libro 18
José Mari Melgarejo Lanero
 
100 problemas maravillosos de matemáticas - Libro 6
100 problemas maravillosos de matemáticas - Libro 6100 problemas maravillosos de matemáticas - Libro 6
100 problemas maravillosos de matemáticas - Libro 6
José Mari Melgarejo Lanero
 
100 problemas maravillosos de matemáticas - Libro 5
100 problemas maravillosos de matemáticas - Libro 5100 problemas maravillosos de matemáticas - Libro 5
100 problemas maravillosos de matemáticas - Libro 5
José Mari Melgarejo Lanero
 
Problemas resueltos de dos ecuaciones con dos incognitas
Problemas resueltos de dos ecuaciones con dos incognitasProblemas resueltos de dos ecuaciones con dos incognitas
Problemas resueltos de dos ecuaciones con dos incognitas
cesar canal mora
 
20 problemas-mcd-mcm
20 problemas-mcd-mcm20 problemas-mcd-mcm
20 problemas-mcd-mcm
I.E RAÑE
 
Problemas resueltos de Inecuaciones Lineales ccesa007
Problemas resueltos de Inecuaciones Lineales  ccesa007Problemas resueltos de Inecuaciones Lineales  ccesa007
Problemas resueltos de Inecuaciones Lineales ccesa007
Demetrio Ccesa Rayme
 
ECUACIONE CON DOS INCOGNITAS
ECUACIONE CON DOS INCOGNITASECUACIONE CON DOS INCOGNITAS
ECUACIONE CON DOS INCOGNITAS
dairocastro1542
 

La actualidad más candente (20)

100 problemas maravillosos de matemáticas - Libro 12
100 problemas maravillosos de matemáticas - Libro 12100 problemas maravillosos de matemáticas - Libro 12
100 problemas maravillosos de matemáticas - Libro 12
 
100 problemas maravillosos de matemáticas - Libro 1
100 problemas maravillosos de matemáticas - Libro 1100 problemas maravillosos de matemáticas - Libro 1
100 problemas maravillosos de matemáticas - Libro 1
 
100 problemas maravillosos de matemáticas - Libro 9
100 problemas maravillosos de matemáticas - Libro 9100 problemas maravillosos de matemáticas - Libro 9
100 problemas maravillosos de matemáticas - Libro 9
 
100 problemas maravillosos de matemáticas - Libro 18
100 problemas maravillosos de matemáticas - Libro 18100 problemas maravillosos de matemáticas - Libro 18
100 problemas maravillosos de matemáticas - Libro 18
 
100 problemas maravillosos de matemáticas - Libro 6
100 problemas maravillosos de matemáticas - Libro 6100 problemas maravillosos de matemáticas - Libro 6
100 problemas maravillosos de matemáticas - Libro 6
 
100 problemas maravillosos de matemáticas - Libro 5
100 problemas maravillosos de matemáticas - Libro 5100 problemas maravillosos de matemáticas - Libro 5
100 problemas maravillosos de matemáticas - Libro 5
 
Problemas resueltos de dos ecuaciones con dos incognitas
Problemas resueltos de dos ecuaciones con dos incognitasProblemas resueltos de dos ecuaciones con dos incognitas
Problemas resueltos de dos ecuaciones con dos incognitas
 
Operaciones combinadas- Potencias y raices
Operaciones combinadas- Potencias y raicesOperaciones combinadas- Potencias y raices
Operaciones combinadas- Potencias y raices
 
Problemas con números naturales
Problemas con números naturalesProblemas con números naturales
Problemas con números naturales
 
Trucos matemáticos
Trucos matemáticosTrucos matemáticos
Trucos matemáticos
 
Conceptos y ejercicios de razones y proporciones
Conceptos y ejercicios de razones y proporcionesConceptos y ejercicios de razones y proporciones
Conceptos y ejercicios de razones y proporciones
 
Modulo
Modulo  Modulo
Modulo
 
Ecuaciones
Ecuaciones Ecuaciones
Ecuaciones
 
1º de bachillerato física
1º de bachillerato física1º de bachillerato física
1º de bachillerato física
 
20 problemas-mcd-mcm
20 problemas-mcd-mcm20 problemas-mcd-mcm
20 problemas-mcd-mcm
 
Problemas resueltos de Inecuaciones Lineales ccesa007
Problemas resueltos de Inecuaciones Lineales  ccesa007Problemas resueltos de Inecuaciones Lineales  ccesa007
Problemas resueltos de Inecuaciones Lineales ccesa007
 
Las fracciones en la vida cotidiana
Las fracciones en la vida cotidianaLas fracciones en la vida cotidiana
Las fracciones en la vida cotidiana
 
ECUACIONE CON DOS INCOGNITAS
ECUACIONE CON DOS INCOGNITASECUACIONE CON DOS INCOGNITAS
ECUACIONE CON DOS INCOGNITAS
 
Guía de matemáticas de 3 grado bloque 2
Guía de matemáticas de 3 grado bloque 2Guía de matemáticas de 3 grado bloque 2
Guía de matemáticas de 3 grado bloque 2
 
02 ecuaciones cuadráticas incompletas
02 ecuaciones cuadráticas incompletas02 ecuaciones cuadráticas incompletas
02 ecuaciones cuadráticas incompletas
 

Similar a 100 problemas maravillosos de matemáticas - Libro 14

100 problemas maravillosos de matemáticas - Libro 4
100 problemas maravillosos de matemáticas - Libro 4100 problemas maravillosos de matemáticas - Libro 4
100 problemas maravillosos de matemáticas - Libro 4
José Mari Melgarejo Lanero
 
100 problemas maravillosos de matemáticas - Libro 3
100 problemas maravillosos de matemáticas - Libro 3100 problemas maravillosos de matemáticas - Libro 3
100 problemas maravillosos de matemáticas - Libro 3
José Mari Melgarejo Lanero
 
100 problemas maravillosos de matemáticas - Libro 7
100 problemas maravillosos de matemáticas - Libro 7100 problemas maravillosos de matemáticas - Libro 7
100 problemas maravillosos de matemáticas - Libro 7
José Mari Melgarejo Lanero
 
10.distribuciones probabilidad
10.distribuciones probabilidad10.distribuciones probabilidad
10.distribuciones probabilidad
Fabián N. F.
 
10.distribuciones probabilidad
10.distribuciones probabilidad10.distribuciones probabilidad
10.distribuciones probabilidad
fanufe
 
Taller de recuperacion de matematica segunda parte ciclo sexto septimo
Taller de recuperacion de matematica  segunda parte ciclo sexto septimoTaller de recuperacion de matematica  segunda parte ciclo sexto septimo
Taller de recuperacion de matematica segunda parte ciclo sexto septimo
Jorge Didier Obando Montoya
 

Similar a 100 problemas maravillosos de matemáticas - Libro 14 (20)

100 problemas maravillosos de matemáticas - Libro 4
100 problemas maravillosos de matemáticas - Libro 4100 problemas maravillosos de matemáticas - Libro 4
100 problemas maravillosos de matemáticas - Libro 4
 
Olimpiada matemática
Olimpiada matemáticaOlimpiada matemática
Olimpiada matemática
 
100 problemas maravillosos de matemáticas - Libro 3
100 problemas maravillosos de matemáticas - Libro 3100 problemas maravillosos de matemáticas - Libro 3
100 problemas maravillosos de matemáticas - Libro 3
 
Congruencia
CongruenciaCongruencia
Congruencia
 
100 problemas maravillosos de matemáticas - Libro 7
100 problemas maravillosos de matemáticas - Libro 7100 problemas maravillosos de matemáticas - Libro 7
100 problemas maravillosos de matemáticas - Libro 7
 
100problemas07 160913091809
100problemas07 160913091809100problemas07 160913091809
100problemas07 160913091809
 
Probl geom con_ecuac_soluc
Probl geom con_ecuac_solucProbl geom con_ecuac_soluc
Probl geom con_ecuac_soluc
 
Sol t10 mec
Sol t10 mecSol t10 mec
Sol t10 mec
 
100problemas09 170819162035
100problemas09 170819162035100problemas09 170819162035
100problemas09 170819162035
 
10.distribuciones probabilidad
10.distribuciones probabilidad10.distribuciones probabilidad
10.distribuciones probabilidad
 
10.distribuciones probabilidad
10.distribuciones probabilidad10.distribuciones probabilidad
10.distribuciones probabilidad
 
Taller de recuperacion de matematica segunda parte ciclo sexto septimo
Taller de recuperacion de matematica  segunda parte ciclo sexto septimoTaller de recuperacion de matematica  segunda parte ciclo sexto septimo
Taller de recuperacion de matematica segunda parte ciclo sexto septimo
 
Mate grado 7°
Mate grado 7°Mate grado 7°
Mate grado 7°
 
The math hatter project. Matemática Cero.
The math hatter project. Matemática Cero.The math hatter project. Matemática Cero.
The math hatter project. Matemática Cero.
 
100 problemas de matemática
100 problemas de matemática100 problemas de matemática
100 problemas de matemática
 
Problemas de ONEM Fase 2
Problemas de ONEM Fase 2Problemas de ONEM Fase 2
Problemas de ONEM Fase 2
 
MATEMÁTICA SEMANA 1
MATEMÁTICA SEMANA 1MATEMÁTICA SEMANA 1
MATEMÁTICA SEMANA 1
 
Problemas 1º, 2º y 3º
Problemas 1º, 2º y 3ºProblemas 1º, 2º y 3º
Problemas 1º, 2º y 3º
 
Tarea semana1 y_2
Tarea semana1 y_2Tarea semana1 y_2
Tarea semana1 y_2
 
Actividad competencias básicas
Actividad competencias básicasActividad competencias básicas
Actividad competencias básicas
 

Último

Ferias de ciencias y estrategia STEAM – PNFCyT 2024.pdf
Ferias de ciencias y estrategia STEAM – PNFCyT 2024.pdfFerias de ciencias y estrategia STEAM – PNFCyT 2024.pdf
Ferias de ciencias y estrategia STEAM – PNFCyT 2024.pdf
JudithRomero51
 
Tema 14. Aplicación de Diagramas 26-05-24.pptx
Tema 14. Aplicación de Diagramas 26-05-24.pptxTema 14. Aplicación de Diagramas 26-05-24.pptx
Tema 14. Aplicación de Diagramas 26-05-24.pptx
Noe Castillo
 
diagnostico final (1). analisis - encuestas
diagnostico final (1). analisis - encuestasdiagnostico final (1). analisis - encuestas
diagnostico final (1). analisis - encuestas
ansomora123
 

Último (20)

6.Deícticos Dos_Enfermería_EspanolAcademico
6.Deícticos Dos_Enfermería_EspanolAcademico6.Deícticos Dos_Enfermería_EspanolAcademico
6.Deícticos Dos_Enfermería_EspanolAcademico
 
📝 Semana 09 - Tema 01: Tarea - Aplicación del resumen como estrategia de fuen...
📝 Semana 09 - Tema 01: Tarea - Aplicación del resumen como estrategia de fuen...📝 Semana 09 - Tema 01: Tarea - Aplicación del resumen como estrategia de fuen...
📝 Semana 09 - Tema 01: Tarea - Aplicación del resumen como estrategia de fuen...
 
Fase 1, Lenguaje algebraico y pensamiento funcional
Fase 1, Lenguaje algebraico y pensamiento funcionalFase 1, Lenguaje algebraico y pensamiento funcional
Fase 1, Lenguaje algebraico y pensamiento funcional
 
ACERTIJO LA RUTA DE LAS ADIVINANZAS OLÍMPICAS. Por JAVIER SOLIS NOYOLA
ACERTIJO LA RUTA DE LAS ADIVINANZAS OLÍMPICAS. Por JAVIER SOLIS NOYOLAACERTIJO LA RUTA DE LAS ADIVINANZAS OLÍMPICAS. Por JAVIER SOLIS NOYOLA
ACERTIJO LA RUTA DE LAS ADIVINANZAS OLÍMPICAS. Por JAVIER SOLIS NOYOLA
 
ACERTIJO DE CARRERA OLÍMPICA DE SUMA DE LABERINTOS. Por JAVIER SOLIS NOYOLA
ACERTIJO DE CARRERA OLÍMPICA DE SUMA DE LABERINTOS. Por JAVIER SOLIS NOYOLAACERTIJO DE CARRERA OLÍMPICA DE SUMA DE LABERINTOS. Por JAVIER SOLIS NOYOLA
ACERTIJO DE CARRERA OLÍMPICA DE SUMA DE LABERINTOS. Por JAVIER SOLIS NOYOLA
 
PROYECTO INTEGRADOR ARCHIDUQUE. presentacion
PROYECTO INTEGRADOR ARCHIDUQUE. presentacionPROYECTO INTEGRADOR ARCHIDUQUE. presentacion
PROYECTO INTEGRADOR ARCHIDUQUE. presentacion
 
Ferias de ciencias y estrategia STEAM – PNFCyT 2024.pdf
Ferias de ciencias y estrategia STEAM – PNFCyT 2024.pdfFerias de ciencias y estrategia STEAM – PNFCyT 2024.pdf
Ferias de ciencias y estrategia STEAM – PNFCyT 2024.pdf
 
Presentación Propuesta de Proyecto Social Colorido y Juvenil Multicolor y Neg...
Presentación Propuesta de Proyecto Social Colorido y Juvenil Multicolor y Neg...Presentación Propuesta de Proyecto Social Colorido y Juvenil Multicolor y Neg...
Presentación Propuesta de Proyecto Social Colorido y Juvenil Multicolor y Neg...
 
Tema 14. Aplicación de Diagramas 26-05-24.pptx
Tema 14. Aplicación de Diagramas 26-05-24.pptxTema 14. Aplicación de Diagramas 26-05-24.pptx
Tema 14. Aplicación de Diagramas 26-05-24.pptx
 
Proyecto Integrador 2024. Archiduque entrevistas
Proyecto Integrador 2024. Archiduque entrevistasProyecto Integrador 2024. Archiduque entrevistas
Proyecto Integrador 2024. Archiduque entrevistas
 
Power Point: Luz desde el santuario.pptx
Power Point: Luz desde el santuario.pptxPower Point: Luz desde el santuario.pptx
Power Point: Luz desde el santuario.pptx
 
Proceso de admisiones en escuelas infantiles de Pamplona
Proceso de admisiones en escuelas infantiles de PamplonaProceso de admisiones en escuelas infantiles de Pamplona
Proceso de admisiones en escuelas infantiles de Pamplona
 
Proyecto integrador Vereda Cujacal Centro.pptx
Proyecto integrador Vereda Cujacal Centro.pptxProyecto integrador Vereda Cujacal Centro.pptx
Proyecto integrador Vereda Cujacal Centro.pptx
 
diagnostico final (1). analisis - encuestas
diagnostico final (1). analisis - encuestasdiagnostico final (1). analisis - encuestas
diagnostico final (1). analisis - encuestas
 
ensayo literario rios profundos jose maria ARGUEDAS
ensayo literario rios profundos jose maria ARGUEDASensayo literario rios profundos jose maria ARGUEDAS
ensayo literario rios profundos jose maria ARGUEDAS
 
Creación WEB. Ideas clave para crear un sitio web
Creación WEB. Ideas clave para crear un sitio webCreación WEB. Ideas clave para crear un sitio web
Creación WEB. Ideas clave para crear un sitio web
 
Cerebelo Anatomía y fisiología Clase presencial
Cerebelo Anatomía y fisiología Clase presencialCerebelo Anatomía y fisiología Clase presencial
Cerebelo Anatomía y fisiología Clase presencial
 
Presentación Pedagoía medieval para exposición en clases
Presentación Pedagoía medieval para exposición en clasesPresentación Pedagoía medieval para exposición en clases
Presentación Pedagoía medieval para exposición en clases
 
PRESENTACION DE LA SEMANA NUMERO 8 EN APLICACIONES DE INTERNET
PRESENTACION DE LA SEMANA NUMERO 8 EN APLICACIONES DE INTERNETPRESENTACION DE LA SEMANA NUMERO 8 EN APLICACIONES DE INTERNET
PRESENTACION DE LA SEMANA NUMERO 8 EN APLICACIONES DE INTERNET
 
ENUNCIADOS CUESTIONARIO S9 GEOLOGIA Y MINERALOGIA - GENERAL.docx
ENUNCIADOS CUESTIONARIO S9 GEOLOGIA Y MINERALOGIA - GENERAL.docxENUNCIADOS CUESTIONARIO S9 GEOLOGIA Y MINERALOGIA - GENERAL.docx
ENUNCIADOS CUESTIONARIO S9 GEOLOGIA Y MINERALOGIA - GENERAL.docx
 

100 problemas maravillosos de matemáticas - Libro 14

  • 2. Ana, Blanca y Carmen tienen entre las tres 490 monedas de 1 euro. Ana gastó la quinta parte de sus monedas, Blanca gastó la tercera parte de sus monedas y Carmen gastó la cuarta parte de sus monedas. Ahora las tres chicas tienen todas igual cantidad de monedas. ¿Cuántas monedas tenía inicialmente cada una? SOLUCIÓN Sean , , la cantidad de monedas que tienen, respectivamente, Ana, Blanca y Carmen. Según el enunciado, + + = 490 = = ⇒ + + = 490 = = ! = ! ⇒ + ! + ! = " #" ! = $ = 490 ⇒ ⇒ = $ × $ ⇒ = 150 ⇒ ( = ! = ! × 150 ⇒ = 180 = ! = ! × 150 ⇒ = 160 Por lo tanto, Ana tenía 150 monedas Blanca tenía 180 monedas Carmen tenía 160 monedas
  • 3. Calcula la superficie del rectángulo de la figura. SOLUCIÓN Nombramos los elementos auxiliares que se ven en la figura adjunta y observamos que hay tres triángulos rectángulos a los que aplicamos el teorema de Pitágoras: + = + 6 = 2 + 2 − = 6 ª ª + + 36 = 4 + 4 − 4 + = 36 ª ª ª ª + 4 − 4 + − + + 36 = 36 − 4 ⇒ 4 − 4 − 36 = 36 − 4 ⇒ ⇒ 8 − 4 − 72 = 0 ÷" 2 − − 18 = 0 ⇒ × 2 − = 18 Y el área del rectángulo es × 2 − = 18 cm2
  • 4. Divide el conjunto de los números naturales desde 1 hasta 100 inclusive en dos conjuntos A y B tales que A contenga 70 números, B contenga 30 números y la suma de todos los números de A sea igual a la suma de todos los números de B. SOLUCIÓN Usamos la fórmula de la suma de los términos de una progresión aritmética para obtener de forma rápida una solución. La suma de los 100 primeros números naturales es × = 5050 Por lo tanto, los dos conjuntos deben sumar cada uno = 2525 La suma de los 70 primeros números naturales es × = 2485 y, como 2525 − 2485 = 40, podemos construir el conjunto A tomando todos los primeros 70 números naturales excepto el 31, al que podemos sustituir por el 71 = 31 + 40 y el conjunto B estará formado por los restantes. A = {1,2,3,…,28,29,30,32,33,34,…,69,70,71} B = {31,72,73,74,...,98,99,100} Nota: evidentemente, ni este reparto es único y ni la forma de distribuir los números es única.
  • 5. Se quieren colocar cerillas sobre algunos de los segmentos punteados en la cuadrícula de la figura. Empezando con la cerilla marcada, se pone cada una a continuación de la anterior y se termina al conectar la última cerilla con la primera. En cada celda se escribe el número de cerillas colocadas alrededor de ella. Algunos de esos números ya están reflejados. ¿Cuál es el mínimo número de cerillas que hay que poner? SOLUCIÓN Se trata de hacer el camino más corto con las condiciones impuestas en algunas casillas. 16 cerillas
  • 6. Sean ABC y ABD dos triángulos unidos por su lado AB. El triángulo ABC tiene ^BAC=90o y AB=2AC. El triángulo ABD tiene ^ADB=90o y AD=BD. El segmento CD corta al segmento AB en O. Calcula BO si se sabe que AC=4. SOLUCIÓN Llamamos = , valor buscado. = 2 = 2 × 4 = 8 y el triángulo es rectángulo e isósceles, por lo que su altura corta a la hipotenusa en su mitad y forma dos triángulos rectángulos y también isósceles: y ⇒ = = = = 4 Observemos que, entonces, los triángulos rectángulos y son iguales por el teorema de Thales ⇒ = = = 2 En conclusión, = = + = 2 + 4 = 6
  • 7. ¿Cuántos números naturales son tales que el producto de sus cifras es 121? SOLUCIÓN Las cifras de un número son los números naturales entre 0 y 9 y no es posible que un producto de algunos ellos (iguales o no) de cómo resultado 11 ni, por supuesto, 11 = 121. Por lo tanto, la respuesta es ninguno
  • 8. ¿Para que valores reales de x se verifica que el resultado de la expresión es un número entero? SOLUCIÓN Si < 0 ⇒ | | = | | = | | = = −2, valor entero. Si = 0 ⇒ | | = | | = | | = , que no es un valor real. Si > 0 ⇒ | | = | | = | | = = 0, valor entero. Por tanto, la expresión dada es un número entero ∀ ∈ ∋ ≠ (para todo número real excepto el cero)
  • 9. ¿Cuál es el menor número de elementos que hay que quitar del conjunto para que el producto de los restantes elementos del conjunto sea un cuadrado perfecto? SOLUCIÓN Multiplicando todos, obtenemos = 10 × 20 × 30 × 40 × 50 × 60 × 70 × 80 × 90 ⇒ ⇒ = 2 × 5 × 2 × 5 × 2 × 3 × 5 × 2 × 5 × 2 × 5 × 2 × 3 × 5 × 2 × 5 × 7 × 2 × 5 × 2 × 3 × 5 ⇒ ⇒ = 2 × 3 × 5 × 7 Para que sea un cuadrado perfecto, todos los factores deben tener un exponente par, por lo que el factor 7 debe desaparecer del producto y también un 2 y un 5 con él para que se quite el número 70 = 2 × 5 × 7 Quedaría entonces = 10 × 20 × 30 × 40 × 50 × 60 × 80 × 90 = 2 × 3 × 5 Ahora sobran un 2 y un 5, por lo que basta eliminar el 10 = 2 × 5 (*) El producto queda = 20 × 30 × 40 × 50 × 60 × 80 × 90 = 2 × 3 × 5 = 2 × 3 × 5 = 720000 Hay que eliminar, del conjunto, 2 números (10 y 70) PostData: Razonando de manera similar (*) podrían sobrar tres 2 y un 5 o un 2, dos 3 y un 5, por lo que, además del 70 se debería eliminar el 40 = 2 × 5 o el 90 = 2 × 3 × 5. Definitivamente, siempre dos números.
  • 10. Calcula la proporción entre las superficies de la parte amarilla y de la parte naranja de la figura adjunta. SOLUCIÓN Tomamos = 1 como medida de referencia, radio del círculo pequeño y mitad del lado del cuadrado grande. El radio del círculo grande , que a su vez es el lado del cuadrado pequeño (amarillo), lo obtenemos a partir del triángulo rectángulo isósceles aplicando el teorema de Pitágoras: = + = 1 + 1 = 2 ⇒ = √2 Entonces, las superficies de los dos cuadrados y de los dos círculos son: Área del cuadrado pequeño (amarillo): Á = √2 = 2 Área del cuadrado grande: Á _ = 2 = 4 Área del círculo pequeño: Á _ = × 1 = Área del círculo grande: Á _ = × √2 = 2 Calculamos ahora la superficie naranja que será la parte que está dentro del cuadrado grande más la parte que está fuera de él: Á = Á _ − Á _ ! + Á " #$%#_&'Á " #()*_& = +4 − , + -'. = 4 − + − 2 = 2 La proporción pedida es Á " )/)%$00) Á " 1)%)12) = = 1
  • 11. En una estación de montaña hay hoteles de 1, 2, 3 y 4 estrellas. Hay más de un hotel de 4 estrellas y sumando el número de estrellas de todos los hoteles de la estación se obtiene 69. Más de la mitad de las estrellas corresponden a hoteles de sólo 1 estrella y el número de hoteles de 3 estrellas es 4 veces el de los que tienen 4 estrellas. ¿Cuántos hoteles de 2 estrellas hay en la estación? SOLUCIÓN Llamamos , , , al número respectivo de hoteles de 1, 2, 3, 4 estrellas. Según el enunciado, observamos que 1. ≥ 2 2. + 2 + 3 + 4 = 69 3. > = 34,5 ⇒ ≥ 35 4. = 4 . ⇒ ≥ 8 Por lo tanto, + 2 + 3 + 4 = 69 . ⇒ + 2 + 3 + = 69 ⇒ 2 + 4 = 69 − . ⇒ 2 + 4 ≤ 69 − 35 ⇒ ⇒ 2 + 4 ≤ 34 ÷ + 2 ≤ 17 ⇒ ≤ 17 − 2 . ⇒ ≤ 17 − 2 × 8 ⇒ ≤ 1 Como hay hoteles de 2 estrellas, la cantidad de ellos es = 1
  • 12. En un barco pirata hay un cofre con monedas de oro. Cinco de los piratas reciben su parte con el siguiente procedimiento: primero Al recibe 1/8 del total y luego Ben recibe 1/6 de lo que queda en el cofre. Más tarde Carl recibe 1/7 de lo que quedaba después de que les dieran a los dos primeros, a continuación Dan recibe 1/5 de lo que queda y, finalmente, a Ed le dan 1/4 de lo que resta. Si hay tres piratas que recibieron igual cantidad de monedas, determina quienes son. SOLUCIÓN Sea la cantidad inicial de monedas de oro que tiene el cofre. • Al recibe y, en el cofre, quedan − = monedas. • Ben recibe × = y, en el cofre, quedan − = − = monedas. • Carl recibe × = y, en el cofre, quedan − = = monedas. • Dan recibe × = y, en el cofre, quedan − = = monedas. • Ed recibe × = y, en el cofre, quedan − = − = monedas. Se observa inmediatamente que monedas reciben Al, Dan y Ed
  • 13. Calcula la proporción entre la superficie azul y la del cuadrado en la figura adjunta. SOLUCIÓN Tomamos = 1 como medida de referencia, lado del cuadrado. Entonces, = Como los triángulos y son semejantes se puede establecer la proporción = ⇒ = = ⇒ = , luego + = ⇒ + = 1 ⇒ = 1 ⇒ = La proporción pedida es Á Á ! " = × 2 = 1 × 2 3 2 1 = 2 3 2 = 1/3
  • 14. Se tienen dos bidones, cada uno de ellos con 100 litros de capacidad, e, inicialmente, contienen entre los dos 100 litros de agua. Se agrega agua al primer bidón hasta completar su capacidad y luego se vierte agua del primer bidón al segundo hasta completar la capacidad del segundo. Finalmente se vierten 12 litros del segundo bidón en el primero. Así resulta que en el segundo bidón hay 10 litros más de agua que en el primero. Determina cuánta agua tenía inicialmente cada bidón. SOLUCIÓN Sean , las cantidades, en litros de agua, que tiene cada bidón inicialmente: + = 100 Al segundo bidón le faltan 100 − litros para tener completa su capacidad. Se llena el primer bidón hasta los 100 litros y se vierten los litros necesarios al segundo bidón para completarlo: ahora tendrá el segundo bidón 100 litros y el primero litros, ya que ha vertido 100 − litros para completar el segundo. Al verter 12 litros del segundo al primero quedarán en el primer bidón + 12 litros y en el segundo bidón 100 − 12 = 88 litros. La última afirmación del enunciado nos dice que 88 = + 12 + 10 ⇒ + 22 = 88 ⇒ = 66 Además, + = 100 ⇒ = 100 − = 100 − 66 = 34 Inicialmente había 34 litros en el primer bidón 66 litros en el segundo bidón
  • 15. La bandera de Lancre es un rectángulo con dimensiones en la proporción 3 : 5. La bandera se divide en cuatro rectángulos de igual área, como se muestra. ¿Cuál es la proporción de las longitudes de los lados del rectángulo blanco? SOLUCIÓN Llamamos , a los lados del rectángulo blanco. 3 será la longitud del lado vertical de la bandera y, por la proporción especificada, 5 es la longitud de su lado horizontal. Al tener todos los rectángulos la misma superficie se verifica que 3 × 5 − = × ⇒ 15 − 3 = ⇒ 15 = 4 ⇒ ⇒ = ⇒ = 4/15
  • 16. Si calcula SOLUCIÓN Es evidente que + + 1 = 0 ⇒ = ±√ ≠ 0 y que + + 1 = 0 ⇒ = −1 − Entonces = × = × −1 − = − − =⏞ − + 1 + ⇒ = 1 Como 1234 = 411 × 3 + 1 ⇒ = × = × = y además + + 1 = 0 ⇒ + 1 = − , se concluye que + = + = = = - 1 Otra forma de resolverlo es usando las dos soluciones complejas = ±√ = ±√ = − + √ − − √ ! ⇒ ⇒ = " cos120° + × sen 120° cos240° + × sen 240° ! Usando la fórmula de Moivre, = * cos 120° + × sen 120° = cos 3 × 120° + × sen 3 × 120° cos 240° + × sen 240° = cos 3 × 240° + × sen 3 × 240° ! ⇒ ⇒ = * cos 3 × 120° + × sen 3 × 120° = cos 360° + × sen 360° cos 3 × 240° + × sen 3 × 240° = cos 720° + × sen 720° ! - .° × /.° 0111111112 = 1 + 0 × = 1 y, a partir de aquí, seguir como en la resolución anterior.
  • 17. Albert y Bernard son compañeros de trabajo de Cheryl y quieren saber cuándo es su cumpleaños. Cheryl les da una lista de diez posibles fechas: 15 de mayo, 16 de mayo, 19 de mayo, 17 de junio, 18 de junio, 14 de julio, 16 de julio, 14 de agosto, 15 de agosto, 17 de agosto. Después, Cheryl les dice por separado a Albert y Bernard, respectivamente, el mes y el día de su cumpleaños. Entonces Albert señala: "No se cuándo es el cumpleaños de Cheryl, pero se que Bernard tampoco lo sabe", a lo que Bernard responde: "Al principio no sabía cuándo era el cumpleaños de Cheryl, pero ahora ya lo se". Albert reflexiona y concluye: "Entonces yo también se cuándo es su cumpleaños". ¿Cuándo es el cumpleaños de Cheryl? SOLUCIÓN Como Albert conoce el mes de cumpleaños y determina que Bernard no puede saber la fecha exacta de nacimiento es imposible que el cumpleaños sea un 18 o un 19, días únicos que propone Cheryl, por lo que a Albert no le ha dicho que su cumpleaños sea en mayo ni en junio. El cumpleaños solo puede ser, entonces, en julio o en agosto: 14 de julio, 16 de julio, 14 de agosto, 15 de agosto o 17 de agosto. Bernard contesta que antes no lo sabía pero ahora sabe cuándo es el cumpleaños, conociendo solo el día. Esto descarta el día 14 porque hay dos posibles: quedan los días 16 de julio, 15 de agosto y 17 de agosto. Finalmente Albert sabe definitivamente, después de todas las declaraciones, cuándo es el cumpleaños sabiendo el mes por lo que debe descartar duplicidades sabiendo el mes de cumpleaños (: ¡hay dos días posibles de agosto!) y determina que el día de cumpleaños de Cheryl es el 16 de julio
  • 18. Sea ABCD un cuadrado de papel de lados AB = BC = CD = DA =10 El cuadrado se dobla a lo largo de una línea recta, haciendo coincidir el vértice A con el punto medio del lado BC Esta línea recta corta al lado AB en E y al lado CD en F Calcula la medida de EF SOLUCIÓN Según el esquema de la figura el ángulo es tal que tan = = = Por construcción, = 90° − ⇒ cot = tan ⇒ cot = Como sen = !" # ⇒ sen = !" $% = & ' = ( ⇒ sen = √ En el triángulo rectángulo * , sen * = sen ⇒ +% $% = √ ⇒ $% = √ ⇒ ⇒ = ×√ = 5×√5
  • 19. Un cuadrado tiene dos de sus vértices en una semicircunferencia y los otros dos en el diámetro de la misma, como se muestra en la figura. El radio de la circunferencia es de 1 cm. ¿Cuál es el área del cuadrado? SOLUCIÓN Llamamos a la mitad del lado del cuadrado. Según el triángulo rectángulo que se construye y aplicando en él el teorema de Pitágoras, + 2 = 1 ⇒ + 4 = 1 ⇒ = El área del cuadrado es 2 = 4 = 4 × = 4/5 cm2
  • 20. Sean ABC un triángulo isósceles con AB = AC y D el punto medio del lado BC. La perpendicular a AC, trazada por D, corta al lado AC en E. Sea F en AB tal que EF es paralela a BC. Si BC =12 y CE = 4, calcula la medida del segmento EF. SOLUCIÓN Fijándonos en el esquema de la derecha, tenemos, por construcción, el triángulo rectángulo donde la hipotenusa es = = = 6 y el cateto = 4 Por el teorema del cateto, × = ⇒ = = = = De lo anterior = − = 6 − = ⇒ = 2 × = 2 × = 20/3
  • 21. Un gráfico consta de 16 vértices y algunos segmentos que los conectan, como en la imagen. Hay un gusano en el vértice A y, en cada movimiento, puede moverse desde un vértice a cualquier vértice vecino a lo largo de un segmento de conexión. ¿En cuál o cuales de los vértices P, Q, R, S, T puede estar el gusano después de 2019 movimientos? SOLUCIÓN El gusano alcanza, desde , los vértices , , en una cantidad par de movimientos y los vértices , en una cantidad impar. Siempre. Por tanto, después de 2019 movimientos el gusano puede estar en uno de estos vértices: Q – S
  • 22. Una librería ofrece cuadernos a 5 euros y realiza los siguientes descuentos: • en una compra de hasta 35 cuadernos inclusive hace un descuento del 5% • si se compran entre 36 y 55 cuadernos inclusive el descuento es del 12% • si se compran 56 o más cuadernos, el descuento es del 20% Pablo compró con un descuento del 5% y al día siguiente compró nuevamente, esta vez con un descuento del 12%. Si Pablo hubiese comprado los cuadernos todos juntos, en una sola compra, le hubiese correspondido un descuento del 20% y habría gastado 39 euros menos de lo que gastó. Determina cuántos cuadernos compró cada día. SOLUCIÓN Sean , la cantidad de cuadernos que compró, respectivamente, el primer y el segundo día. Es evidente que 1 ≤ ≤ 35; 36 ≤ ≤ 55; 56 ≤ + ⇒ 56 ≤ + ≤ 90 Según el enunciado, 5 × + 5 × = 5 × + × + 39 × 95 + 88 = 80 + 80 + 780 ⇒ ⇒ 15 + 8 = 780 ⇒ = ⇒ = 52 − , con 36 ≤ ≤ 55 Como = 52 − es un valor natural ⇒ es múltiplo de 15 ⇒ = 45 ⇒ = 52 − ×# = 52 − 24 ⇒ = 28 El primer día compró 28 cuadernos El segundo día compró 45 cuadernos
  • 23. Los números naturales a, b y c tienen cada uno tres cifras, y en cada número la primera cifra es la misma que la última. Si b=2a+1 y c=2b+1, ¿cuántos valores distintos hay para a? SOLUCIÓN Sea = ⇒ = 100 + 10 + = 101 + 10 . Se deduce entonces que • = 2 + 1 = 2 × 101 + 10 + 1 = 202 + 20 + 1 • = 2 + 1 = 2 × 202 + 20 + 1 + 1 = 404 + 40 + 3 Como es de tres cifras, las posibilidades son: • = 1 ⇒ = 404 + 40 + 3 = 40 + 403 ⇒ ⇒ = 8 ⇒ = 40 × 8 + 403 ⇒ = 727 ⇒ = 363 ⇒ = 181 = 9 ⇒ = 40 × 9 + 403 ⇒ = 767 ⇒ = 383 ⇒ = 191 • = 2 ⇒ = 808 + 40 + 3 = 40 + 811, y es imposible que sea capicúa de tres cifras. Hay, para , dos valores distintos: 181, 191
  • 24. Sea N=8+98+998+9998+…..+999…(100 veces 9)…98 el resultado de la suma de 101 números que tienen el último dígito 8 y los demás dígitos 9: desde el 8, que tiene cero nueves, hasta el que tiene 100 dígitos nueve. Calcula la suma de los dígitos de N. SOLUCIÓN = 8 + 98 + 998 + +9998 + ⋯ + 99999. . .9999 8 = 10 − 2 + 10 − 2 + + 10 − 2 + ⋯ + 10 − 2 Entonces, = 10 + 10 + 10 + ⋯ + 10 − 2 × 101 = × ! ! − 202, usando la fórmula "# = $× ! ! de la suma de los % primeros términos de una progresión geométrica de razón &. = × ! ! − 202 = ×' ! ( − 202 = × ... )*+,-. / − 202 = 11111. . .111 0 − 202 ⇒ = 11111. . .111 1 0908, por lo que la suma de sus cifras es 1 + 1 + 1+. . . +1 + 1 + 1 1 + 0 + 9 + 0 + 8 = 98 × 1 + 9 + 8 = 115
  • 25. Si se elimina cualquier cifra de un número de 4 cifras dado, el número de 3 cifras resultante es un divisor del número original. ¿Cuántos números de 4 cifras tienen esta propiedad? SOLUCIÓN Sea el número de cuatro cifras = tal que, según se deduce del enunciado, ≠ 0, ≠ 0, y sea el número de tres cifras Entonces, ∃ ∈ ∋ × = ⇒ × = 10 × + ⇒ − 10 × = !" #$$$$$$$$$$$$% = 0 De lo anterior = 0, y sea el número de tres cifras = 0 = 10 × También, ∃ℎ ∈ ∋ ℎ × 0 = ⇒ ℎ × 10 × = 100 × + 10 ⇒ ℎ × = 10 × + ⇒ ⇒ ℎ − 10 × = ' !" #$$$$$$$$$$% = 0 Por lo tanto = 00 y, por fin, tomando los números 00 = 100 y 00 = 100 se sigue por hipótesis que • ∃( ∈ ∋ ( × 00 = ⇒ ( × 100 = 1000 + 100 ⇒ ( = 10 + ⇒ ( − 10 × = • ∃) ∈ ∋ ) × 00 = ⇒ ) × 100 = 1000 + 100 ⇒ ) = 10 + ⇒ ) − 1 × = 10 De la combinación de las igualdades se deduce que ) − 1 × ( − 10 × = 10 ⇒ ) − 1 × ( − 10 = 10 Posibilidades: • ) − 1 = 1 ⇒ = 10 . No tiene sentido por ser cifras y • ) − 1 = 2 ⇒ 2 = 10 ⇒ = 1; = 5 … … … … … … … … … … . … . . = 1500 • ) − 1 = 5 ⇒ 5 = 10 ⇒ / = 1; = 2 … … … … … … … … … … … . = 1200 = 2; = 4 … … … … … … … … … … … . = 2400 = 3; = 6 … … … … … … … … … … … . = 3600 = 4; = 8 … … … … … … … … … … … . = 4800 4 • ) − 1 = 10 ⇒ 10 = 10 ⇒ = ⇒ ⎩ ⎪ ⎪ ⎪ ⎨ ⎪ ⎪ ⎪ ⎧ = 1; = 1 … … … … … … = 1100 = 2; = 2 … … … … … … = 2200 = 3; = 3 … … … … … … = 3300 = 4; = 4 … … … … … … = 4400 = 5; = 5 … … … … … … = 5500 = 6; = 6 … … … … … … = 6600 = 7; = 7 … … … … … … = 7700 = 8; = 8 … … … … … … = 8800 = 9; = 9 … … … … … … = 9900 4 En total, 14 números
  • 26. Sea ABC un triángulo tal que ^A=45o y ^C=30o y sea D el punto medio del lado BC. Calcula la medida del ángulo ^CAD. SOLUCIÓN Buscamos el ángulo = sabiendo que = 45° y = 30° ⇒ = 180° − − ⇒ ⇒ = 105° y, además, = = Aplicamos el teorema de los senos a los triángulos y : = ° ! = " # $ª &ª ⇒ ° ! = " ⇒ °×() () °× = *+ ° ,+° ⇒ ⇒ √. . × cot − 1! = °23+°! ,+° = °×() 3+°2() °× 3+° ,+° = √. . × & $ 2 √4 $ & $ ⇒ cot − 1 = 1 + √3 ⇒ ⇒ cot = 2 + √3 ⇒ tg = * .2√, = . √, 8.2√,9×8. √,9 ⇒ tg = 2 − √3 ⇒ = arctg 82 − √39 = 15o
  • 27. Un contenedor que tiene forma de caja con caras rectangulares está parcialmente lleno con 120 m3 de agua. La profundidad del agua es de 2 m, 3 m ó 5 m, según la cara de la caja que se encuentre en el suelo, como se muestra en la figura. ¿Cuál es el volumen del contenedor? SOLUCIÓN Sean , , las longitudes de los lados de la caja correspondientes a las medidas parciales de 2, 3, 5 m El volumen de la caja es y el enunciado indica que 2 = 120 3 = 120 5 = 120 ª× ª× ª 2 × 3 × 5 = 120 × 120 × 120 ⇒ 30 = 120 ⇒ = ⇒ ⇒ = = 120 × = 120 × √4 = 120 × 2 = 240 m3
  • 28. Sobre la mesa hay 21 cartas, una con cada uno de los números naturales de 1 hasta 21 ambos inclusive. Miguel selecciona 4 cartas y se las muestra a Marga. Luego Marga le quita a Miguel una carta, la que ella quiera. Si la suma de los números de las 3 cartas con las que se quedó Miguel es múltiplo de 3, gana Marga y si no, gana Miguel. Determina de cuántas maneras puede Miguel elegir las 4 cartas para estar seguro de ganar, no importando lo bien que juegue Marga y teniendo en cuenta que dos elecciones de las mismas 4 cartas, pero en distinto orden, se consideran la misma elección. SOLUCIÓN Los números de las cuatro cartas deben ser tales que, sumados tres a tres, el resultado no sea múltiplo de 3 Si ordenamos los números según el resto que se obtiene al dividirlos por 3 obtenemos esta clasificación: • 3 , que son los números 3, 6, 9, 12, 15, 18, 21. Total, 7 números • 3 + 1, que son los números 1, 4, 7, 10, 13, 16, 19. Total, 7 números • 3 + 2, que son los números 2, 5, 8, 11, 14, 17, 20. Total, 7 números En el conjunto de las 4 cartas no puede haber 3 o más cartas pertenecientes a al mismo grupo de los anteriormente citados pues desechando una de ellas (en caso de cartas de dos grupos, la de distinto grupo a las otras 3) las otras suman un múltiplo de 3 En el conjunto de las 4 cartas no pueden estar representados los tres grupos, pues le bastaría a Marga dejar una de cada grupo para que su suma fuese múltiplo de 3: 3 + 3 + 1 + 3 + 2 = 3 × + + + 1 En resumen, las posibilidades de una victoria segura de Miguel es que seleccione 2 de un grupo y 2 de otro: cualquier carta que quite Marga dejará una suma de dos cartas de un grupo y una tercera de otro grupo cuya suma es imposible que sea múltiplo de 3. Es cuestión, entonces, de formar conjuntos de dos cartas de cada grupo de 7 y, dichos conjuntos, combinarlos de dos en dos, con cada uno de cada uno de los 3 grupos. Las maneras de elección de las cuatro cartas para una victoria segura de Miguel son, entonces, × × = 7! 2! × 5! × 7! 2! × 5! × 3! 2! × 1! = 7 × 6 2 × 7 × 6 2 × 3 = 1323
  • 29. Tres amigos, Alex, Guille y Marc, salen a caminar todos los días. Si Alex no lleva sombrero, entonces Guille sí. Si Guille no lleva sombrero, entonces Marc sí. Hoy Marc no lleva sombrero. ¿Quién lleva sombrero hoy con toda seguridad? SOLUCIÓN La afirmación " ⇒ í " [ ] equivale a la afirmación " ⇒ í " [ ′] La afirmación " ⇒ í " [ ] equivale a la afirmación " ⇒ í " [ ′] Entonces, como hoy ⇒ í , y no puede deducirse ninguna conclusión más. Hoy lleva sombrero Guille
  • 30. De un cuadrado de papel de lado 1 hay que recortar dos triángulos equiláteros iguales. Halla el máximo valor posible del lado de los triángulos. SOLUCIÓN Sea la medida del lado de los triángulos equiláteros y revisamos las dos maneras de conseguir los mayores triángulos equiláteros. Según se aprecia en la figura de la derecha, esta es una forma de elegir los dos triángulos equiláteros iguales de mayor lado dentro del cuadrado de lado 1. Observamos, en el triángulo rectángulo que = 1, = − 1 − = 2 − 1 y = 60° ⇒ tg = ⇒ tg 60° = ⇒ = √3 ⇒ 2 − 1 = √ = √ ⇒ = × 1 + √ ! = "√ # = 0,788675 Otra forma de elegir los dos triángulos equiláteros iguales de mayor lado dentro del cuadrado de lado 1 es la que se observa en la figura de la derecha. En este caso, aplicando el teorema de Pitágoras en el triángulo rectángulo que es la mitad del equilátero, la altura del triángulo equilátero es ℎ = ( − ! = ( ) * ⇒ ⇒ ℎ = ×√ Ahora bien, ℎ es la mitad de la diagonal del cuadrado: ℎ = √ )" ) = √ por lo que ×√ = √ ⇒ = √ √ ⇒ ⇒ = √# = 0,816497 En resumen, mayor valor posible del lado corresponde a la segunda opción y es √- . = 0,816497
  • 31. Un número natural n se llama 6-bueno si su divisor más grande, excluyendo n, es igual a n–6. ¿Cuántos números 6-buenos hay? SOLUCIÓN − 6 el divisor más grande del número natural . Se cumple entonces que = − 6 × , siendo el número natural ≤ − 6. De la expresión anterior, = ⇒ = 1 + Posibilidades: • − 6 = 1 ⇒ = 7 y = 7 … 7 = 1 × 7. Como se excluye contándolo como divisor para este problema, = 7 cumple la condición. • − 6 = 2 ⇒ = 8 y = 4 … 8 = 2 × 4. Existe un divisor de = 8 mayor que − 6: > − 6. • − 6 = 3 ⇒ = 9 y = 3 … 9 = 3 × 3, por lo que = 9 cumple la condición. • − 6 = 6 ⇒ = 12 y = 2 … 12 = 6 × 2, por lo que = 12 cumple la condición. Los números 6-buenos son 7, 9, 12, luego son 3
  • 32. La figura muestra un cubo desplegado. Escribe en cada cara del cubo un número entero del 1 al 6 sin repetir sabiendo que, al armar nuevamente el cubo, si para cada vértice se calcula la multiplicación de los números de las tres caras que concurren en ese vértice, se obtienen, en algún orden, los números 10, 12, 20, 24, 30, 36, 60 y 72 y teniendo en cuenta que el 1 ya está colocado. SOLUCIÓN Según los números obtenidos en los vértices, las únicas descomposiciones posibles en productos de 3 dígitos de 1 a 6 de los valores extremos son 10 = 1 × 2 × 5 y 72 = 3 × 4 × 6, por lo que los números de ambos productos estarán en caras opuestas dos a dos. En principio, podemos considerar que 1, 2, 5 confluyen en un vértice, como se ve en la figura de la derecha. Está claro que la única cara que combinará también con las 1 y 2 deberá ser la 6 para construir el 12 = 1 × 2 × 6, por lo que ésta última cara se opondrá a la 5. Además, la otra cara que combinará con las 1 y 5 deberá ser la 4 para construir el 20 = 1 × 4 × 5, por lo que ésta última cara se opondrá a la 2. La cara restante es la 3, que se opondrá a la cara 1, quedando así el desarrollo del cubo:
  • 33. Una caja contiene 4 bombones y 1 caramelo. Diego y Ángel sacan por turnos una golosina de la caja y se la comen. Quien saque el caramelo gana. Si Diego extrae primero, ¿cuál es la probabilidad de que gane Ángel? SOLUCIÓN Ángel únicamente puede ganar al efectuar la segunda o la cuarta extracción. La probabilidad de que Ángel saque el caramelo en la segunda extracción se calcula teniendo en cuenta que en la primera extracción Diego no saca el caramelo: = × = La probabilidad de que Ángel saque el caramelo en la cuarta extracción se calcula teniendo en cuenta que en la primera y tercera extracciones Diego no saca el caramelo ni Ángel en la segunda: = × × × = Por tanto, la probabilidad pedida es + = + = 2/5
  • 34. Jon hizo la lista de los números enteros positivos de cuatro dígitos ABBC que son múltiplos de 9 y tales que A, B, C son dígitos distintos con B=A+C. Calcula cuántos números tiene la lista de Jon. SOLUCIÓN = 9 ⇒ + + + = + 2 + = 9 3 = 9 ⇒ = 3 , siendo un número natural porque = + ≠ 0 ya que , , son cifras distintas entre sí. Como es una cifra y 0 ≤ , < , las posibilidades son: • = 1 ⇒ = 3 ⇒ = 1, = 2 … … … 1332 = 2, = 1 … … … 2331 • = 2 ⇒ = 6 ⇒ = 1, = 5 … … … 1665 = 2, = 4 … … … 2664 = 4, = 2 … … … 4662 = 5, = 1 … … … 5661 • = 3 ⇒ = 9 ⇒ ⎩ ⎪ ⎪ ⎨ ⎪ ⎪ ⎧ = 1, = 8 … … … 1998 = 2, = 7 … … … 2997 = 3, = 6 … … … 3996 = 4, = 5 … … … 4995 = 5, = 4 … … … 5994 = 6, = 3 … … … 6993 = 7, = 2 … … … 7992 = 8, = 1 … … … 8991 Total, 2 + 4 + 8 = 14 números
  • 35. Se muestran dos cuadrados adyacentes con longitudes de lados a y b (a<b). ¿Cuál es el área del triángulo sombreado? SOLUCIÓN Viendo el esquema representado en la figura la superficie pedida es igual a = Á − Á − Á − Á = × 2 − × 2 − × 2 − × = + × 2 − × 2 − × − 2 − × − = + − − + − 2 + 2 2 = a2 /2 unidades cuadradas
  • 36. En una reunión cada invitado saludó a cada uno de los restantes con un apretón de manos. Hubo 36 apretones de manos entre dos mujeres y 28 apretones de manos entre dos varones. Calcula cuántos apretones de manos hubo entre un varón y una mujer. SOLUCIÓN Llamamos ℎ al número de hombres de la reunión. Los apretones de mano entre ellos fueron , = ! !× ! ⇒ ⇒ × = 28 ⇒ ℎ − ℎ − 56 = 0 ⇒ ℎ = 8: hay 8 hombres. Llamamos al número de mujeres de la reunión. Los apretones de mano entre ellas fueron , = ! !× ! ⇒ ⇒ × = 36 ⇒ − − 72 = 0 ⇒ = 9: hay 9 mujeres. Los apretones de manos entre hombres y mujeres fueron 8 × 9 = 72
  • 37. Para calcular (a+b)/c, Laura teclea en la calculadora a+b/c y el resultado es 11, siendo a, b y c enteros positivos. Luego teclea b+a/c y se sorprende al ver que el resultado es 14. Se da cuenta de que la calculadora está diseñada para calcular las divisiones antes que las sumas. ¿Cuál es el resultado correcto de (a+b)/c ? SOLUCIÓN + = 11 ⇒ = 11 − + = 14 ⇒ = 14 − ª ª = 25 − + ⇒ + = 25 − × + ⇒ + = Como , , son enteros positivos, de la expresión anterior se deduce que • + 1 = 5 ⇒ = 4 ⇒ + = 11 ⇒ + = 11 + = = 20 ª ª + = 11 ! = 9 ⇒ # = 8 = 12 • + 1 = 25 ⇒ = 24 ⇒ % + = 11 ⇒ + = 11 + = 24 ª ª + = 11 ! = 13 , imposible porque no sería un valor entero. En resumen, = 8, = 12, = 4 ⇒ = ' = 5
  • 38. El número n tiene 100 cifras, todas iguales a 9. Calcula la suma de las cifras de n2 SOLUCIÓN = 999 … … .9 = 10 − 1 ⇒ = 10 − 1 = 100 − 2 × 10 + 1 ⇒ ⇒ = 1 000 … 0 − 2 000 … 0 + 1 = 999 … 9 8 000 … 0 + 1 ⇒ = 999 … 9 8 000 … 0 1 luego la suma de sus cifras es 9 + 9 + 9 + ⋯ + 9 + 8 + 0 + 0 + 0 + ⋯ + 0 + 1 = 99 × 9 + 8 + 1 = 900
  • 39. ¿Para qué valores enteros n el número es primo? SOLUCIÓN Si − 2 − 3 = 0 ⇒ = ± × = ± = −1 3 ⇒ − 2 − 3 = + 1 × − 3 La expresión será un número primo si uno de los dos factores anteriores es igual a ±1 y el otro factores es, en valor absoluto, primo. Posibilidades: • + 1 = −1 ⇒ = −2 − 3 = −5 ⇒ | − 2 − 3| = | −1 × −5 | = |5| = 5, número primo • + 1 = 1 ⇒ = 0 − 3 = −3 ⇒ | − 2 − 3| = |1 × −3 | = |−3| = 3, número primo • − 3 = −1 ⇒ = 2 + 1 = 3 ⇒ | − 2 − 3| = |3 × −1 | = |−3| = 3, número primo • − 3 = 1 ⇒ = 4 + 1 = 5 ⇒ | − 2 − 3| = |5 × 1| = |5| = 5, número primo En todos los casos se obtienen números primos, por lo que los valores pedidos son cuatro: -2, 0, 2, 4
  • 40. Sea ABCD un rectángulo con BC < CD y M, N los puntos medios de los lados BC y CD respectivamente. Este rectángulo es tal que el triángulo AMN es rectángulo con ^MNA = 90o . Si BC = 5, calcula la medida de CD. SOLUCIÓN Llamamos = ⇒ = = . Como = = 5 ⇒ = Observamos que = 90° − = 90° − 180° − 90° − ⇒ = y, por tanto los triángulos rectángulos y son semejantes. Establecemos, entre ellos, la relación de semejanza = ⇒ = ⇒ ⇒ ! = 5 × ⇒ # = ⇒ = 50 ⇒ = √50 = 5×√2 ≈ 7,07
  • 41. Una poligonal DEFB, con DE ⊥ ⊥EF y EF FB, se encuentra dentro del cuadrado ABCD como se muestra en la figura. Como DE = 5, EF = 1 y FB = 2, ¿cuál es la longitud del lado del cuadrado? SOLUCIÓN Trazamos la diagonal al cuadrado , que corta al segmento en Llamamos = ⇒ 1 − = Como los segmentos y son paralelos ⇒ = , por lo que los triángulos rectángulos y son semejantes. Establecemos, entre ellos, la razón de semejanza = ⇒ = ⇒ 5 = 2 − 2 ⇒ ⇒ 7 = 2 ⇒ = y 1 − = Por lo tanto, la diagonal del cuadrado es = + = √ + + √ + ⇒ ⇒ = 2 + + 5 + = 2 × 1 + ! + 5 × 1 + ! = 2 × " ! + 5 × " ! ⇒ ⇒ = 7 × " ! = × √50 ⇒ = √50, habiendo aplicado el teorema de Pitágoras en los triángulos rectángulos citados. Volviendo a aplicar el mismo teorema en el triángulo rectángulo $ , $ + $ = %&% '((() 2 × $ = 50, de donde ⇒ $ = 25 ⇒ $ = 5
  • 42. Un biólogo que estudia una colonia de aves migratorias hizo las siguientes observaciones a lo largo de un día: • A mediodía se fueron 30 machos que ya no regresaron, y quedaron en la colonia 2 hembras por cada macho. • A la tarde se fueron 90 hembras, que ya no regresaron, y quedaron en la colonia 3 machos por cada hembra. Determina cuántas aves tenía la colonia antes del mediodía. SOLUCIÓN Sean , el número respectivo de aves macho y aves hembra que tiene la colonia antes del mediodía. Según las dos afirmaciones del enunciado, se cumple que 2 × − 30 = − 30 = 3 × − 90 ⇒ 2 − 60 = − 30 = 3 − 270 ⇒ ⇒ 2 − = 60 3 − = 240 ª × ª 2 − = 60 5 = 540 ⇒ = = ⇒ = 84 = 108 ⇒ + = 84 + 108 = 192 aves
  • 43. Se seleccionan al azar tres números diferentes del conjunto {1, 2, 3, ..., 10}. ¿Cuál es la probabilidad de que uno de ellos sea la media aritmética de los otros dos? SOLUCIÓN Para que uno de los elegidos sea la media aritmética de los otros dos, estos deben ser de la misma paridad. La probabilidad de que se elijan dos números, de los diez, de la misma paridad es 1 × = y que el tercero sea la medía aritmética de anteriores (que siempre se encontrará entre los diez y será distinto de los dos primeros) es × = Como la media artimética puede ser, también, el primero o el segundo de los números elegidos, la probabilidad pedida es 3 × = 1/6
  • 44. Si se agregan a la derecha de 2019 tres dígitos a, b, c, el número de siete dígitos 2019abc es divisible por 541. Halla todos los posibles valores de los dígitos a, b, c. SOLUCIÓN 2019 = 2019000 + = 541 Como 2019000 = 3731 × 541 + 529 ⇒ 3731 × 541 + 529 + = 541 ⇒ 529 + = 541 Entonces, las posibilidades son: • 529 + = 541 × 1 = 541 ⇒ = 541 − 529 = 12 ⇒ = 0, = 1, = 2 • 529 + = 541 × 2 = 1082 ⇒ = 1082 − 529 = 553 ⇒ = 5, = 5, = 3 • 529 + = 541 × 3 = 1623 ⇒ = 1623 − 529 = 1094, imposible. Los posibles valores son a = 0, b = 1, c = 2 a = 5, b = 5, c = 3
  • 45. La sucesión a1, a2, a3, … comienza con a1=49 Para an+1, el término se obtiene añadiendo 1 a la suma de las cifras de an y elevando al cuadrado el resultado. Por ejemplo, a2=(4+9+1)2 =196 Calcula el valor de a2019 SOLUCIÓN = 49 = 4 + 9 + 1 = 14 = 196 = 1 + 9 + 6 + 1 = 17 = 289 = 2 + 8 + 9 + 1 = 20 = 400 = 4 + 0 + 0 + 1 = 5 = 25 = 2 + 5 + 1 = 8 = 64 = 6 + 4 + 1 = 11 = 121 = 1 + 2 + 1 + 1 = 5 = 25 = 2 + 5 + 1 = 8 = 64 = 6 + 4 + 1 = 11 = 121 ………… ………… ………… ………… ………… ………… ………… Como puede observarse, a partir del quinto término de la sucesión los valores se repiten cada tres términos, de manera que = 25; = 64; = 121, ∀ ∈ ! Como 2019 = 3 × 673 = 3 × 672 + 3 ⇒ = 64
  • 46. Paco y Joaquín se reparten una bolsa de caramelos con el siguiente procedimiento: Paco saca uno, Joaquín saca dos, Paco saca tres, Joaquín saca cuatro, y así cada uno, en su turno, saca uno más que los que acaba de sacar el otro. Cuando uno de los dos se encuentra con que no quedan suficientes caramelos para sacar uno más que los que sacó el otro se lleva todo lo que queda y concluye el reparto. Si Paco sacó en total 2000 caramelos, ¿cuántos caramelos había inicialmente en la bolsa? SOLUCIÓN Es evidente que en la tanda número , Paco se lleva 2 − 1 caramelos y Joaquín 2 caramelos. Si la última llevada (los restos) es de Joaquín, Paco se llevaría tandas completas de caramelos, por lo que el total sería 1 + 3 + 5 + 7 + ⋯ + 2 − 1 = × = = 2000, lo cual es imposible pues es una cantidad natural. Por lo tanto, Paco es el que se lleva la última cantidad < 2 + 2 una vez que ha recibido tandas completas. Es decir, + = 2000 y es el número cuyo cuadrado es más proximo a 2000 y menor que ese valor. Como 44 < √2000 < 45 ⇒ = 44 Paco y Joaquín reciben 44 tandas completas de caramelos y el resto son = 2000 − = 2000 − 44 = 64 caramelos recibidos por Joaquín al inicio de la tanda 45. El total de caramelos a repartir es 1 + 3 + 5 + ⋯ + 2 − 1 + 2 + 4 + 6 + ⋯ + 2 + 64 = = 1 + 2 + 3 + ⋯ + 2 + 64 =⏞ 1 + 2 + 3 + ⋯ + 88 + 64 = × + 64 = 89 × 44 + 64 = 3980 caramelos (Paco se lleva 2000 y Joaquín 1980)
  • 47. El cuadrado de la figura se rellena con los números 1, 2, 3, 4 y 5 de tal manera que cada fila y cada columna contienen cada uno de ellos exactamente una vez. Además, la suma de los números en cada una de las tres regiones con bordes en negrita es igual. ¿Qué número está en la esquina superior derecha? SOLUCIÓN Es un cuadro 5 × 5 que tiene, en cada fila o columna, los números de 1 a 5, por lo que la suma de todos ellos es 1 + 2 + 3 + 4 + 5 × 5 = 15 × 5 = 75. La zona inferior derecha tiene ya colocado un 2: las cinco casillas restantes sumarán 23. Como no puede haber más de tres números iguales entre ellos (debido a la cantidad de columnas y/o filas vacías), la única distribución posible será la que se ve en el margen derecho. En el espacio central la suma de las casillas también es 25 y las casillas vacías deben contener los valores que se ven en la imagen de la izquierda, al estar colocados los restantes en las respectivas filas y/o columnas. Las tres casillas que quedan suman 25 − 2 × 1 + 3 − 2 × 1 + 2 + 3 = 5, por lo que dichas casillas deben tomar los valores 1, 2, 2, quedndo como se ve en la imagen derecha. En resumen, y mediante una deducción elemental se obtiene la distribución de valores en las casillas del espacio central intermedio: A partir de aquí, y con las condiciones establecidas, es muy sencillo completar el cuadro hasta llegar a que permite decir que el número pedido es 3
  • 48. Sea ABCD un trapecio de bases AB y CD , y lados no paralelos BC y DA , con ^BAD=^ADC=90o . La perpendicular a la diagonal AC trazada desde B corta a AC en E. Si AB=125, AE=35 y CE=50, calcular el área del trapecio ABCD. SOLUCIÓN Señalamos en la imagen los valores y alguna línea y punto auxiliar. Sea = = ℎ y = = ⇒ = 125 − Además, = + = 35 + 50 = 85 En el triángulo rectángulo aplicamos el teorema de Pitágoras: = − = 125 − 35 = 14400 Ahora aplicamos el mismo teorema en el triángulo rectángulo : = + = 14400 + 50 = 16900 Calculamos y ℎ usando el teorema de Pitágoras en los triángulos rectángulos y : + = + = ⇒ ℎ + = 85 ℎ + 125 − = 16900 ⇒ ℎ + = 7225 ℎ + 15625 − 250 + = 16900 ⇒ ⇒ ℎ + = 7225 15625 − 250 + 7225 = 16900 ⇒ ℎ + = 7225 250 = 5950 ⇒ ℎ = 7225 − = 6658,56 ⇒ ℎ = 81,6 = ! ! = 23,8 La superficie del trapecio es "#$%& × = ( $) × ℎ = ( $ *,+ × 81,6 = 6071,04 unidades cuadradas
  • 49. Dos triángulos isósceles cuyos lados miden x, x, 6 y x, x, 8, respectivamente, tienen igual área. Halla x. SOLUCIÓN Llamando , a las respectivas alturas de ambos triángulos isósceles, se verifica, por el teorema de Pitágoras, que ⎩ ⎨ ⎧ = − = − ⇒ = √ − 9 = √ − 16 á !!!!" ×√$%&' = ×√$%&( ⇒ ⇒ 3 × √ − 9 = 4 × √ − 16 ⇒ 9 × + − 9, = 16 × + − 16, ⇒ 9 − 81 = 16 − 256 ⇒ 7 = 175 ⇒ ⇒ = (12 1 = 25 ⇒ = 5
  • 50. Ángela tiene un tablero de 115×7, o sea, de 115 filas con 7 casillas cada una y debe colocar fichas en las casillas del tablero siguiendo las siguientes reglas: • En cada casilla puede colocar una sola ficha. • No pueden quedar dos filas idénticas, es decir, no puede haber dos filas que tengan las mismas casillas ocupadas y las mismas casillas vacías. Calcula la máxima cantidad de fichas que puede colocar Ángela en su tablero. SOLUCIÓN Representamos como a la casilla rellenada con una ficha y como a la casilla vacía. La cantidad de posibilidades distintas de rellenado de filas es el número de variaciones con repetición de dos elementos ( y ) tomados de 7 en 7: , , … serían algunas de estas variaciones. La cantidad de filas posibles es , = 2 = 128 con, por simetría, × = 448 casillas y 448 casillas . Por tanto, hay 128 − 115 = 13 filas que no aparecerán, y deberán ser las que contengan menos casillas . Las describimos: • la fila , sin casillas . • las filas con una y seis : , = ! !× ! = 7 filas, que contienen 7 × 1 = 7 casillas en total. • 13 − 1 − 7 = 5 de las filas con dos y cinco que serían, en total, , = ! !× ! = 21. Las cinco filas de este tipo contienen 5 × 2 = 10 casillas en total. La mínima cantidad de casillas que no estarían en el tablero, de todas las variaciones posibles, sería 0 + 7 + 10 = 17 por lo que la cantidad máxima de casillas que puede haber es 448 − 17 = 431
  • 51. Se considera un número n de cuatro cifras, cuadrado perfecto, tal que todas sus cifras son menores que 6. Si a cada cifra se le suma 1, el número resultante es otro cuadrado perfecto. Halla n. SOLUCIÓN Sea = = ⇒ 1000 + 100 + 10 + = Además, 1000 × + 1 + 100 × + 1 + 10 × + 1 + + 1 = ⇒ − = 1000 + 100 + 10 + 1 Es decir, − = 1111 = 101 × 11 ⇒ + × − = 101 × 11 ⇒ + = 101 − = 11 , única distribución posible para que los cuadrados sean de cuatro cxifras. + = 101 − = 11 ª ª ª ª 2 = 90 ⇒ = 45 2 = 112 ⇒ = 56 ⇒ = = 45 = 2025
  • 52. Sean a y b dos números naturales tales que Determina el mayor valor posible de a/b. SOLUCIÓN = ⇒ + 5 = + 15 − 2 − 30 ⇒ 10 − 2 − 30 = 0 ⇒ 5 = + 15 ⇒ = Entonces, = = = + ⇒ = + Evidentemente, el mayor valor de la fracción corresponderá al menor valor posible de , teniendo en cuenta que tanto él como = son números naturales por lo que debe ser = 5 ⇒ = = 4 En resumen, = 4/5
  • 53. Sea un cuadrado ABCD de lado 1 y un cuadrado interior de lado 1/2. Halla el radio de la circunferencia que es tangente a dos de los lados del cuadrado ABCD y que pasa por un vértice del cuadrado interior. SOLUCIÓN Llamamos a la longitud del radio de la circunferencia. Observamos que la diagonal del cuadrado grande vale, aplicando el teorema de Pitágoras, en el triángulo : = √1 + 1 = √2 Aplicando el teorema de Pitágoras en , = + = + = = √ = √ = Aplicando el teorema de Pitágoras en , = √ + = √2 = √2 × Con todo lo anterior, = + + = √ + + √2 × = √2 ⇒ + √2 × = √ ⇒ = √ × √ ⇒ ⇒ = √ × √ × √ × √ = √ = (2 – √2)/2 = 0,2929
  • 54. Sea ABC un triángulo tal que el ángulo A es el triple del ángulo B Si BC = 5 y CA = 3, calcula la medida del lado AB SOLUCIÓN Sea = el valor buscado y = ⇒ = 3 Aplicando el teorema de los senos, = ⇒ 3 × sen 3 = 5 × sen ⇒ ⇒ 3 × sen 2 × cos + cos2 × sen = 5 × sen ⇒ ⇒ 3 × 2 × sen × cos × cos + cos − sen × sen = 5 × sen ÷ !!!" ⇒ 3 × 2 × cos + cos − sen = 5 ⇒ 3 × 3 × cos − 1 + cos = 5 ⇒ 4 × cos − 1 = ⇒ ⇒ 4 × cos = + 1 = % ⇒ cos = % & = ⇒ cos = ' Aplicaremos ahora el teorema del coseno. Para ello, calculamos ( = 180° − − = 180° − − 3 ⇒ ⇒ ( = 180° − 4 , y, entonces, = 3 + 5 − 2 × 3 × 5 × cos ( = 9 + 25 − 30 × cos 180° − 4 ⇒ ⇒ = 34 + 30 × cos 4 Por otro lado, cos 2 = cos − sen = 2 × cos − 1 = 2 × − 1 = - − 1 = & Y cos 4 = cos 2 − sen 2 = 2 × cos 2 − 1 = 2 × . & / − 1 = 0 − 1 = − 1 0 Resultando así que = 34 + 30 × cos4 = 34 + 30 × .− 1 0 / = 34 − 12 ⇒ = ⇒ = ' = 3,266
  • 55. Sea f una función tal que • f(n) = 0 si las cifras de las unidades de n es 4 • f(a×b) = f(a) + f(b) Calcula f(2019) SOLUCIÓN Como 6 × 4 = 24, 6 + 4 = 6 × 4 = 24 ⇒ 6 + 0 = 0 ⇒ 6 = 0 Entonces, 2019 × 6 = 12114 ⇒ 2019 + 6 = 2019 × 6 = 12114 ⇒ 2019 + 0 = 0 ⇒ ⇒ 2019 = 0
  • 56. Sea P el número que se obtiene al multiplicar los factoriales de los primeros 2008 enteros positivos: Halla uno de estos factoriales tal que, eliminándolo, la multiplicación de los 2007 factoriales restantes sea un cuadrado perfecto. SOLUCIÓN Si tenemos en cuenta que ! × + 1 ! = ! × ! × + 1 = + 1 × ! ., el producto propuesto queda = 1! × 2! × 3! × … … … × 2007! × 2008! = 1! × 2! × 3! × 4! × … … … × 2007! × 2008! ⇒ ⇒ = 2 × 1! × 4 × 3! × 6 × 5! × … … … × 2006 × 2005! × 2008 × 2007! ⇒ ⇒ = 2 × 4 × 6 × … × 2006 × 2008 × 1! × 3! × 5! × … … … × 2005! × 2007! ⇒ ⇒ = 2 × 4 × 6 × … × 2006 × 2008 × , con = 1! × 3! × 5! × … … … × 2005! × 2007! De lo anterior se deduce que = 2 × 4 × 6 × … × 2006 × 2008 × = 2 × 1 × 2 × 3 × … × 1003 × 1004 × ⇒ = 2 × 1004! × ⇒ = 1004! × 2 × siendo el factorial a eliminar el 1004!
  • 57. Halla un número natural n tal que si a su expresión se le coloca un 2 por la izquierda y un 1 por la derecha, el número resultante sea igual a 33n SOLUCIÓN Si es el número de dígitos de incrementado en una unidad, 2 1 = 2 × 10 + 10 + 1 = 33 ⇒ ⇒ 2 × 10 + 1 = 23 ⇒ = × = … Es cuestión de ir dividiendo 2000 … entre 23 hasta que de un resto de 22: 2000 … 23 160 22 86 Esto significa que 2000 = 23 × 86 + 22 ⇒ 2000 + 1 = 23 × 86 + 23 ⇒ 2001 = 23 × 87 ⇒ = = 87 siendo 2871 = 33 × 87
  • 58. Halla los dígitos X, Y, Z, con X > Y > Z tales que la siguiente resta entre números de tres cifras sea correcta. SOLUCIÓN Con las condiciones expuestas se deduce, a partir de cada resta individual de derecha a izquierda que 10 + − = 10 + − 1 − = − 1 − = ⇒ 10 + − = = 9 − 1 − = ⇒ = 10 − 4 − 9 ⇒ = 5 = 9 2 = 8 ⇒ = 4 X=9; Y=5; Z=4
  • 59. Halla un número de tres cifras sabiendo que la suma de sus cifras es 9, el producto de las mismas es 24 y, además, el número leído de derecha a izquierda es 27/38 del número primitivo. SOLUCIÓN Sea el número El enunciado nos dice que + + = 9 × × = 24 = × , siendo , , dígitos = × ⇒ 38 × 100 + 10 + = 27 × 100 + 10 + ⇒ 2662 − 110 − 3773 = 0 ÷ ÷ 242 − 10 − 343 = 0 + + = 9 ⇒ = 9 − − , luego 242 − 10 − 343 = 0 ⇒ 242 − 10 × 9 − − − 343 = 0 ⇒ ⇒ 252 = 333 + 90 ⇒ = "#$% & ⇒ = + "#$% & ⇒ = + $"# % En resumen, ' = + $"# % × × = 24 , siendo , , dígitos Se obtiene, de lo anterior, que = 2 ⇒ = 2 + $× # % = 3 ⇒ = ( )×" = ( × = 4 En número es 342
  • 60. Si la escalera mecánica está detenida Antonio la sube en 30 segundos y si la escalera mecánica está funcionando una persona que no se mueve la sube en 60 segundos. Determina cuánto tarda Antonio en subir si la escalera funciona pero él camina sobre ella. SOLUCIÓN Llamamos a la longitud de la escalera y al tiempo pedido. La velocidad de Antonio subiendo la escalera que está en marcha será la suma de la velocidad de Antonio con la escalera parada y la velocidad de subida de la escalera: = 30 + 60 = 2 + 60 = 3 60 = 20 ⇒ = 20 ⇒ = 20 segundos
  • 61. Sea ABCD un rectángulo cuyos lados miden AB=a y BC=b. Dentro del rectángulo se trazan dos circunferencias tangentes exteriormente de manera que una es tangente a los lados AB y AD y la otra es tangente a los lados CB y CD. Calcula la distancia entre los centros de las circunferencias en función de a y b. SOLUCIÓN Dibujamos y señalamos puntos y líneas: es el centro de la circunferencia de radio y es el centro de la circunferencia de radio La longitud pedida es = = + que es la hipotenusa del triángulo rectángulo , siendo sus catetos = e = Observando las longitudes horizontales y las verticales se observa que + + = = + + = = ⇒ + = + = ⇒ = − = − Y por el teorema de Pitágoras en el triángulo rectángulo : = + ⇒ = − + − ⇒ ⇒ = − 2 + + − 2 + ⇒ − 2 × + × + + = 0 ⇒ ⇒ = + ± + − + = + ± √ + 2 + − − ⇒ = + ± √2 Es evidente que ≤ < + ⇒ < + , luego el valor pedido es + − √!
  • 62. Sea ABCD un cuadrado de lados AB=BC=CD=DA=16 y P un punto en el lado BC La recta perpendicular a AP trazada por A corta a la prolongación del lado CD en Q Si AP=20, calcula DQ SOLUCIÓN Está claro que los ángulos y son iguales porque sus lados son perpendiculares dos a dos: , y , Además = = 16 por lo que los triángulos citados son iguales: = = 20 Aplicando el teorema de Pitágoras en el triángulo , = − = 20 − 16 = 400 − 256 = 144 ⇒ = √144 = 12
  • 63. Se escriben las cifras de 2019 como sigue: Calcula cuántos dígitos se deben escribir para que la suma de los dígitos escritos sea 3900. SOLUCIÓN Como 2 + 0 + 1 + 9 = 12, en la primera presencia del año el valor de la suma es 12. En la segunda presencia es 12 × 2 = 24, en la tercera 12 × 3 = 36 y, sucesivamente, en la n-sima es 12 × = 12 La suma de los dígitos hasta la n-sima presencia es 12 + 24 + 36 + ⋯ + 12 = 12 × 1 + 2 + 3 + ⋯ + = = 12 × × ⇒ 6 × × + 1 = 3900 ⇒ 6 + 6 − 3900 = 0 ÷ + − 650 ⇒ = ±√ × ⇒ ⇒ = ±√ = ± , habiendo usado la fórmula de la suma de los n primeros términos de una progresión aritmética. Por lo tanto, en el contexto del problema, = = 25 La primera presencia del año exige 4 dígitos, la segunda 8, la tercera 12 y, sucesivamente, la n-sima 4 En ese momento, la cantidad de dígitos es 4 + 8 + 12 + ⋯ + 4 = 4 × 1 + 2 + 3 + ⋯ + = 4 × × Entonces, el número de dígitos es 4 × × = 2 × 25 × 26 = 1300
  • 64. Calcula la suma de los dígitos del número SOLUCIÓN = 10 − 2019 = 1 000. . .000 − 2019 = 999. . .999 7981 Por lo tanto, la suma de sus dígitos es 2015 × 9 + 7 + 9 + 8 + 1 = 18135 + 25 = 18160
  • 65. Un cuadrado ABCD se divide en dos cuadrados y tres rectángulos, como se muestra en la figura. El área de cada uno de los cuadrados es a y el área de cada uno de los dos rectángulos más pequeños es b. Si a + b = 24 y la raiz cuadrada de a es un número natural, halla todos los valores posibles del área del rectángulo sperior. SOLUCIÓN Como es un cuadrado perfecto, llamamos a la longitud del lado de los cuadrados pequeños, valor entero positivo. De ahí, la longitud del lado del cuadrado grande es 2 y está claro que < . Como + = 24 = 16 ⇒ = √ = √16 = 4 La superficie buscada es, entonces Á ! − 2 × $ + % = $2 %& − 2 × $ + % = = 8& − 2 × 24 = 64 − 48 = 16 unidades cuadradas
  • 66. Se tiene un cubo de arista n, pintado de rojo y se divide el cubo en n3 cubitos de arista 1. La cantidad de cubitos que no tienen ninguna cara pintada es igual a 27 veces la cantidad de cubitos que tienen exactamente 2 caras pintadas. Halla n. SOLUCIÓN Evidentemente > 2 porque, en caso contrario, no habría cubitos interiores, sin pintar. La cantidad de cubitos interiores, sin pintar, es − 2 pues de cada cara hay que quitar los que están en la superficie. Los cubitos con dos caras pintadas son los que se encuentran en las 12 aristas, excepto los de las esquinas: 12 × − 2 Así, − 2 = 27 × 12 × − 2 ÷ ; − 2 = 27 × 12 = 3 × 3 × 2 = 3 × 2 ⇒ − 2 = 3 × 2 Es decir, − 2 = 18 ⇒ = 18 + 2 = 20
  • 67. Se pretende cubrir totalmente un cuadrado de lado k (k entero mayor que uno) con los siguientes rectángulos: 1 rectángulo de 1x1, 2 rectángulos de 2x1, 4 rectángulos de 3x1, ... , 2n rectángulos de (n+1)x1, de tal manera que los rectángulos no se superpongan ni excedan los límites del cuadrado. Halla los valores de k para los que esto es posible y, para cada valor de k encontrado, dibuja una solución. SOLUCIÓN Se trata de encontrar > 1 entero tal que = 1 × 1 × 1 + 2 × 2 × 1 + 4 × 3 × 1 + ⋯ + 2 × + 1 × 1 ⇒ ⇒ = 2 × 1 + 2 × 2 + 2 × 3 + ⋯ + 2 × + 1 Tenemos en cuenta la suma de los primeros términos de una progresión geométrica de razón : × Como 2 , 2 , 2 , … , 2 es una progresión geométrica de ! − # + 1 términos de razón 2, su suma es entonces $×% &'$( ) = 2 − 2 De ahí, = 2 × 1 + 2 × 2 + 2 × 3 + ⋯ + 2 × + 1 = = 2 + 2 + 2 + ⋯ + 2 + 2 + 2 + ⋯ + 2 + 2 + ⋯ + 2 + ⋯ + 2 + 2 + 2 ⇒ ⇒ = 2 − 2 + 2 − 2 + 2 − 2 + ⋯ + 2 − 2 + 2 − 2 ⇒ ⇒ = 2 × + 1 − 2 + 2 + 2 + ⋯ + 2 = 2 × + 1 − 2 − 2 ⇒ ⇒ = 2 × + 1 − 1 + 2 ⇒ = 2 × + 1 Ahora bien, = 2 × + 1 ⇒ − 1 = 2 × ⇒ + 1 × − 1 = 2 × y impar y, como 2 > , deberá cumplirse que * + 1 = 2+ − 1 = 2, × -, donde . > / y . + / = + 1 y obsérvese que la diferencia entre los dos factores es de dos unidades. Esta igualdad se cumple únicamente para = 3 y = 7: 1 + 1 = 22 − 1 = 2 × 3 -, por lo que k = 7 y una solución es
  • 68. Toni sumó cinco números naturales consecutivos y el resultado que obtuvo es el número de cinco cifras 1x84x, con el dígito de las unidades x igual al de las unidades de mil. Determina los cinco números que sumó Toni y da todas las posibilidades. SOLUCIÓN Sean los números , + 1, + 2, + 3, + 4 Según el enunciado, + + 1 + + 2 + + 3 + + 4 = 1 84 ⇒ 5 + 10 = 10000 + 1000 + 840 + , por lo que 5 = 10830 + 1001 ⇒ = ⇒ = 2166 + 200 + Como es un dígito y un número natural, los únicos valores admisibles son: = 0 ⇒ = 2166 + 200 × 0 + ⇒ = 2166 = 5 ⇒ = 2166 + 200 × 5 + ⇒ = 3167 y las quíntuplas posibles son 2166, 2167, 2168, 2169, 2170 3167, 3168, 3169, 5170, 3171
  • 69. A cada número entero positivo n de dos cifras se le resta la suma de los cuadrados de sus dígitos. ¿Para qué valores de n esta diferencia es la mayor posible? SOLUCIÓN Sea el número de dos cifras genérico = 10 + con la cifra de las decenas y la de las unidades. Es evidente que ≥ y la igualdad solo ocurre cuando = 0 o = 1. Será entonces, en estos casos, cuando el valor propuesto = 10 + − − sea, en cada decena, mayor. Por lo tanto, hay que analizar el valor = 10 + − − = 10 + − − = 10 − con unidades = 0 o = 1. Planteando la función cuadrática = 10 − , como = 10 − 2 = 0 si = 5 y " 5 = " = −2 < 0 ⇒ ⇒ tiene un máximo en = 5, luego este valor da diferencia máxima cuando = 0 o = 1. En conclusión, la diferencia máxima posible es = 5 = 10 × 5 − 5 = 25 con los números n = 50 y n = 51
  • 70. Un tren viaja de Zaragoza a Barcelona, con dos paradas intermedias, primero Lérida y después Tarragona. Cuando se detiene en Lérida, la cantidad de pasajeros que sube es igual a 3/4 de los pasajeros que viajaron de Zaragoza hasta Lérida, y bajan 39 pasajeros. En la estación de Tarragona, la cantidad de pasajeros que sube es igual a 3/4 de los pasajeros que viajaron de Lérida hasta Tarragona, y bajan 39 pasajeros. La cantidad de pasajeros que llegaron a Barcelona es igual a la cantidad de pasajeros que salieron de Zaragoza. Averigua cuántos pasajeros salieron de Zaragoza. SOLUCIÓN Sea la cantidad de pasajeros que salieron de Zaragoza y llegaron a Barcelona. De Lérida salen + − 39 = − 39 pasajeros, y de Tarragona salen − 39 + × − 39 − 39 = = × − 39 − 39 pasajeros, por lo que × − 39 − 39 = ⇒ − − 39 = ⇒ ⇒ 49 − 1092 − 624 = 16 ⇒ ⇒ 49 − 16 = 1092 + 624 ⇒ 33 = 1716 ⇒ = = 52 pasajeros Curiosamente, siempre va en el tren la misma cantidad de pasajeros: 52
  • 71. La víctima de un accidente morirá a menos que reciba en los próximos 10 minutos una transfusión de sangre tipo A-Rh positivo. Se dispone de gran número de donantes de los cuales sólo se sabe que el 40% tienen sangre de ese tipo. Se necesitan dos minutos para determinar el tipo de sangre del posible donante y dos minutos para realizar la transfusión. ¿Cuál es la probabilidad de que se salve si el hospital dispone de un sólo equipo de tipificación de sangre? SOLUCIÓN Hay un 40% de donantes adecuados y un 60% de donantes no adecuados. La probabilidad de que en los dos primeros minutos se identifique un donante bueno es = La probabilidad de que se encuentre al donante entre los minutos dos y cuatro es × = × = Que sea entre los minutos cuatro y seis × × = × × = Que sea entre los minutos seis y ocho × × × = × × × = Inmediatamente después de la localización se realiza la transfusión hasta, como mucho, el minuto diez. La probabilidad de que se salve es + + + = = = 0,8704 = 87,04 %
  • 72. Sea ABCD un cuadrilátero de lados AB, BC, CD y DA tal que ^ABC=90o , ^ACD=90o y BC=CD. Las diagonales AC y BD se cortan en O. Si ^AOD=110o , calcula ^BAC. SOLUCIÓN Sea = el ángulo a determinar, en el triángulo rectángulo El triángulo es isósceles pues = ⇒ = ° 2 × + = 180° ⇒ ⇒ 2 × + + 90° = 180° ⇒ 2 × + = 90° [ ] Por otro lado, en el triángulo , + + = 180° ⇔ + + = 180° ⇒ ⇒ + 110° + = 180° ⇒ + = 70° [ ] Restando, [ ] − [ ]: = 20° % & ° 20° + = 70° ⇒ = 50° Y, en el triángulo rectángulo , = 90° − = 90° − 50° = 40o
  • 73. Para la construcción de una pista circular de patinaje sobre hielo, se tienen las propuestas de dos empresas. La empresa Ice&Co cobra 40 euros por m2 de pista, 30 euros el metro de cerco y un adicional fijo de 400 euros para gastos generales. La empresa WinterSports cobra 36 euros por m2 de pista, 40 euros el metro de cerco y un adicional fíjo de 1200 euros. ¿Para qué valores del diámetro de la pista es más ventajoso contratar la empresa Ice&Co? SOLUCIÓN Sea el radio, en metros, de la pista de hielo que se desea construir. La empresa Ice&Co cobrará = 40 × × + 30 × 2 × × + 400 = 40 + 60 + 400 euros y la empresa WinterSports cobrará = 36 × × + 40 × 2 × × + 1200 = 36 + 80 + 1200 euros por el trabajo. Habrá un valor concreto del radio en el que el coste sea el mismo. Lo calculamos: 40 + 60 + 400 = 36 + 80 + 1200 ⇒ 4 − 20 − 800 = 0 ⇒ = ± × ⇒ ⇒ = ±√ !"# = √ = ×$ % & ⇒ = ' × (1 + $1 + ) = 10,86 m Para valores de < 10,86: , 10 = 40 × 10 + 60 × 10 + 400 = 4600 + 400 = 14851,33 € 10 = 36 × 10 + 80 × 10 + 1200 = 4400 + 1200 = 15023,01 € 0 ⇒ ⇒ < : más ventajoso el contrato con Ice&Co Para valores de > 10,86: , 11 = 40 × 11 + 60 × 11 + 400 = 5500 + 400 = 17678,76 € 11 = 36 × 11 + 80 × 11 + 1200 = 5236 + 1200 = 17649,38 € 0 ⇒ ⇒ > : más ventajoso el contrato con WinterSports Es decir, que el contrato con Ice&Co es más ventajoso para valores del diámetro menores de 2 = 2 × 10,86 = 21,72 metros
  • 74. En una reunión de 152 científicos, algunos son matemáticos y los demás son físicos. El promedio de las edades de todos los científicos es de 41 años, el promedio de las edades de los matemáticos es 35 años, y el promedio de las edades de los físicos es 51 años. Determina cuántos científicos de esta reunión son matemáticos. SOLUCIÓN Sean , la cantidad respectiva de matemáticos y de físicos existentes en la reunión: + = 152 La media de edades de los matemáticos es á = 35 ⇒ á = 35 La media de edades de los físicos es í " = 51 ⇒ #í = 51 O sea, la suma de todas las edades de los científicos es 35 + 51 , por lo que la media de edades de los científicos es $% &%'" '%( = 41 ⇒ 35 + 51 = 41 × 152 = 6232 Entonces, , + = 152 35 + 51 = 6232 - 'ª→%'×'ª 0111112 , 51 + 51 = 7752 35 + 51 = 6232 - 'ª4(ª 0112 16 = 1520 ⇒ = '%(6 '7 = 95 matemáticos
  • 75. Sean x y d números naturales tales que el resto de dividir x por d es igual a 4 y el resto de dividir 14x por d es 17. Halla el resto de dividir 210x por d. SOLUCIÓN = × + 4 14 = × + 17 ⇒ 14 × + 4 = + 17 ⇒ 14 + 56 = + 17 ⇒ − 14 = 56 − 17 ⇒ ⇒ × − 14 = 39 La única factorización de 39, aparte de la trivial, es 39 = 13 × 3, pero > 17 ⇒ ⇒ = 39 − 14 = 1 Por lo tanto, 14 = + 17 ⇒ 14 = 39 + 17 × !!!!!!" 210 = 15 × 39 + 17 = 39 × 15 + 255 Como 255 = 39 × 6 + 21 ⇒ 210 = 39 × 15 + 39 × 6 + 21 ⇒ 210 = 39 × 15 + 6 + 21 y el resto pedido es 21
  • 76. En la expresión Agustín reemplazó cada * por un signo + o un signo – de modo que quedaron 5 signos de cada clase, y realizó la expresión indicada. El resultado es un número positivo de dos dígitos que es múltiplo de 7. Determina qué número obtuvo Agustín e indica una posible asignación de los signos + y – con la que se obtiene ese número. SOLUCIÓN La suma de los diez primeros números naturales es 55: 1 + 2 + 3 + 4 + 5 + 6 + 7 + 8 + 9 + 10 = 55 Por lo tanto, el número múltiplo de 7 resultante de la operación debe ser 49 = 7 × 7 ≥ ≥ 7 × 2 = 14 Analizamos cada caso: • = 7 × 7 = 49. Como 55 − 49 = 6, debería haber 5 números negativos cuya suma fuera = 3 para, así, la suma de los demás positivos sería 55 − 3 = 52 y el total 52 − 3 = 49. Imposible porque no hay cinco números que sumen −3 • = 7 × 6 = 42. Como 55 − 42 = 13, debería haber 5 números negativos cuya suma fuera , lo cual es imposible por no ser un número entero. • = 7 × 5 = 35. Como 55 − 35 = 20, debería haber 5 números negativos cuya suma fuera = 10 para, así, la suma de los demás positivos sería 55 − 10 = 45 y el total 45 − 10 = 35. Imposible porque no hay cinco números que sumen −10 • = 7 × 4 = 28. Como 55 − 28 = 27, debería haber 5 números negativos cuya suma fuera , lo cual es imposible por no ser un número entero. • = 7 × 3 = 21. Como 55 − 21 = 34, debería haber 5 números negativos cuya suma fuera = 17 para, así, la suma de los demás positivos sería 55 − 17 = 38 y el total 38 − 17 = 21. Los cinco números que sumasen −17 podrían ser −1 − 2 − 3 − 4 − 7 = −17 • = 7 × 2 = 14. Como 55 − 14 = 41, debería haber 5 números negativos cuya suma fuera , lo cual es imposible por no ser un número entero. En resumen, el número que obtuvo Agustín es 21 con esta posible operación: – 1 – 2 – 3 – 4 + 5 + 6 – 7 + 8 + 9 + 10 y otra posible operación es: – 1 – 2 – 3 + 4 – 5 – 6 + 7 + 8 + 9 + 10
  • 77. Se dispone de 98 tarjetas y en cada una de ellas está escrito uno de los números 1, 2, 3, ..., 98 (no hay números repetidos). Ordena las 98 tarjetas de modo tal que, al considerar dos tarjetas consecutivas, la diferencia entre el número mayor y el número menor escritos en ellas sea siempre mayor que 48. SOLUCIÓN Es evidente que la tarjeta número 49 debe iniciar o terminar la secuencia, pues solo hay otro número entre los indicados que se diferencie, con él, un valor superior a 48: es 98, que debe ir a su lado. A partir de ahí, por lógica, una secuencia adecuada es 49-98-48-97-47-96-46-95-… … …-5-54-4-53-3-52-2-51-1-50
  • 78. Sea ABCD un cuadrado de lado 6. Sean P en el lado BC y Q en el lado CD tales que las rectas AP y AQ dividen al cuadrado en tres figuras de áreas iguales. Calcula el área del triángulo APQ SOLUCIÓN Como las tres partes son iguales los triángulos rectángulos y son iguales: = Sea = = El área del triángulo es la tercera parte del área del cuadrado: × = × ⇒ ⇒ = × 6 ⇒ 3 = 12 ⇒ = 4 Entonces, Á = Á = 3 ! Á = 12 ⇒ Á + Á = 12 ⇒ Á + # = 12, pues $ = $ = 6 − = 6 − 4 = 2 Á + 2 = 12 ⇒ Á = 10 unidades cuadradas
  • 79. Determina todas las funciones f tales que cualesquiera que sean los números reales x, y. SOLUCIÓN Consideramos − + 2 + 1 = + × − , ∀ , ∈ Si = = 0, − + 2 + 1 = + × − 0 − 0 + 1 = 0 × 0 ⇒ ⇒ 0 = 1 ⇒ 0 = ±1 Si = , − + 2 + 1 = + × − − + 2 + 1 = 2 × 0 ⇒ ± 2 + 1 = 2 × ±1 ⇒ 2 = ± 2 + 1 , ∀ ∈ ⇒ = ± + 1 , ∀ ∈ Verificando ahora las dos expresiones: • − + 2 + 1 + × − + 1 − + 1 + 2 + 1 = − + 2 + 1 + + 1 × − + 1 = − + + + − + 1 ⇒ ⇒ − 1 − + 1 + 2 + 1 = − + 2 + 1 + + 1 × − + 1 = − + 2 + 1 ⇒ Iguales. Ok • − + 2 + 1 + × − ! ! − − 1 − − − 1 + 2 + 1 = − + + 2 + 1 − − − 1 × − + − 1 = − + + + − + 1 ⇒ ⇒ − 1 − + 1 + 2 + 1 = − + + 2 + 1 − − − 1 × − + − 1 = − + 2 + 1 ⇒ Distintos Las funciones son, entonces, f(x) = x + 1, con x real
  • 80. Angelines escribe una lista de números naturales. El primer número es el 1; luego escribe los múltiplos de 2, desde 2 hasta 22 ; a continuación escribe los múltiplos de 3, desde 3 hasta 32 ; luego los múltiplos de 4, desde 4 hasta 42 , y así siguiendo hasta escribir, por primera vez, el 2019. La lista empieza de la siguiente manera: Determina cuántos números tiene la lista de Angelines. SOLUCIÓN Para cada , la lista hasta consta, exactamente, de números. Como 2019 = 3 × 673, el número 2019 aparecerá en primer lugar como tercero de la lista de los factores de 673. Por lo tanto, la lista constará de todos los correspondientes de 1 a 672 más los tres factores citados: 1 + 2 + 3 + 4 … + 672 + 3 =⏞ é ó é !1 + 672" × 672 2 + 3 = 673 × 336 + 3 = 226131 números
  • 81. Halla el menor entero positivo n tal que las 73 fracciones sean todas irreducibles. SOLUCIÓN Evidentemente, debe ser impar. Si es par hay 36 fracciones, al menos, simplficables. Como las fracciones son del tipo , si = − 2 la fracción es = = = siempre reducible. En conclusión, debe ser > 91 − 2 = 89 e impar. Además, como el primer número lo obtendremos cuando + 2 sea primo: = 95 Si = 91, = = es reducible Si = 93, = = es reducible
  • 82. La aldea de Astérix comparte un banquete de camaradería a escote, con precio fijo, en honor a Obélix, que es invitado y no paga. El coste total asciende a 1680 denarios y hay que pagarlo. Dividen entre el número de participantes que pagan pero el dinero no alcanza porque 4 de ellos se han marchado, así que cada uno de los presentes debe agregar 1 denario. Calcula cuántos comensales hubo en el almuerzo. SOLUCIÓN Sea el número inicial de comensales. El gasto de cada uno debe ser euros. Como faltan 4 para pagar, el gasto de los − 4 que quedan debe ser de + 1 euros: − 4 × + 1 = 1680 ⇒ 1680 + − − 4 = 1680 ⇒ − 4 − 6720 = 0 ⇒ ⇒ = 2 ± √2 + 6720 = 2 ± √6724 = 2 ± 82 En el contexto del problema, el número de participantes que debían pagar en el almuerzo es = 2 + 82 = 84 y, con Obélix, 85
  • 83. Las mesas de la biblioteca están numeradas correlativamente. ¿Cuál es el número de la mesa en donde está leyendo Blanca? SOLUCIÓN Leyendo los números de las mesas desde la posición de Blanca, su mesa es la número 67
  • 84. Se eligen cinco números naturales distintos, a, b, c, d, e, ordenados de menor a mayor: 1≤a<b<c<d<e. Se calcula el mínimo común múltiplo de cada número con el siguiente: mcm(a, b); mcm(b, c); mcm(c, d); mcm(d, e); y, finalmente, se efectúa la suma de sus inversos ¿Cuál es el máximo valor que puede tener el resultado final S ? SOLUCIÓN Cuanto más pequeños sean los valores de los “mínimo común múltiplo” más grandes serán las fracciones y, por tanto, . Es evidente que debe ser = 1; = 2. A partir de ahí si pretendemos que = 3, para que haya un mínimo común múltiplo mcm , lo más pequeño posible debe ser = 6. Por el mismo razonamiento, debe ser = 12 para que los denominadores sean los más pequeños posibles y, así, sea quizás el mayor valor posible: = , + , + , + , = , + , + , + , ⇒ ⇒ = + + + = = Haciendo los números sucesivas potencias de 2 conseguimos un valor mayor, que es el mayor posible: = , + , + , + , = , + , + ,! + !, ⇒ ⇒ = + + ! + = ! = 15/16
  • 85. Se tienen dos figuras superpuestas: el cuadrado ABCD de lado 6 y el triángulo isósceles ABE de base AB, con AE=BE y E fuera del cuadrado. Si el área de la superposición es igual a 3/4 del área del cuadrado, calcula el área de la porción del triángulo que no se superpone con el cuadrado. SOLUCIÓN Se trata de calcular la superficie del triángulo isósceles : × , siendo la longitud de su base y ℎ la de la altura. La superficie del cuadrado es 6 = 36 unidades cuadradas. Como el área de superposición (el trapecio sombreado ) es del área del cuadrado, × = × 36 ⇒ 3 + 18 = 27 ⇒ 3 = 9 ⇒ = 3 unidades lineales. Los triángulos y , por lo que se puede establecer la proporción = ⇒ = ⇒ = ℎ ⇒ ⇒ ℎ + 6 = 2ℎ ⇒ ℎ = 6 unidades lineales. En resumen, la superficie buscada es × = × = 9 unidades cuadradas
  • 86. La figura adjunta está formada por tres cuadrados iguales, en color naranja, y tres rombos iguales, en color azul. Halla los ángulos de los rombos. SOLUCIÓN Llamamos al ángulo de los rombos correspondiente al vértice común de los cuadriláteros. Como los cuadrados tienen ángulos de 90° ⇒ 3 + 3 × 90° = 360° ⇒ 3 = 360° − 270° = 90° ⇒ = 30° El otro ángulo de los rombos es 180° − = 180° − 30° = 150° En resumen, los rombos tiene los ángulos de medidas 30o y 150o
  • 87. En el cuadrado ABCD, sea P en el lado AB tal que AP2 =BP×BC y sea M el punto medio de BP. Si N es el punto interior del cuadrado tal que AP=PN y MN es paralelo a BC, calcula la medida del ángulo ^BAN. SOLUCIÓN Sea la longitud del lado del cuadrado y = = ⇒ = − . Según el enunciado, = × ⇒ = − × ⇒ − + = 0 ÷ ⇒ − + = 0 ÷ − 1 + = 0. Haciendo = ⇒ + − 1 = 0 ⇒ = √ ! ⇒ = √" , y solo tomando el valor positivo por su construcción como cociente de longitudes. Como = − × ⇒ − = = Entonces, # = $% = &% &$ = ⇒ # = + # = + = + ' ⇒ # = × 1 + ' En el triángulo rectangulo # , # = − # = − = − ' = − ' ! ⇒ ⇒ # = × 1 − ' ! ⇒ # = × (1 − ' ! En resumen, tan , = -. &- = ×( / 0 × / = ( / 0 / = ( / × / ( / ⇒ tan , = 1 / / = ( ' ' = 1 √234 √234 ⇒ ⇒ tan , = ( " √" 5 √" = ( 6" √"7×65 √"7 65 √"7×65 √"7 = ( 8 9×√" ! ⇒ tan , = :5 − 2 × √5 ⇒ , = 36o
  • 88. Una empresa maderera obtuvo un contrato para cortar árboles de un bosque y los ecologistas iniciaron una protesta en su contra. Para evitar las protestas, el gerente de la empresa agregó la siguiente cláusula al contrato: “En el bosque, el 99% del total de árboles son pinos, y la empresa sólo cortará pinos. Cuando se termine el contrato, el 97% del total de árboles del bosque serán pinos.” Determina qué porcentaje del bosque será cortado por la empresa al cumplirse esta cláusula del contrato. SOLUCIÓN Llamamos al número total de árboles existentes y al número de pinos que se cortarán. La cantidad de pinos existentes es de × 99% = Si cortamos pinos quedarán − que deben ser el 97% de los árboles que quedan: − . Por lo tanto, − = − × 97% = × ⇒ 99 − 100 = 97 − 97 ⇒ 2 = 3 ⇒ = × O sea, se cortarán los del bosque: el 66,67 % del bosque
  • 89. Del entero positivo n se sabe que: • n no es múltiplo de 5 • n – 96 es múltiplo de 128 • n tiene 2001 dígitos • todos los dígitos de n son pares • la suma de los dígitos de n es 2×2001 – 4 = 3998 • la suma de los cuadrados de los dígitos de n es 4×2001 = 8004 Halla n. SOLUCIÓN Sean , , … , los dígitos de , todos pares. Según el enunciado, + + ⋯ + = 3398 = 2 × 2001 − 4 + + ⋯ + = 8004 = 4 × 2001 Tomando los valores mitades de los dígitos, + + ⋯ + = 2001 − 2 = 1999 + + ⋯ + = 2001 , de donde se deduce inmediatamente que debe haber, como mitades de dígitos, 1997 unos, 1 dos y 3 ceros. En consecuencia, los dígitos del número son 1997 doses, 1 cuatro y 3 ceros. Por otro lado, el enunciado dice que − 96 = 128 ⇒ = 128" + 96 ⇒ = 32 × #4" + 3$ Es múltiplo de 32: sus cinco últimas cifras deben formar un número múltiplo de 32 Las dos últimas cifras (teniendo en cuenta las que tiene y que no es múltiplo de 5) que le permiten ser múltiplo de 4 son 04 y 24, por lo que acaba en 4 De esas terminaciones, únicamente 024 y 224 le permiten ser múltiplo de 8: acaba en 24 De las anteriores terminaciones, 0224 y 2224 le permiten ser múltiplo de 16: acaba en 224 De las anteriores terminaciones, 00224 y 20224 le permiten ser múltiplo de 32: acaba en 0224 Entonces, Si acabase en 20224, ninguna combinacón de dos dígitos más por delante #02, 20, 22$ y restándole 96 da un múltiplo de 128 Si acabase en 00224, = 32 × #4" + 3$ = 10& ' + 00224 = 10& ' + 7 × 32 = 10& ' + #4 + 3$ × 32, por lo que debería ser 10& ' = 32 × 4( y ' debería ser múltiplo de 4. La única posibilidad, con las cifras que quedan, es que acabe en 20 Por lo tanto. El número debe ser ) = *** … … ***+,,,-,,,. //& 01232 444**5
  • 90. Sea ABC un triángulo acutángulo. Se considera el punto D del lado AB tal que CD es perpendicular a AB, y el punto E del lado AB tal que CE es la bisectriz del ángulo ^ACD. Sea F el punto del lado BC tal que ^BAF = ^ACE, y G el punto de intersección de AF y CE. Si se sabe que el triángulo CFG es equilátero, calcula los ángulos del triángulo ABC. SOLUCIÓN Dibujamos el esquema del problema y llamamos = = = Como el triángulo es equilátero, sus tres ángulos miden 60° En el triángulo rectángulo , = 90° − = 90° − Por lo tanto, = 180° − = 180° − 90° − ⇒ = 90° + En el triángulo , + + = 180° + 90° + + = 180° ⇒ = 90° − 2 Al ser opuestos por el vértice, = ⇒ 60° = 90° − 2 ⇒ 2 = 30° ⇒ = 15° En el triángulo , + + = 180° ⇒ + 90° + + = 180° $ %&° = 90° − 2 = 90° − 30° ⇒ ⇒ = 60° (*) Observemos que = + = + 60° $ %&° = 15° + 60° ⇒ = 75° (**) En el triángulo , + + = 180° ⇒ 60° + + 75° = 180° ⇒ = 180° − 60° − 75° ⇒ = 45° (***) De (*), (**), (***) se concluye que A = 60o , B = 45o , C = 75o
  • 91. Juan midió el largo del terreno de su tío con pasos de 54 cm. Después midió el tío con pasos de 72 cm. Quedaron marcadas en total 61 pisadas pero, a veces, la misma marca correspondía a dos pisadas, una de Juan y otra del tío. ¿Cuál es el largo del terreno? SOLUCIÓN mcm 54 , 72 = mcm 2 × 3 , 2 × 3 = 2 × 3 = 216, por lo que el largo del terreno es un múltiplo de 216: 216 metros. Por lo tanto, Juan da = 4 pasos y su tío da = 3 pasos, coincidiendo en las pisadas veces además de la pisada inicial. Por lo tanto, el total de pisadas visibles será 4 + 3 − + 1 = 61 ⇒ 6 = 60 ⇒ = 10, y el largo del terreno es 216 = 216 × 10 = 2160 metros
  • 92. Miguel hizo la lista de todas las progresiones aritméticas de números enteros positivos tales que la diferencia es igual a 3 y la suma de sus términos es igual a Calcula cuántas progresiones tiene la lista de Miguel. SOLUCIÓN La progresión aritmética se caracterizará por el primer término, , la diferencia, 3, y el número de términos, > 1. Último término es + 3 × − 1 = + 3 − 3, por lo que la suma de todos los términos es × = 2010 ⇒ 2 + 3 − 3 = 4020 ⇒ 2 = − 3 + 3, valor entero y par Como 4020 = 2 × 3 × 5 × 67, probamos por los factores divisores de 4020 Observamos que, salvo el signo, la expresión debe ser entera y par, por lo que si es impar la fracción debe ser par, lo que siempre sucede y si es par la fracción debe ser impar, para que 2 sea par. • impar o = 3 ⇒ 2 = − 3 × 3 + 3 ⇒ 2 = 1334 ⇒ = 667 … … … … … … … … = 3; = 667 o = 5 ⇒ 2 = − 3 × 5 + 3 ⇒ 2 = 792 ⇒ = 396 … … … … … … … … = 5; = 396 o = 3 × 5 = 15 ⇒ 2 = − 3 × 15 + 3 ⇒ 2 = 226 ⇒ = 113 … … … = 15; = 113 o = 67 ⇒ 2 = ! − 3 × 67 + 3 ⇒ 2 < 0, contradictorio con el enunciado o > 67 ⇒ < 0, contradictorio con el enunciado • par o = 2 = 4 ⇒ 2 = − 3 × 4 + 3 ⇒ 2 = 996 ⇒ = 498 … … … … … … = 4; = 498 o = 2 × 3 = 12 ⇒ 2 = − 3 × 12 + 3 ⇒ 2 = 302 ⇒ = 151 … … … = 12; = 151 o = 2 × 5 = 20 ⇒ 2 = − 3 × 20 + 3 ⇒ 2 = 144 ⇒ = 72 … … … = 20; = 72 o = 2 × 3 × 5 = 60 ⇒ 2 = − 3 × 60 + 3 ⇒ 2 < 0, contradictorio con el enunciado o > 60 ⇒ < 0, contradictorio con el enunciado En total hay 6 progresiones aritméticas en la lista
  • 93. Halla el área de la zona sombreada de la figura, que está comprendida entre el triángulo equilátero y la semicircunferencia de radio 1. SOLUCIÓN Dibujamos los segmentos auxiliares que se ven, nombramos vértices y señalamos los datos del problema. Por los valores, se deduce inmediatamente que los cuatro triángulos, en los que queda dividido el triángulo equilátero original , , , , son equiláteros de lado unidad. Es evidente también que los tres segmentos circulares , , tienen la misma superficie y esta se obtiene restando el área del respectivo sector circular menos el área del triángulo equilátero asociado. Entonces, el área pedida es = Á − Á + Á + Á ! #$%#&'(% ) $ &(' &* %+$#$ ,----------------. = Á + Á ⇒ ⇒ = Á + Á 01 − Á 2 %$á &'! 4&$'á %! ) $ &(' &* %+$#$ ,----------------. = Á 01 Es decir, la superficie buscada es igual a la de uno de los sectores circulares: = Á 01 = 5×78×90° ;90° = π/6 = 0,5236 unidades cuadradas
  • 94. En un rectángulo de 1 x 101, dividido en casillas cuadradas de 1 x 1, Guille escribió un número entero en cada casilla de manera tal que la suma de los tres números escritos en tres casillas consecutivas era siempre igual a 9. Luego borró todos los números escritos excepto el de la tercera casilla y el de la décima casilla, contadas de izquierda a derecha: un 3 y un 10 respectivamente. Halla el número que había escrito Guille en la última casilla. SOLUCIÓN Señalamos las primeras casillas con valores desconocidos: Se cumple que + + 3 = 9 + 3 + = 9 3 + + = 9 ⇒ = = , por lo que se repite la secuencia numérica continuamente: Se deduce inmediatamente que = 10, luego + + 3 = 9 ⇒ 10 + + 3 = 9 ⇒ = −4 y se rellena la tabla con el número 10 en las casillas 3 − 2, con el número −4 en las casillas 3 − 1 y con el número 3 en las casillas 3 , ∀ ∈ , hasta la casilla 101 = 3 × 34 − 1: El número que escribió Guille en la última casilla es – 4
  • 95. Sean p=2×3×5×7×11×13×... el producto de todos los números primos hasta 2020 y q=3×5×7×9×11×13×... el producto de todos los números impares hasta 2020. Halla la penúltima cifra de la derecha del producto p×q. SOLUCIÓN El producto constará de un solo número par, el 2, y, el resto, una cantidad notable de números impares de los cuales más de uno acaba en 5. Por uno cualquiera de estos últimos y el 2, el producto × acaba en 0 y, debido al resto de múltiplos de 5 y de solo impares, acaba en 50 por lo que el penúltimo número de la derecha deberá ser un 5
  • 96. Félix hizo la lista de todos los números de 7 dígitos distintos que se forman con los dígitos 1, 2, 3, 4, 5, 6 y 7. Si los números de la lista de Félix están ordenados de menor a mayor, calcula qué posición de la lista ocupa el número 3654721. SOLUCIÓN Delante del citado número estarán • todos los que empiezan por 1 y por 2: 2 × • todos los que empiezan por 31, 32, 34 y por 35: 4 × • todos los que empiezan por 361, 362 y por 364: 3 × • todos los que empiezan por 3651 y por 3652: 2 × • todos los que empiezan por 36541 y por 36542: 2 × • todos los que empiezan por 365471: En total, 2 × + 4 × + 3 × + 2 × + 2 × + = 2 × 6! + 4 × 5! + 3 × 4! + 2 × 3! + 2 × 2! + 1! = = 2 × 720 + 4 × 120 + 3 × 24 + 2 × 6 + 2 × 2 + 1 = 1440 + 480 + 72 + 12 + 4 + 1 = 2009 números delante del indicado. Por lo tanto estará en la posición 2009 + 1 = 2010
  • 97. Halla las tres últimas cifras del número SOLUCIÓN Como 19 = 20 − 1, según el binomio de Newton, 19 = 20 − 1 = × 20 × 1 − × 20 × 1 + ⋯ + × 20 × 1 − × 20 × 1 Si observamos, todos los términos, salvo los cuatro últimos, tendrán un factor que contendrá una potencia de 20 con exponente mayor o igual a 4 por lo que esos términos finalizarán en, al menos, cuatro ceros, lo que no afectará al problema que nos ocupa teniendo en cuenta que, al haber restas, el primer término de esos cuatro sí que interviene en la solución final. Por lo tanto calculamos los cuatro últimos términos del desarrollo anterior, cuyo resultado nos permitirá obtener las tres últimas cifras de la potencia propuesta: + 97 94 × 20 × 1 − 97 95 × 20 × 1 + 97 96 × 20 × 1 − 97 97 × 20 × 1 = = + 97! 94! × 3! × 8000 − 97! 95! × 2! × 400 + 97! 96! × 1! × 20 − 97! 97! × 0! × 1 = = + 97 × 96 × 95 6 × 8000 − 97 × 96 2 × 400 + 97 × 20 − 1 = = +97 × 16 × 95 × 8000 − 97 × 48 × 400 + 97 × 20 − 1 = 1177659539 Las tres últimas cifras son 539
  • 98. Sean A, B y C tres puntos en una recta r con B entre A y C, y sea D un punto exterior a r. Se traza la recta paralela a r por el punto D que denominamos s. Se traza la bisectriz del ángulo ^ABD que corta a la recta s en P y se traza la bisectriz del ángulo ^CBD que corta a la recta s en Q. Si BP = 12 y BQ = 5, calcula BD. SOLUCIÓN Dibujamos todo el esquema propuesto y nombramos = a la longitud pedida. Es evidente que, por el teorema de Thales, = = y = = , por lo que los triángulos y son isósceles y = = = ⇒ = 2 . Como 2 + 2 = 180° ⇒ + = 90° ⇒ el triángulo es rectángulo en y, por el teorema de Pitágoras, = + ⇒ 2 = 12 + 5 = 169 ⇒ 2 = √169 = 13 ⇒ = = 13/2 = 6,5 unidades lineales
  • 99. Sean A y B dos números naturales de tres cifras cada uno. Si se escriben las cifras de A a continuación de las cifras de B se obtiene un número de seis cifras que es igual a 6 veces el número de seis cifras que se obtiene al escribir las cifras de B a continuación de las de A. Halla A y B. SOLUCIÓN El enunciado dice que = 6 × , escribiendo los nuevos números de seis cifras concatenando los dos de tres. Entonces, = 6 × ⇒ 1000 × + = 6 × 1000 × + = 6000 × + 6 × ⇒ ⇒ 994 × = 5999 × ⇒ 2 × 7 × 71 × = 7 × 857 × ÷ ⇒ 2 × 71 × = 857 × Como 857 es primo, y primo con 2 × 71 = 142, se deduce que A = 142 y B = 857
  • 100. Un kiosco vende pastillas de chocolate a 40 céntimos de euro cada una, caramelos a 2 por 10 céntimos y bombones a 10 céntimos cada uno. Marta gastó 8,8 euros en 44 golosinas que repartió en partes iguales (de cada variedad) entre sus 4 hijos sin que le sobrase nada. ¿Cuántas golosinas y de qué clases le correspondieron a cada uno? SOLUCIÓN Como Marta tiene 4 hijos y reparte equitativamente, en las golosinas de cada hijo gastó , = 2,2 euros y le dio = 11 golosinas. Llamamos , , a la cantidad respectiva de pastillas de chocolate, caramelos y bombones que le da a cada uno. Según el enunciado, + + = 11 40 + 5 + 10 = 220 ÷ + + = 11 8 + + 2 = 44 ª ª 7 + = 33 ⇒ = 33 − 7 Las posibilidades son: • = 1 ⇒ = 33 − 7 = 33 − 7 = 26 ⇒ = 11 − − = 11 − 1 − 26 < 0: imposible • = 2 ⇒ = 33 − 7 = 33 − 14 = 19 ⇒ = 11 − − = 11 − 2 − 19 < 0: imposible • = 3 ⇒ = 33 − 7 = 33 − 21 = 12 ⇒ = 11 − − = 11 − 3 − 12 < 0: imposible • = 4 ⇒ = 33 − 7 = 33 − 28 = 5 ⇒ = 11 − − = 11 − 4 − 5 = 2 • = 5 ⇒ = 33 − 7 = 33 − 35 < 0: imposible • Para valores superiores de obtendríamos siempre valores negativos de Por tanto, a cada hijo le repartió 4 pastillas de chocolate 2 caramelos 5 bombones
  • 101. Se dan dos rectángulos iguales: ABCD y APQR, tales que P está en el interior del rectángulo ABCD y el lado PQ del rectángulo APQR intersecta al lado DC del rectángulo ABCD en el punto E. Si se sabe que: • AB=CD=AP=QR=8 • AD=BC=AR=PQ=12 • DE=1, halla el área de la figura ABCEP. SOLUCIÓN La superficie pedida (rellenada en naranja) será igual ala superficie del rectángulo menos las superficies de los triángulos rectángulos y : Á = Á − Á − Á En el triángulo rectángulo aplicamos el teorema de Pitágoras: = + = 12 + 1 = 144 + 1 = 145 En el triángulo rectángulo aplicamos el teorema de Pitágoras: = + ⇒ 145 = 8 + ⇒ ⇒ = 145 − 64 = 81 ⇒ ⇒ = √81 = 9 Entonces, Á = Á − Á − Á ⇒ ⇒ Á = × − × − × = 8 × 12 − × − ×! = 96 − 6 − 36 = 54 unidades cuadradas